You are on page 1of 128

Defamation

The type of fault that a plaintiff must prove will depend upon the status of the plaintiff.

Public Figures:

Some plaintiffs are public figures. A person becomes a public figure by achieving a pervasive
fame or notoriety or by voluntarily assuming a central role in a specific public controversy.
Those who voluntarily assume a central role in a specific public controversy are often described
as "limited-purpose public figures."

Classifying the plaintiff as a public figure is important because to win a defamation case, public
figures must prove a higher degree of fault. Specifically, public figures (limited and otherwise)
must prove "actual malice" which is defined as knowledge that the statement made about
plaintiff was false at the time the statement was made or a reckless disregard as to the falsity of
the statement. This is a subjective test so it will not be helpful to ask whether a reasonable person
would have or should have known that the statement was false. The mindset of the actual
defendant here is what matters.

Private Figures:

Unlike with public figures, for a private person to prove defamation only negligence is required.
In this context that means that a reasonable person would have or should have realized that the
statement made by defendant was false. Unlike the test for a public figure, this is an objective
test. Negligence is only constitutionally required, however, if the statement is a matter of public
concern. If the statement is not a matter of public concern then constitutional restrictions will not
apply but state law might still require a showing of some degree of fault such as negligence.

Also worth noting is that although a private figure is required to prove negligence in situations
involving a matter of public concern, it might still be in the interest of a private figure to go the
step further and prove actual malice. Where negligence is proven, only actual injury damages
(damages to reputation, etc.) are recoverable by plaintiff. But if actual malice is proven, damages
may be presumed and punitive damages are likewise recoverable.

Hearsay Exceptions

On the MBE, you'll need to distinguish between hearsay exceptions and hearsay exemptions. The
reason for this is straight-forward: one answer choice might read "non-hearsay" (which indicates
that a hearsay exemption applies) and another might read "hearsay but an exception" (which
indicates that a hearsay exception applies).

Prior Statements by Testifying Witnesses:

The first category of hearsay exemptions falls under the umbrella of statements by a testifying
witness. If a prior statement is inconsistent with the testifying witness's in-court testimony and if
that prior statement was made under oath at a prior proceeding, the statement will be deemed
non-hearsay as an exemption and may be offered for its truth. Note that if that prior statement by
the witness wasn't made under oath it can still be used to impeach the witness but because it
wasn't made under oath it will not be deemed an exemption to the hearsay rule and will not be
admissible to prove the truth of the matter asserted.

In addition, a prior statement that is consistent with a testifying witness's in-court testimony and
is offered to rebut a charge that the witness is lying or exaggerating or offered to rehabilitate a
witness whose credibility has been impeached on some other ground will be admissible as an
exemption to the hearsay rule.

Lastly, a prior statement of identification of a person as someone the witness perceived earlier
even if the witness cannot now remember the identification is properly admissible as an
exemption to the hearsay rule.

Statements (or admissions) of a Party Opponent:

When one party offers a statement made by an opposing party, that statement is non-hearsay
under the Federal Rules. There are a few to look out for:

--Judicial admissions: Statements made in pleadings, stipulations, etc., are all admissions when
offered by an opposing party.

--Adoptive admissions: If a party remains silent when a reasonable person would have responded
in the face of an accusation, the silence may be considered itself as an implied admission but
only if the party to be charged with the admission heard and understood the accusation and was
both physically and mentally capable of denying it. It also must be true that a reasonable person
would have denied the accusation. Importantly, silence in the face of an accusation by police in a
criminal case is very rarely considered an admission by silence.

Vicarious Admissions:

Because there are quite a few of these, I consider them their own category.

--Authorized Spokesperson: Statements of a person authorized by a party to speak on that party's


behalf can be admitted as a non-hearsay exemption against the party.

--Principal-Agent: Statements by an agent or employee concerning any matter within the scope
of the agency or employment made while the agency or employment relationship exists are non-
hearsay.

--Partners: After a partnership exists, an admission of one partner relating to matters within the
scope of the partnership business is admissible as a non-hearsay statement.

--Co-Conspirators: Admissions of one co-conspirator made to a third party in furtherance of a


conspiracy to commit a crime or civil wrong at a time when the declarant was participating in the
conspiracy are admissible as a non-hearsay exemption against all co-conspirators.

And one last point to note: Before admitting any statement as one of the above non-hearsay
vicarious admissions, the court must make a preliminary determination of the declarant's
relationship with the party against whom the statement is offered to ensure that a relationship
exists that would warrant the exemption.
The Uniform Prudent Investor Act

Not the most exciting of legal topics here but if you review the trust essays that have appeared on
the UBE in the past, this pops up from time to time. The idea here is that in a "prudent investor"
jurisdiction, only prudent investments are permissible regardless of the trust's terms.

There are similarities between the standards here and the standards learned when studying
negligence. A trustee must exercise reasonable care, skill, and caution when investing and
managing trust assets. But a trustee with special skills or expertise who has represented him/her
self as having such skills will have to use such skills or expertise when investing.

Further, a trustee must act exclusively for the beneficiary when investing and managing trust
assets. If there are multiple beneficiaries, the trustee must act impartially never benefiting one or
more than one at the expense of others.

Importantly, investment decisions must be evaluated in the context of the entire trust portfolio
and as part of an overall investment strategy. In other words, risk is often inherent when
investing and so the fact that certain investments might seem risky should not lead to a
conclusion that the trustee has not acted with prudence, especially if those risky investments
were balanced with safer investments. A trustee should always diversify investment unless it is
reasonably determined that the purpose of the trust is better served absent diversification.

When analyzing this issue of compliance or non-compliance with the Prudent Investor Act, there
are certain elements to address. Some of those elements are as follows: general economic
conditions; the possible effect of inflation or deflation; the expected tax consequences of
investment decisions or strategies; the role that any individual investment plays within the
overall investment strategy; the expected total return from income and appreciation of capital;
other resources for the beneficiaries; needs of the beneficiaries for liquidity and regularity of
income, etc.

A trustee who acts in substantial compliance with the Prudent Investor Act will not be held liable
for non-compliance. A trustee may also delegate investment decisions but in doing so the trustee
must act prudently in selecting an agent, establishing the scope and terms of the delegation, and
reviewing the agent's investment decisions.

Res ipsa loquitur

Res ipsa loquitur directly translates to "the thing speaks for itself." It doesn't always do as good
a job of explaining itself and so I'll attempt to do that here.

Res ipsa loquitur should be considered when analyzing the tort of negligence; specifically, when
addressing whether a duty of care has been breached. Generally, a breach occurs when
defendant's conduct falls short of the standard of care required of any specific defendant.

There are a variety of theories that plaintiff might use to prove that defendant breached the duty
of care required of that specific defendant. Perhaps defendant skewed from the custom that
reasonable people generally adhere. It should be noted though that custom is not dispositive and
it's especially important to note that just because defendant adhered to custom doesn't end the
inquiry. After all, the custom itself might not be reasonable.
Another theory to prove breach is violation of a statute. Existence of a duty owed to plaintiff and
the violation of that duty may be established by proof that defendant violated a statute. This is
known as "negligence per se" and it merely establishes breach. Causation and damages must still
be proven.

Sometimes, however, there is no direct evidence of an act that might be used to establish breach
of the duty of care. Instead, circumstantial evidence of negligence is required. When there is no
direct evidence, you should consider res ipsa loquitur. It's a doctrine that allows for the inference
of negligence from the very nature of the accident or injury. In other words, the fact that the act
or injury occurred may in some circumstances tend to establish that a duty was breached.

There are specific elements to infer breach of duty based on res ipsa loquitur. Plaintiff must show
that the accident causing the injury is a type that would not normally occur unless someone were
negligent. The logic here is that because the injury would not normally have occurred unless
someone had been negligent, it's reasonable to infer that someone was negligent since the injury
did occur. Plaintiff must also show that the inferred negligence is attributable to defendant and
this is often proven by the fact that defendant was in exclusive control of the instrumentality that
caused the injury.

If res ipsa loquitur is established so that breach of the duty of care is inferred, plaintiff has then
established a presumption of negligence so that no directed verdict may be granted for defendant
on that issue. This isn't a win for plaintiff, though; the presumption or inference of negligence
may still ultimately be rejected.

MODIFICATON OF TRUSTS

When I work on essay writing with students, they have the option to choose which subjects to
focus on throughout the essays. Somewhere near the top of the list is Trusts. It's a difficult
subject that is tested often. A topic that shows up with some frequency is the modification of a
trust. A trust can be modified by the settlor, by the beneficiaries, by the court, or by the trustee.

The Settlor: A settlor can modify a trust unless the terms expressly state that the trust is
irrevocable. Some states hold that a trust is irrevocable unless expressly stated otherwise, but this
is not the rule on the Uniform Bar Exam which follows the Uniform Trust Code.

The Beneficiaries: A trust may be modified upon the consent of the settlor and all beneficiaries
even if such modification or termination conflicts with a material purpose of the trust. If only the
beneficiaries consent without the consent of the settlor, the trust may still be modified but only if
no material purpose of the trust would be frustrated by the modification. If there is no material
purpose frustrated and all beneficiaries consent to termination of the trust, the trustee must upon
termination of the trust distribute the trust property to the beneficiaries.

The Court: Assume here that modification by the beneficiaries alone is not available since not
all beneficiaries have consented to modify the trust. A court may still modify if the trust could
have been modified had all beneficiaries consented (no material purpose would have been
frustrated) and if the interests of any non-consenting beneficiaries will be adequately protected.
In addition, a court may also modify a trust if unanticipated circumstances threaten the purpose
of the trust, continuation of the existing trust is impracticable, or if the value of the trust is
insufficient to justify the cost of administering the trust.

The Trustee: A trustee can modify (terminate) a trust if the trust purpose is less than $50,000
and the amount is insufficient to justify the cost of administering the trust as long as the trustee
provides the beneficiaries with notice. A trustee can also modify a trust by combining several
trusts into one trust or by dividing one trust into several trusts provided that doing so does not
frustrate any purpose of the trusts or impair the rights of beneficiaries. Notice to the beneficiaries
is required of such an intent to combine or divide trusts but obtaining consent from the
beneficiaries is not.

Discovery: Disclosure of Expert Testimony & Pretrial Disclosure

In a recent post, I posted about the initial disclosures required of parties as part of discovery.
This post will focus on an additional two types of required disclosure: disclosure of expert
testimony, and pretrial disclosure.

First, a party must disclose to other parties the identities of expert witnesses expected to be used
at trial. If the expert has been specially retained to provide expert testimony or if the expert
regularly gives expert testimony as an employee of the party then there is an additional
requirement that the disclosure of the expert include a report prepared and signed by the expert
stating the expert's qualifications, the opinions to be expressed, the basis of those opinions, and a
list of cases in which the expert has testified over the previous 4 years. Also required is the
compensation of the expert, among other items. If these additional disclosure rules do not apply
to a given expert then only the following must be disclosed: the identity of the expert; the subject
matter on which the expert is expected to present evidence; and a summary of the facts and
opinions to which the expert is expected to testify.

These disclosures must be made at the time directed by the court or in the absence of such
directives, any time period stipulated by the parties provided the time is at least 90 days before
trial. If the evidence is intended solely to rebut another party's disclosure of expert testimony, it
must be made within 30 days after disclosure of the evidence rebutted.

Also required are pretrial disclosures. At least 30 days before trial a party must disclose the
following: the witnesses the party expects to call at trial; the witnesses the party will call if the
need arises; the witnesses whose testimony will be presented by deposition along with a
transcript of the pertinent portions of the deposition; and a list of documents or exhibits the party
expects to offer or might offer if needed.

Within 14 days after this disclosure, the recipient party may serve objections to use of the
depositions at trial and to the admissibility of disclosed documents and exhibits. Such objections
are waived if not made in that time period except for objections that the evidence is irrelevant,
prejudicial, or confusing under Federal Rules of Evidence 402 and 403.
Discovery: Initial Disclosures
A party to a lawsuit must make a reasonable inquiry into the facts of the case and then disclose
all information then reasonably available that is not privileged or protected by work product.
Without waiting for a discovery request a party must make initial disclosures. Initial disclosures
include the following:
--The names, addresses, and telephone numbers of individuals likely to have discoverable
information that the disclosing party may use to support its claims or defenses unless the use
would be used solely for impeachment.

--Copies or descriptions of documents, electronically stored information, and tangible things that
are in the disclosing party's possession or control that the disclosing party may use to support its
claims or defenses, unless the use would be solely for impeachment.

--A computation of damages claimed by the disclosing party and copies of material upon which
the computation is based.

--Copies of insurance agreements under which an insurer might be liable for all or part of any
judgment that might be entered.

All of the above must be made within 14 days after the conference of the parties at which they
plan discovery unless a different time is set by court order or by stipulation.

Worth also noting that initial disclosures such as those stated above are not required in cases
such as actions to review an administrative record, actions to enforce an arbitration award, pro se
litigation brought by prisoners, actions to quash or enforce subpoenas, and habeas corpus
petitions.

Shareholder Voting
I often hear from students that Corporations is a dry subject. I can't deny that but it's also tested
with relative frequency on the essay portion of the UBE. One area you might be required to
know well is shareholder voting.

Voting generally takes place at shareholder meetings and only shareholders of record on the
record date may vote at the meeting. The record date may not be more than 70 days before the
meeting and if a record date is not set it is deemed to be the day the notice of the meeting is
mailed to the shareholders. Unless provided otherwise (as in, for example, the articles of
incorporation), each share held by a shareholder is entitled to one vote.

A shareholder need not vote his/her shares in person. Instead, the shareholder can execute a
proxy in writing which will entitled another to vote the shares. Proxies are valid for 11 months
unless provided otherwise and are generally revocable at the will of the shareholder. A proxy
will be irrevocable only if it so states and if it is coupled with an interest or given as a security to
another. There can be no fraud, material misstatements, or omissions in connection with the
solicitation of proxies.

For a vote to take place at a shareholder meeting, there must be a quorum. A quorum is usually a
majority of outstanding shares entitled to vote unless the bylaws of the corporation or articles of
incorporation require a greater number. Notably, once a quorum is present, it is not broken by
withdrawal of shares at the meeting.

If a quorum is present, shareholders will be deemed to have approved a matter if the votes cast in
favor of the matter exceed the votes cast against it. Once again, the bylaws or articles can adjust
this requirement by requiring a greater proportion. Directors, however, may only be elected by a
plurality of the votes cast.

As stated above, the general rule is that each share held by a shareholder is entitled to one vote.
The articles of incorporation may instead provide for cumulative voting when electing directors.
Under cumulative voting, each shareholder is entitled to vote a number of votes equal to the
number of the shareholder's voting shares multiplied by the number of directors to be elected.
That total can be divided among various directors or may be cast in total for just one director.

For example, assume a shareholder with 10 shares is participating in a vote for 2 open board
seats. Under a cumulative voting scheme the shareholder would have 20 votes to cast.

Worth noting is that in certain situations shareholders may take action without a meeting. To act
without a meeting though will require unanimous written agreement of all shareholders entitled
to vote.

Default Rules (Article 9 UCC)


For sure, three areas to know well for Article 9 UCC on the MEE are attachment, perfection, and
priorities. Another area that is tested that is often overlooked is default. Specifically, the rules
that determine the responsibility of the secured party as to the collateral after the debtor has
defaulted by not paying back the loan.

The secured party (the creditor) has some options. The secured party can sue on the debt itself,
take possession of the collateral (importantly, only if doing so will not be a breach of the peace),
or sell the collateral by a public or private commercially reasonable sale and then collect any
deficiency after sale.

In the case of a sale, the debtor as well as other secured parties (if any) are generally entitled to
notice. The notice must be sent within a reasonable time and must be detailed as to the parties,
collateral, time and method of sale, etc.

A detail worth noting is that if the debtor has paid 60% of the cash price on a purchase money
security interest ("PMSI") in consumer goods or 60% of the loan on a non-PMSI in consumer
goods, the secured party must dispose of the collateral within 90 days after re-possessing it or the
debtor will be entitled to recover it in conversion. This rule is very specific to consumer goods (a
type of collateral). With respect to any other type of collateral, the secured party may retain the
collateral in full satisfaction of the debt but only if the debtor consents to the retention in an
authenticated record after default or if the debtor (or any other secured party) does not object to
the retention within 20 days after notice is sent by the secured party.

Until the secured party has sold the collateral or has discharged the debt by retention of the
collateral, the debtor or other secured party may redeem the collateral by paying all obligations
plus additional reasonable expenses.

If a secured party fails to follow the requirements as to how to handle the collateral after the
debtor has defaulted, the secured party will be liable for actual damages caused by that failure
which could amount to entirely denying the secured party the right to collect a deficiency from
the debtor or allowing the secured party to recover a deficiency but subtracting from that amount
any actual damages that the debtor can prove has resulted from the secured party's failure to
follow the default rules.
Nonconforming Uses
I've written previously about certain topics on the subject of zoning that are showing up with
greater frequency on the MBE and on the MEE. This post will dig a bit deeper into one of those
topics: nonconforming uses.

The purpose of a zoning ordinance is to prevent certain land uses within the zoned area. These
ordinances are generally enacted by municipalities to control and direct the development of
property within their borders. But even those ordinances that prohibit a given use may allow
others to continue the now-prohibited use of their property if they were using their property in
the now-prohibited way prior to the ordinance taking effect.

These uses that are allowable notwithstanding the zoning ordinance are known as nonconforming
uses and are said to be justified both on grounds of fairness and practicality. The goal of the
nonconforming use doctrine is to protect prior investments. Problems arise when a person
attempts to take a prior use and change it in such a way that in effect a new investment is created.

The takeaway here is to remember that a nonconforming use cannot be extended or intensified in
ways that constitute a substantial change to the property. In contrast, insubstantial changes are
allowable and in general repairs to the property that render the property practicable for current
purposes will be deemed as insubstantial changes. Doubts as to whether a change is substantial
or insubstantial are generally resolved against the change, however.

The basis for disallowing substantial changes to the property that is claiming protection under the
nonconforming use doctrine is that the policy behind allowing nonconforming uses is aimed at
protecting investments undertaken prior to the zoning ordinance. But when the property
undergoes substantial change, it's more likely that the nature of the investment is changed in such
a way that the changes are intended to protect a future investment rather than a prior one. And
because the protection of future investments was never a policy driven by the doctrine, the use of
that property with the substantial changes will lose its status as a protected nonconforming use
and may then violate any zoning ordinance in place at that time.

Congressional Powers

The Taxing Power:

The essential requirement here is that either the tax bear some reasonable relationship to revenue
production or that Congress has the power to regulate the activity taxed.

The Spending Power:

Congress may spend to provide for the common defense and general welfare. This is quite broad;
spending may be for any public purpose, though the power may be limited by other
constitutional provisions. Worth noting that although Congress can tax and spend for the general
welfare, it cannot legislate for the general welfare. Watch out for that trap on the MBE.
The Commerce Power:

Congress has the power to regulate all foreign and interstate commerce. The federal law
regulating interstate commerce must either regulate the channels of interstate commerce or
regulate the instrumentalities of interstate commerce. In addition, the power extends to the
regulation of activities that have a substantial effect on interstate commerce. This may even
include intrastate activity if such activity is economic or commercial and there is a rational basis
for Congress to conclude that the activity in the aggregate substantially affects interstate
commerce.

The War Power:

Congress declares war, and raises/supports armies. It also provides for and maintains a navy.
Economic regulation during war and in the postwar period is likely to be upheld. Congress also
has the power over regulation of armed forced through the military courts and tribunals. These
military courts have jurisdiction over all offenses committed by persons who are members of the
armed services both at the time of the offense and when charged. American civilians, on the
other hand, may be tried by military courts only if warfare forces the federal courts to shut down.

The Investigatory Power:

The power of Congress to investigate is implied and investigation may be expressly or impliedly
authorized by the appropriate congressional house.

The Property Power:

Congress has the power to dispose of and make rules for territories and other properties of the
United States. This is an absolute power to dispose of property but note the rules of eminent
domain should be considered when property is taken. Although Congress has no federal police
power it does have police-type powers of the District of Columbia, federal lands, military bases,
and Indian reservations.

The Bankruptcy Power:

Congress has the power to establish uniform rules of bankruptcy but this power is nonexclusive.
States may legislate in the field as long as their laws do not conflict with federal law (as per the
Supremacy Clause).

The Postal Power:

The postal power is exclusive. Though Congress may place reasonable restrictions on the use of
the mails, it may not deprive any citizen or group of citizens of the general mail privilege.

The Power Over Citizenship:

Congress has plenary power over aliens. Aliens have no right to enter the United States but
resident aliens are entitled to notice and a hearing before they can be deported. Congress may not
take away the citizenship of any citizen (native born or naturalized) without consent.

The Admiralty Power:


Congress's admiralty power is exclusive unless it leaves maritime matters to state jurisdictions.

The Power to Coin Money:

Congress has the power to coin money and to fix standards for weights and measures.

The Patent/Copyright Power:

Congress has the power to control the issuance of patents and copyrights.

The Necessary and Proper Power:

Congress has the power to make all laws necessary and proper for executing any of the powers
mentioned above.

Interlocutory Appeals & Removal Jurisdiction

Interlocutory appeals can show up in a variety of contexts on the MBE. A difficult issue to look
out for is one that involves both interlocutory appeals and removal of a case from state court to
federal court.

Imagine a situation in which federal jurisdiction is based entirely on a claim arising under federal
law rather than on diversity of citizenship of the parties. Further assume that a 2nd claim is
added to the first that would not on its own have an independent jurisdictional basis but because
it arises out of the same transaction or occurrence as the first claim it would be allowable in
federal court under supplemental jurisdiction. The plaintiff brings the case in state court but the
case is properly removed by the defendant to federal court. The federal court then dismisses the
first claim (the federal claim) leaving only the 2nd claim (the claim that had no independent basis
for federal jurisdiction). The federal court then remands the 2nd claim back to state court from
which the case was removed. Plaintiff who now would prefer the case to remain in federal court
appeals the decision to remand the case and defendant moves to dismiss the appeal for lack of
appellate jurisdiction.

Plenty of interesting jurisdictional issues above but the purpose of this post is to focus on
whether the plaintiff's appeal of the decision to remand the case back to state court is proper. If
not, then the appellate court lacks jurisdiction to hear that appeal. And the answer is that it
depends.

Remand orders that are based on a defect in the removal procedure or a lack of federal
jurisdiction are not appealable. Those claims never should have been in federal court. In contrast,
if a district court chooses not to exercise supplemental jurisdiction over a claim and therefore
remands that claim back to state court, that decision was discretionary since it could have
exercised jurisdiction over the claim even though the federal claim that was the basis for
jurisdiction had been dismissed. That discretionary decision to remand is immediately appealable
for abuse of discretion.

As such, under the facts as stated here, the appellate court would not lack jurisdiction to hear the
appeal.
A tricky issue with a tricky distinction. Worth noting since it comes up in questions.

Forgery
The crime of forgery has been showing up in some of the released MBE questions so it's worth
noting the different angles that might be tested. Here's what I've gathered is important to know:

Forgery consists of the following:

Making or altering a writing with apparent legal significance so that it is false with intent to
defraud.

The falsity is apparently important; to be forgery one must represent that the writing is
something that it is not. It's not enough if the writing contains a misrepresentation. So, for
example, an intentionally falsified receipt might be forgery, but a receipt that is merely
inaccurate will not be.

In addition, the "apparent legal significance" element is one that is tested. So, a contract may be
forged, as may a will. But creating a document and then signing it as Abraham Lincoln or
creating a picture and then signing it as Pablo Picasso will not be forgery. Neither the painting
nor the writing has any legal significance.

Situations may arise in which the defendant fraudulently causes a third person to sign a
document that the third person does not realize he is signing. This, too, is forgery. Importantly,
though, if the third person knows he is signing a document then it is not forgery even if the third
person was fraudulently induced into signing it.

Uttering a forged instrument is slightly different than the crime of forgery. This consists of
offering as genuine an instrument that may be the subject of forgery and is false with the intent to
defraud.

Future Interests (Examples and Illustrations)

By far, the best way to understand future interests in through examples. But before working
through examples it's important to understand which future interests attach to which present
possessory estates.

Present possessory estate -----> Future interest.

Fee simple absolute -----> No future interest

Fee simple subject to a condition subsequent -----> Right of re-entry.

Fee simple determinable -----> Possibility of reverter

Fee simple subject to an executory interest -----> Executory interest


Life estate -----> Reversion or remainder

Examples:

(1)
From x to y and y's heirs.
--Y has a present estate called a fee simple absolute.
--X has nothing. X has given his entire interest to y and has therefore retained no future interest.

(2)
From x to y but if y fails to use the property for farming then x has the right to enter the
property and terminate y's interest.
--Y has a present estate called a fee simple subject to a condition subsequent.
--X has retained as a future interest called a right of re-entry (or a right of termination). If y
violates the condition, x can re-enter the property and terminate y's interest.

(3)
From x to y for so long as y uses the property for farming.
--Y has a present estate called a fee simple determinable.
--X has retained as a future interest a possibility of reverter. If y violates the condition, x need
not take any action. The interest will automatically then revert back to x.

(4)
From x to y for so long as y uses the property for farming and if y ceases to use the
property for farming then to z.
--Y has a present estate called a fee simple subject to an executory interest.
--Z has a future interest called an executory interest. If y violates the condition the property will
automatically shift to z.
--X has nothing. Between the interest given to y and z, x has retained no future interest.

(5)
From x to y for so long as y uses the property for farming and if y ceases to use the
property for farming then to z for so long as z uses the property for farming.
--Y has a present estate called a fee simple subject to an executory interest.
--Z has a future interest called an executory interest. But z's executory interest is determinable
since it is conditioned on z using the property for farming.
--X has retained a future interest called a possibility of reverter. If z violates the condition, the
property will automatically shift back to x.

(6)
From x to y for life.
--Y has a present estate called a life estate
--X has retained a future interest called a reversion. Once y's life is over, the property will shift
back to x. Unlike with a possibility of reverter, this is certain to happen.

(7)
From x to y for life and then to z.
--Y has a present estate called a life estate
--Z has retained a future interest called a remainder. When y's life ends, the property will pass to
z.
--X has nothing. Between the life estate to y and the remainder to z, x has retained no future
interest.

Personal Jurisdiction (Constitutional Requirements)

Personal jurisdiction can intimidate many studying for the bar exam but it's actually a straight-
forward issue when approached systematically. It all begins with minimum contacts.

Minimum Contacts:

Personal jurisdiction requires that the defendant at least have minimum contacts with the forum
state such that if the forum state exercises jurisdiction over the defendant, such jurisdiction
would be fair and reasonable. A commonly used test for assessing minimum contacts is to ask
whether the defendant purposefully availed himself of the privilege of conducting activities in
that state. If so, the defendant should know that he might be haled into court to defend himself in
that state. A complication can arise if the defendant manufactures a product in state x but then
sells it to another in state y. The defendant has then placed that product in the stream of
commerce; it's very possible the product might then end up in state z injuring someone in that
state. Generally, there is a need to show that the defendant has done more than merely placed the
product into the stream of commerce; rather, an intent to benefit from doing business in state z is
often required especially if there are no other contacts between the defendant and state z.
Similarly, merely maintaining a passive website from which those from the forum state can order
products is generally insufficient. A stronger case for jurisdiction over the defendant will be
found if the defendant actively targets the state through the website.

Relatedness of Claim to Conduct (Specific vs General Jurisdiction):

When analyzing minimum contacts, it's important to determine whether the claim for which the
defendant has been sued is related to the contacts that the defendant had with the forum state. If
the claim is related to the contacts with the forum, the court is more likely to find specific
jurisdiction as to that claim. But if the claim is unrelated to the contacts with the forum, then for
the court to hold that the defendant is subject to general jurisdiction in that forum, it must be true
that the defendant is "at home" in that forum. For example, a person is at home in the state in
which he is domiciled, and a corporation is at home in the state in which it is incorporated as
well as the state in which it has its principal place of business.

Fairness:

Contacts with the forum are not enough. The exercise of jurisdiction must not "offend traditional
notions of fair play and substantial justice." The court will consider any inconvenience to the
defendant by requiring the defendant to defend in the forum. Only, however, if the defendant is
put at a severe disadvantage due to the inconvenience will this factor hold any weight. Another
factor is the interest that the forum state has in providing redress to its residents. Also considered
is the interest of the plaintiff in obtaining convenient and effective relief. Judicial efficiency is
likewise considered.

Notice:

Assuming there are minimum contacts and that the fairness requirements above are satisfied, it is
also required that a reasonable method be used to notify the defendant of a pending lawsuit so
that the defendant may have an opportunity to be heard. And this requirement is the basis of the
service of process rules, many of which are also tested on the MBE.

The Wharton Rule


The Wharton Rule is an interesting one. When it comes up in practice questions students often
tell me they've heard of it or read about it but it hasn't stuck in memory. It's simple enough and
it's worth knowing how it relates to the crime of conspiracy.

First a quick review of conspiracy. A conspiracy under the common law required an agreement
between 2 or more persons with an intent to enter into the agreement and an intent by at least 2
persons to achieve the criminal objective of the agreement. In addition, under the common law
an overt act was required to provide some evidence of the agreement. An act of mere preparation
would suffice as the overt act.

When the Wharton Rule is implicated, the first step is to consider some crimes that cannot be
accomplished by just one person. A few that come to mind are bribery and adultery but there are
plenty others. Under the Wharton Rule, where two or more people are necessary for the
commission of a crime, there will be no conspiracy to commit that crime unless more parties
participate in the agreement than are necessary for the crime.

In other words, if 2 people agree to a transaction that would constitute the crime of bribery then
at least 3 people will be required to charge them with conspiracy to commit bribery. Or more
abstractly, if the commission of a crime requires n number of people, then conspiring to commit
that crime will require at the least n+1.

One exception to note is that the Wharton Rule does not apply to agreements with "necessary
parties not provided for" by the substantive offense. Thus, if, for example, there is a state statute
prohibiting the sale of narcotics and that statute imposes criminal liability on the seller but not on
the buyer, then both the buyer and the seller may be charged with conspiracy to sell narcotics
even though both parties are in fact required for commission of the offense.
Impeachment (Crimes vs. Prior Bad Acts)
At its core, impeachment is meant to raise issues as to the credibility of a witness. There are quite
a few ways to impeach a witness. Two methods that cause some confusion since there are
similarities and differences among them are impeachment by conviction of a crime and
impeachment by a prior bad act.

Impeachment by Conviction of a Crime:

A witness may be impeached by proof that the witness has been convicted of a crime. Felonies
involving dishonesty as well as felonies not involving dishonesty are allowable to impeach. It's
more difficult, however, to offer a felony not involving dishonesty since the court will have the
discretion to exclude it if the witness being impeached is a criminal defendant and the
prosecution has not shown that the conviction's probative value outweighs its prejudicial effect
or if the witness is not a criminal defendant and the court determines that the conviction's
probative value is substantially outweighed by its prejudicial effect. Note that the standard for a
criminal defendant provides a greater level of protection.

Also to note is that generally if more than 10 years have passed since the date of conviction or
from the date of release from confinement (whichever is later), a conviction for purposes of
impeachment will not be allowable. Juvenile convictions are likewise not allowable nor are
convictions obtained in violation of the defendant's constitutional rights. If a witness has been
pardoned, the conviction for which the witness was pardoned may not be used to impeach the
witness if the pardon is based on innocence or if the witness has not been convicted of a
subsequent felony.

To impeach a witness by conviction of a crime, a prior conviction may be shown be either direct
or cross examination. Introducing a record of the judgment is also allowable.

Impeachment by Prior Bad Act:

A witness may not have a crime on record but an attorney may want to impeach that witness on
some act that did not rise to the level of a convicted crime. This is allowable subject to
discretionary control of the trial judge. A witness may be interrogated with respect to an act of
misconduct (a "prior bad act") only if the act is probative of truthfulness. Importantly, this is only
allowable on cross examination, and the cross examiner must inquire about the act in good faith.

So, for example, it may be proper to ask a witness whether the witness had been terminated from
a reason job after the witness was caught embezzling money, since that act is probative of the
witness's likelihood of testifying truthfully. But asking whether the witness had been terminated
for harassing another employee would be improper for this purpose since the act of harassment is
not probative of truthfulness.

Extrinsic evidence to prove prior bad acts (proving the act with a method other than cross
examination) is not permitted. In addition, the cross examiner may not reference any
consequence the witness may have suffered as a result of the bad act.
Removal Jurisdiction
There are many fine details to know about the right that a defendant has to remove a case to
federal court. The difficulty is that the MBE tests all of them.

Not all cases brought in state court can be removed to federal court. The case must be one that
could have originally been brought in federal court. Only defendants can remove and if there is
more than one defendant, all defendants who have been properly joined and served must join in
the petition to remove. If the case is removed to federal court, venue will lie in the federal district
court embracing the place where the state action is pending.

Situations will arise when a case filed in state court contains a claim that arises under federal
law, and that claim is joined with state law. Assuming there is no diversity among the parties and
further assuming that supplemental jurisdiction does not apply to allow the federal court to hear
the state claim, the entire case can be removed to federal court but the state law claim must be
remanded back to state court. In another circumstance, there might be no federal question
involved and no diversity of citizenship among the parties. Removal to federal court will still be
permitted if the non-diverse parties are thereafter dismissed from the action so that diversity then
exists. After the dismissal of the non-diverse parties, removal is proper provided the time limits
for doing so have not expired.

An important point to note is that if the only basis for removing a case to federal court is
diversity of citizenship, the case will not be removable if any of the defendants is a citizen of the
state in which the state action was brought. Further, if diversity is the basis for removal, the case
may not be removed more than one year after it was commenced in state court unless the district
court finds that the plaintiff has acted in bad faith by acting in a way that was intended to prevent
the defendant from acquiring the knowledge that the case was removable.

A defendant seeking to remove must file a notice of removal in the federal district court in the
district and division within which the action is pending. A copy of the notice should be sent to
the other parties and to the state court in which the case was brought. This notice of intent to
remove must be filed within 30 days after defendant receives notice that the case has become
removable. If there are multiple defendants and if a later-served defendant files a notice to
remove, the earlier-served defendants may join in the removal even if the 30-day period has
expired for those earlier-served defendants.

The plaintiff can always file a motion to have a removed case remanded back to state court. A
case will be remanded if there is no federal jurisdiction. In addition, the federal court has the
discretion to remand a case to state court once all federal claims have been resolved.

Equitable Subrogation (mortgages)


Equitable subrogation in the context of mortgages doesn't show up with much frequency on the
MBE but it's given very little focus in many of the outlines even though it does show up on the
exam.

When a lender advances funds used to satisfy a senior mortgage, the doctrine of equitable
subrogation allows the lender to "step into the shoes" of the senior mortgagee. The significance
here is that it will allow the lender to obtain priority over all interests that were junior to the
senior mortgagee.

Some jurisdictions (the majority view) take the view that a lender will not take priority over liens
that were junior to the senior mortgage if the lender had actual knowledge of those liens at the
time that payment was made. The minority view is that either actual or constructive knowledge
of the junior liens is sufficient to avoid application of the doctrine. The Restatement (Third) of
Property, in contrast, treats notice or knowledge of the other liens as irrelevant, instead stating
that the determining factor is simply whether the lender satisfied the senior lien and whether
application of the doctrine will prevent unjust enrichment. The rationale here is that because the
lender is merely taking the place of the senior mortgagee, the junior mortgagees are in no worse
position than they would have been in had the doctrine not applied to give the lender that priority
over them.

It's enough for purposes of the MBE to recognize that this doctrine exists and to understand how
it can grant priority to a lender who pays off the debt owed to a senior mortgagee. And because
it's an equitable doctrine, equitable defenses such an unclean hands and laches should always be
considered.
Equitable Servitudes
I was surprised to find that I hadn't yet written about the topic of equitable servitudes here on the
blog since it shows up with some frequency on the MBE.

Generally, these servitudes are created in a writing that satisfies the Statute of Frauds. The
exception is for negative equitable servitudes which may be implied from a common scheme of
development. So, if a developer subdivides land and some but not all of the deeds to the
subdivisions contain negative equitable servitudes (servitudes that restrict the use of the land), all
may be bound provided there was both a common scheme of development and notice was
provided to those whom it is claim are bound by the servitude.

As to the common scheme of development, that will be found only if at the time that sales in the
subdivision began the developer had a plan that all parcels would be subject to the same
restriction. And as to notice, it may be actual or record notice but often on the MBE the notice is
a type of inquiry notice whereby the neighborhood appears to conform to common restrictions.

Assume that someone is bound by an equitable servitude. The next issue is to determine whether
that burden will run to the successors of the person bound. It will if the original parties that
agreed to the servitude also agreed that the servitude would be enforceable by and against
assignees. Also, the assignee of the promisor (the person who promised to be bound) must have
notice of the servitude, and the servitude must touch and concern the land. To touch and concern
the land requires that the servitude restricts the person burdened by the servitude in his/her use of
the land.

For the benefit of the servitude to pass on to assignees of the original parties it is merely required
that it was intended that the benefit would pass on and that the servitude touch and concern the
benefited property.

As with all issues of equity, a court will not enforce an equitable servitude if the person seeking
enforcement is violating a similar restriction on his/her own land (unclean hands) or if the
benefited party acquiesced in the violation by the burdened party. Likewise, a court will not
enforce an equitable servitude if the benefited party acted in such a way that a reasonable person
would believe the servitude was abandoned or if the benefited party fails to bring suit against the
violator within a reasonable time (laches). Lastly, the servitude will not be enforced if the
neighborhood has changed so significantly that enforcement would be inequitable.

To terminate an equitable servitude requires any of the following: a written release from the
benefit holder; merger of the benefited and the burdened land; or condemnation of the burdened
property.
Creation of a Trust
A good place to begin in the study of Trusts is with the creation of a trust. It's also a straight-
forward way to score a lot of points on an essay that tests this issue.

And so here are the elements, all of which should be addressed should this issue show up:

Capacity: First, you'll need a settlor (the person who creates the trust) with the capacity to create
it. The capacity required is the same as the capacity required to create a will. Look out for things
like undue influence, fraud, and duress. All of these will prevent a settlor from having the
necessary capacity and without the capacity, no trust is created.

Intent: The settlor with capacity must intend to create a trust. Under most circumstances a
writing is not required to prove intent but always keep in mind the Statute of Frauds when real
property is involved. The settlor must intend that the trust take effect immediately and not at
some future time. And, importantly, a settlor's expression of hope (rather than a direct
instruction) that the property be used in a certain way is not evidence that a trust was intended.
That's known as precatory language and such language cannot be used to create a trust.

Trustee: There's got to be a trustee. A trust, however, will not fail if the trustee dies, refuses to
accept performance, or resigns. The court will appoint a successor unless it is clear the settlor
intended the trust to continue only if that particular trustee served. The trustee must have
enforceable obligations and anyone who has the capacity to acquire and hold property for his/her
own benefit has the capacity to act as a trustee.

Beneficiaries: Beneficiaries enforce the trust and so without them there can be no trust (there
are exceptions for both honorary and charitable trusts). Any person capable of taking or holding
title to property can be a beneficiary of a private trust. Although notice to the beneficiary as to
the existence of the trust is not required, acceptance is required and it may be express or implied.
A beneficiary cannot be forced to accept an interest in the trust; rather, a beneficiary may
disclaim the interest by filing a written instrument stating an intent to disclaim.

Trust Property: Where there is no trust property there can be no trust since without property
there would be no obligations required by the trustee. The property must be existing property that
the settlor has the power to convey. A future interest will suffice, but an interest not yet in
existence will not.

Trust Purpose: There must be a valid trust purposes and a purpose will not be valid if it is
illegal, contrary to public policy, impossible to achieve, or intended to defraud the settlor's
creditors. If a condition attached to the trust violates public policy the trust might still be valid if
the settlor has expressed an alternative desire. In such a case, the condition can be ignored.
Amount in Controversy
The required monetary amount for purposes of diversity jurisdiction is a straight-forward issue.
Even so, there are many testable angles, and the MBE tests them all. It's worth knowing all of the
following when preparing for Civil Procedure:

Actions brought in federal court under diversity must be in excess of $75,000, exclusive of
interests and costs (more on this later). This amount in controversy is entirely determined by the
plaintiff's good-faith allegation. In other words, the complaint can be dismissed on this basis only
if it appears that there is no legal possibility of a recovery exceeding $75,000. And the fact that
$75,000 is not ultimately recovered is not dispositive on whether the claim was in good faith.

In determining what should be included in the claim of $75,000, interest and costs are excluded.
But it should be noted that interest that constitutes a part of the claim is included. Interest
excluded is interest payable by virtue of a delay in payment. Also included are attorneys' fees
that are recoverable by contract or by statute as are punitive damages claims permitted under
state substantive law.

Importantly, for purposes of satisfying the jurisdictional amount a plaintiff may aggregate all
claims against a single defendant. This is entirely different than a plaintiff attempting to
aggregate claims against multiple defendants; in those instances, aggregation is allowable only if
defendants are jointly liable to plaintiff. If instead there are several plaintiffs, aggregation is
allowable only if the plaintiffs attempting to aggregate are seeking to enforce a single right in
which they have a common or undivided interest. It's unlikely that multiple plaintiffs can
aggregate.

It'll also be important to understand how supplemental jurisdiction can affect the required
jurisdictional amount. Claims that do not meet the $75,000 requirement may still invoke
jurisdiction if there is at least one claim that does meet the requirement and if the claims that do
not meet the requirement arise from the same "common nucleus of fact" as the claim that
satisfies the requirement.

Anti-Lapse Statutes
In the most recent post I outlined the concept of ademption, an area in the subject of Wills that is
also seen on the MBE. Another area that creeps its way onto the test is anti-lapse statutes. The
concept is straightforward, but you should know it well:

As a review, ademption occurs when a testator leaves property to a beneficiary in a will but at
the time that the testator dies that property is no longer in the estate. Situations will also arise
when a testator leaves property to a beneficiary in the will and at the time that the testator dies
the beneficiary is no longer alive. How should this be addressed?

The general rule is that the gift lapses. But nearly all states have anti-lapse statutes that operate to
save the gift if the predeceasing beneficiary was in a specified degree of relationship to the
testator. States vary but often the beneficiary will need to be a descendant of the testator, a
grandparent of the testator, or a descendant of the testator's grandparents. In addition, it'll be
necessary that the beneficiary leaves descendants who survive the testator.

For example, assume that the testator leaves property in his will to his grandfather. His
grandfather is not alive when the testator dies and as per the general rule the gift left by the
testator to his grandfather would lapse. But because the grandfather is in a degree of relationship
often contemplated by anti-lapse statutes, instead of the gift lapsing the next step is to determine
whether the grandfather has any living descendants. If so, then rather than the gift lapsing, the
gift will pass to the grandfather's descendant(s).

Anti-lapse statutes apply unless there is a contrary provision in the will. In other words, if a
testator states in a will that the gift should go to "x" only if "x" survives the testator, then any
anti-lapse statute should be ignored. That gift will lapse if "x" is not alive when the testator dies.
Ademption
Wills is a very heavily tested subject on the MEE, the essay portion of the Uniform Bar Exam.
And, although far more limited, it also shows up (in Property questions) on the MBE. One topic
that appears on the MBE is ademption.

Ademption is implicated when a testator leaves property in a will but the property is no longer in
the testator's estate at the time of death. What to do in such a situation?

First is to note that ademption only applies to specific gifts in a will. A specific gift is a gift that
can only be satisfied by receipt of the specific property described in the will. If such a gift is no
longer in the testator's estate at the time of death, the gift is adeemed and the beneficiary who
was supposed to receive the gift receives nothing. If, however, some of the gift remains (for
example, a tract of land is left to the beneficiary and some of the land is sold prior to the
testator's death), then the beneficiary will receive whatever remains.

Also tested are gifts that are not adeemed. For example, a general gift is a gift in a will of a
specific dollar amount. If testator leaves a dollar amount to a beneficiary in a will and there is not
enough cash in the estate at the time of death to satisfy that gift, that general gift of cash is not
adeemed; rather, the gift will be satisfied by selling other assets.

There are a few additional important points to keep in mind: assume a specific gift of land is no
longer in the estate at the time of testator's death. Although the gift is adeemed, the beneficiary of
that gift may be entitled to proceeds from the sale of that land if the land sale contract was
executory at the time testator died. The beneficiary may also be entitled to casualty insurance
proceeds for loss of personal property that was left to the beneficiary in a will if the proceeds are
paid after the testator's death. Lastly, a condemnation award (if land is condemned after testator
dies) may go to the beneficiary in the will who was supposed to receive the condemned land as a
specific gift.

Analytical Approaches to Conflict of Laws

Conflict of Laws is a very convoluted subject. Perhaps the most convoluted on the UBE.
Thankfully, there has been a limit to how it's tested; usually it'll show up as one question in a set
of questions testing another subject. But it does show up, and it'll be important to understand
how the rules apply to a variety of subjects such as Torts, Property, Contracts, Family Law, etc.

But a really good start before diving into all those details is to understand the three analytical
approaches that apply to any choice-of-law issue. The approaches will vary depending upon the
subject-matter at issue, but the foundation used in deciding which law to apply in a given case
doesn't change.

The first approach is called the "vested rights approach of the First Restatement." If applying this
approach you'll first want to characterize the area of law (Torts, Contracts, Property, etc.). And
then next is to determine the particular choice-of-law rule that applies to that subject matter. For
example, in Torts the rule is to apply the law of the place where the wrong occurred and in
Contracts the rule is to apply the law of the place where the contract was made or performed.
Essentially, you're applying the law of the place where the parties' rights vested (where the event
occurred that was necessary to create the cause of action).

The second approach is called the "most significant relationship approach of the Second
Restatement." This approach directs you to apply the law of the state that has the most significant
relationship to the occurrence or transaction and the parties. In determining which state has the
most significant relationship, there are a variety of relevant factors including the following: the
needs of the interstate and international judicial system; the relevant policies of the forum; the
relevant policies of other interested states; the protection of justifiable expectations; the basic
policies underlying the particular area of law; the need for certainty, predictability, and
uniformity; and the ease of determining and applying the chosen law.

Lastly is "interest analysis." With this approach, we're directed to first start from the assumption
that the forum will apply its own law. Then we're told to consider whether the forum has any
interest in the litigation; if not, it's called a "false conflict" and the forum will apply the law of
the state other than the forum state. If the forum and the other state both have an interest then
there is a "true conflict" and the forum will reconsider its own policies. If the forum finds that it
has a legitimate interest it will apply its own law and if it finds that it is a "disinterested forum"
then it will dismiss the case if forum non conveniens is available. If it is a disinterested forum
and forum non conveniens is not available the forum will make its own judgment as to which
law should apply or it will apply the law of the state that most closely resembles its own law.

Quite confusing, but address these issues with the above approaches and there are plenty of
points to be scored.
\
Intestate Shares
Some of the material required for Wills has to do with situations in which the decedent left a will
at the time of death. But also required for the exam is to know the procedure when the decedent
had no will at the time of death or when the will is denied probate. This procedure, known as
intestate distribution, can become complex.

Generally, if there is no will of which to dispose all of the property, the surviving will take a
large percentage (1/3rd or 1/2 most commonly). Under the Uniform Probate Code, the surviving
spouse takes the entire estate provided that any descendants of the surviving spouse are also
descendants of the decedent. And if there are no descendants, the surviving spouse generally gets
the entire estate as well.

After the spouse receives a share, the children get a share, but the share will depend on the
method of distribution. In some states the children will take their share per capita with
representation which means that the property is divided into equal shares at the first generational
level at which there are living takers. Each living person at that level takes a share and the share
of each deceased person at that level passes to that person's issue by right of representation.

So, assume that D dies, leaving 5 children, V, W, X, Y, and Z. At the time that D died, V and W
were alive, but X, Y, and Z had already died. X had 2 children, C1, and C2, Y had one child,
C3, and Z had 3 children, C4, C5, and C6. Had all 5 of V, W, X, Y, and Z been alive, this would
be simple: V, W, X, Y, and Z would receive 1/5th of the estate. But here only V and W were
alive so only they will each get their 1/5 share.

As to X, Y, and Z, each of their children will get what each of X, Y, and Z would have gotten
had they been alive. X would have gotten 1/5, so C1 and C2 will get (1/5)/(2) =1/10. Y would
have gotten 1/5 so C3 will get 1/5. And Z would have gotten 1/5 so C4, C5, and C6 will each get
(1/5)/(3)= 1/15.

You can always check to make sure the math is correct since all shares should equal 1.

More importantly for purposes of the UBE (since this is the method used in the Uniform Probate
Code) is a method of distribution called per capita at each generational level. Once again, the
initial division of shares is made at the first generational level at which there are living takers.
But here, the shares of deceased persons at that level are combined and then divided equally
among the takers at the next generational level.

Using the same characters from above, assume again that V and W are alive at the time of D's
death, but that X, Y, and Z have already died. V and W will once again get their 1/5 share. But
this time, the 3 shares of X, Y, and Z (1/5th each) will be combined for a total of 3/5. That 3/5
share will then be distributed equally among C1, C2, C3, C4, and C5. In other words, C1, C2,
C3, C4, and C5 will all get an equal share of 3/5th or (3/5)/(5)=3/25.

Once again, a quick check will determine if all shares when combined equal 1.

Posted by Sean Silverman at 12:05 PM 0 comments


Email ThisBlogThis!Share to TwitterShare to FacebookShare to Pinterest
Labels: Wills
Tuesday, April 9, 2019
The Duty of Loyalty & Conflicting Interest Transactions
The most recent post focused on the duty of care for the directors in a corporation. The other
duty for directors that is likely to be tested in a Corporations essay is the duty of loyalty. This
duty is often tested within the scope of conflicting interest transactions.

First, it's important to know the circumstances under which there has been a potential violation
of the duty of loyalty. A director has a conflicting interest with respect to a transaction if the
director knows that the director or someone related to the director (spouse, parent, child,
grandchild, etc.) is either a party to the transaction or has a beneficial financial interest in the
transaction such that the interest would reasonably be expected to influence the director's
judgment if the director were to vote on the transaction. In addition, the duty of loyalty is
potentially breached if the director is also a director, general partner, agent, or employee of
another entity with which the director is transacting business and if the transaction is of such
importance to the corporation that it would in the normal course of business be brought to the
other members of the board of directors.

Often tested on the essays are situations in which the elements above are all satisfied and yet the
transaction moves forward. A conflicting interest transaction will not be enjoined if any of the
following is true:

--The transaction was approved by a majority of the directors (but must be at least 2) who did not
have a conflicting interest after all material facts have been disclosed to the board.

--The transaction was approved by a majority of the votes entitled to be cast by shareholders who
did not have a conflicting interest after all material facts have been disclosed to those
shareholders.

--The transaction, judged according to circumstances at the time of the transaction, was fair to
the corporation.

Fairness is quite a vague term and so specific factors are used to determine whether the
transaction is fair to the corporation. Courts looks to factors such as adequacy of consideration,
the corporate need to enter into the transaction, the financial position of the corporation, and any
reasonably available alternatives.

Posted by Sean Silverman at 10:35 AM 0 comments


Email ThisBlogThis!Share to TwitterShare to FacebookShare to Pinterest
Labels: Corporations & Limited Liability Companies
Thursday, April 4, 2019
The Business Judgment Rule (the duty of care)
I review old UBE essays regularly and among the essays testing the subject of Corporations, a
significant percentage of them test the business judgment rule. It's important to know it well.

There are two main duties that directors have when managing the corporation: the duty of loyalty
and the duty of care. The business judgment rule relates to both of these duties and this post will
focus on the duty of care.

Directors of a corporation have a duty to manage the corporation to the best of their ability, and
this obligation entails that they discharge those duties in good faith and with the care that an
ordinarily prudent person in a like position would exercise under similar circumstances. The
standard should remind you of the standard for negligence. Further, the directors must act in a
manner that they reasonably believe to be in the best interests of the corporation.

The business judgment rule is a protection of sorts for the directors. Provided they act as outlined
above, they will not be liable for corporate decision that in hindsight turn out to be wrong. In
addition, it's the burden of the person challenging the directors' action to prove that the standard
as outlined above has been not satisfied.

Sometimes, whether the standard has been satisfied could be a close call. It's important to note
that in discharging obligations, a director is entitled to rely on information, reports, or statements
if they have been prepared or presented by corporate officers or employees whom the director
reasonably believes to be reliable. A director is also entitled to rely on the same from legal
counsel, accountants, or others as to matters the director reasonably believes to be within the
competence of those professionals. Likewise, a director may rely on a committee of the board of
directors of which the director is not a member if the director reasonably believes the committee
is worthy of trust.

Secured Parties vs. Buyers


There are two problematic things about UCC Article 9 on the bar exam. Most people don't want
it to show up, and it shows up. An area that appears with some consistency deals with priority
between a secured party and a buyer in the ordinary course of business.

First the easier case: a person who buys or leases collateral from a debtor generally has an
interest in the collateral superior to a secured party who has an unperfectedsecurity interest in
the collateral if the buyer or lessee without knowledge of the security interest gives value and
receives delivery of the collateral. In other words, this is yet another situation in which a bona
fide purchaser ("BFP") wins. There are certain types of collateral (for example "accounts")
which cannot be delivered and so in those situations the delivery requirement is not applied.

Even an unperfected secured party may not be out of luck, though. If that party attaches a
purchase-money security interest ("PMSI") in the debtor's collateral before the buyer or lessee
pays value and receives delivery from the debtor, the unperfected interest will have priority over
the buyer or lessee but only if that party perfects the interest by filing within 20 days after the
debtor takes possession of the collateral.
Commonly tested is the priority between a buyer or lessee and a perfected security interest.
Generally, a perfected security interest wins over the rights of a buyer or lessee who receives the
collateral after the secured party has perfected his interest. But there are many exceptions. All of
the following will allow a buyer or lessee to defeat even a perfected secured party.

--The secured party consents to a disposition of the collateral free of the security interest.

--A buyer or lessee in the ordinary course of business takes free of a perfected security interest
unless the buyer knows that the sale or lease is in violation of the security interest. This is true
even if the buyer knows that another party has a perfected security interest in the goods that they
buyer is going to purchase. That alone will not prevent the buyer from having the status of a
bonafide purchaser for purposes of Article 9.

--A consumer purchasing from a consumer has priority over a secured party who has a PMSI in
consumer goods unless the purchaser knows of the security interest or a financing statement has
been filed by the secured party.

--A buyer or lessee not in the ordinary course of business has priority over future advances or
commitments to make future advances made by a secured party after the secured party learns of
the purchase or lease as well as any future advances made more than 45 days after the purchase
or lease.

Posted by Sean Silverman at 9:43 AM 0 comments


Email ThisBlogThis!Share to TwitterShare to FacebookShare to Pinterest
Labels: Secured Transactions
Tuesday, March 19, 2019
Installment Contracts
Probably the best way to think about installment contracts for purposes of the MBE is that they
act an an exception to the perfect tender rule. The perfect tender rule states that if goods or their
delivery fail to conform to the contract in any way, the buyer may reject all, accept all, or reject
some and accept the rest.

If the contract, however, is an installment contract (one that authorizes the goods to be delivered
in separate lots and to be separately accepted), then an installment can be rejected only if the
nonconformity substantially impairs the value of that installment and if the nonconformity
cannot be cured. In other words, the right to reject a nonconformity in an installment contract is
quite limited. In addition, the entire installment contract (as opposed to just an individual
installment within the contract) is breached only if the nonconformity substantially impairs the
value of the entire contract.

The standard for curing a nonconformity in an installment contract also differs from the standard
in a non-installment contract. In a non-installment contract, the seller may cure a nonconformity
but generally only within the time originally provided for performance. The seller must give
reasonable notice of the intent to cure and then make a new tender of conforming goods. Under
limited circumstances the seller can cure even beyond the time for performance if the seller
reasonably believed that the nonconforming goods would have been acceptable to the buyer and
if the seller provides reasonable notification to the buyer as to the intent to cure beyond the time
for performance.
But under an installment contract, it is far simpler. A defective installment within an installment
contract cannot be reject if the defect can be cured.
Posted by Sean Silverman at 10:04 AM 0 comments
Email ThisBlogThis!Share to TwitterShare to FacebookShare to Pinterest
Labels: Contracts/Sales
Tuesday, March 12, 2019
Perpetuating Testimony
One thing about the MBE: If there is an exception that occurs rarely it's likely to be tested.

Generally to conduct discovery (for example, depositions) a suit must already have been filed.
The federal rules recognize only one exception to this general rule. A person may seek a court
order to perpetuate testimony about a matter in federal court even before suit has been filed but
only if a verified petition is filed showing the subject matter of the expected action and the
petitioner's interest. The petition also must state that the petitioner expects to be a party in a
federal court action but cannot presently bring the action or cause it to be brought. Further,
petitioner must include the facts that petitioner wants to establish by the proposed testimony and
the reasons to perpetuate it. Finally, included must be the names or a description of the persons
whom petitioner expects to be adverse witnesses and their addresses (if known) as well as the the
names, addresses, and expected substance of testimony of any deponent.

Because allowing the petitioner to perpetuate testimony before suit has been filed is an exception
to the rule, it's important to understand the limits. It'll be allowable only where a delay in
discovery is likely to result in loss of evidence. An example might be if a party to the lawsuit is
very ill or is planning to leave the country or if there is evidence that a party might later destroy
or conceal evidence.

As always, know the general rule, but also note that the writers of the MBE prefer to test the
exceptions.

Posted by Sean Silverman at 11:18 AM 0 comments


Email ThisBlogThis!Share to TwitterShare to FacebookShare to Pinterest
Labels: Federal Civil Procedure
Wednesday, February 20, 2019
Good Luck!!
All best to those who are finishing up preparation for the February exam! Posting to resume
shortly to assist those preparing for July.
Posted by Sean Silverman at 12:00 PM 0 comments
Email ThisBlogThis!Share to TwitterShare to FacebookShare to Pinterest
Tuesday, February 19, 2019
Mortgage Modifications
Assume a situation in which bank 1 lends money and secures repayment with a mortgage on the
borrower's home. Later the borrower borrows more money, this time from bank 2, and also
secures that loan with a mortgage on the same home.

First, because bank 1was first it time and there is no reason to adjust the priority rules, bank 1
would be senior to bank 2. But what if after bank 2 secures a mortgage, bank 1 modifies its
mortgage? If we treat bank 1's mortgage as of the date of the modification then bank 2 would
have priority over bank 1 since bank 2's mortgage was prior to bank 1's modification. But if we
treat bank 1's mortgage as of the date that it first secured its mortgage then bank 1 would be
senior to bank 2.

It depends. You'll want to focus carefully on the modification. If the modification of the senior
mortgage makes the senior mortgage more burdensome, then the junior mortgage will take
priority over the senior mortgage because the law will treat the senior mortgage as existing as of
the date of the burdensome modification. This would be true if, for example, the interest rate or
the principal amount of the loan is modified by the senior mortgage.

Not all modifications will lead to this result, though. If the modification of the senior mortgage is
not detrimental to the junior mortgage, then priority will remain as is. An example here could be
decreasing the amount of money that the borrower has to pay each month but not increasing the
interest rate or the principal amount of the loan.

And so the result here will differ depending upon the nature of the modification.
Posted by Sean Silverman at 9:54 AM 0 comments
Email ThisBlogThis!Share to TwitterShare to FacebookShare to Pinterest
Labels: Real Property
Thursday, February 14, 2019
Competency of Witnesses
With all the complicated content tested within Evidence questions, the question as to what makes
a witness competent to testify may seem simple, but the issue shows up and it's important to
know how to address it. It might first be noted that witnesses are presumed competent until the
contrary is established. But to determine whether the contrary has been established, the rules for
competency are important.

There are two basic requirements here: witnesses must have personal knowledge of the matter
that is the subject of the testimony, and the witness must declare he/she will testify truthfully.

If the witness requires an interpreter, the interpreter must be qualified and must take an oath to
make a true translation.

A few situations occur often enough that it's helpful to note them. The competency of an infant
will depend only upon the infant's capacity to testify and the trial judge can make this
determination. A person who has been adjudicated insane may testify provided the person
understands the obligation to speak truthfully and has the capacity to testify accurately. A
presiding judge may not testify as a witness nor may jurors testify before the jury in which they
are sitting.

The Dead Man Act never seems to die. These acts generally provide that a party who stands to
gain or lose by a judgment is incompetent to testify to a personal transaction or communication
with a deceased person when such testimony is offered against the representative or successors in
interest of the deceased. There is advice often given that the Dead Man Act is always a wrong
answer on the MBE. This isn't far from the truth, but it should be noted that although there is no
Dead Man Act in the Federal Rules, a state Dead Man Act can still apply where state law, under
the Erie Doctrine, provides the rule of decision (for example, when a case is in federal court
based only on diversity of jurisdiction).
Posted by Sean Silverman at 10:39 AM 0 comments
Email ThisBlogThis!Share to TwitterShare to FacebookShare to Pinterest
Labels: Evidence
Thursday, February 7, 2019
Jury Trials
Lots to know about jury trials in civil cases for Civil Procedure questions on the MBE. Below is
a list of facts, all of which are fair game on the exam:

~ To avoid waiver, a jury trial must be demanded in a writing within 14 days after the filing of
the last pleading directed to the jury-triable issues. Some practice MBE questions have indicated
that the court has discretion to extend the time limit.

~ If legal and equitable claims are joined in one action, the legal claims should be tried first to
the jury and then the equitable claims should be tried to the court.

~ A federal court must permit a jury trial in a diversity suit even though the state court would
deny a jury trial. The jury must have at least 6 jurors and not more than 12.

~ For purposes of jury selection, the venire (the potential jurors) must represent a reasonable
cross-section of the community. The potential jurors will be questioned ("voir dire") to determine
potential bias. Bias may be express in which case the potential juror may be excused for cause,
or implied in which case bias is not expressly admitted but it is unlikely that an average person in
the juror's position would not be biased. A potential juror with an implied bias must be excused
for cause.

~ Objections to giving or failing to give jury instructions prior to the jury deliberating must be
made before the jury retires to consider the verdict.

~ If a juror wants to view property or places involved in the case, the juror may do so but only
upon court order. Jurors must not communicate with non-jurors about the case.

~ Jury verdicts must be unanimous unless the parties agree otherwise. The court may instruct the
jury to decide by a general verdict (the jury finds for the plaintiff or defendant) or by a special
verdict (the jury makes findings of fact and the court applies the law). In addition, a combination
of the two is allowable in which case the jury enters a general verdict but the court guides the
jury with special interrogatories.

~ If a verdict shows on its face that the jury failed to follow the court's instructions, the jury will
either be asked to re-deliberate or a new trial will be ordered. A new trial is appropriate if the
juror gave false testimony or concealed a material fact relating to the juror's qualifications to
serve. A juror may not testify as to any matter occurring during deliberation, except as to outside
influences on the jury or on the question of whether extraneous prejudicial information was
improperly brought to the jury's attention.
Posted by Sean Silverman at 9:29 AM 0 comments
Email ThisBlogThis!Share to TwitterShare to FacebookShare to Pinterest
Labels: Federal Civil Procedure
Friday, February 1, 2019
Third-Party Complaints (Impleader)
In explaining third-party complaints it's probably first best to understand the characters. Imagine
that X sues Y. X would be the plaintiff, and Y the defendant. If Y then wants to bring Z into the
lawsuit, Y would be known as a third-party plaintiff, and Z would be known as a third-party
defendant. And so the question becomes when is it allowable for Y to file a complaint against Z.

There's a specific rule that addresses this. A defending party (Y) may serve a summons and
complaint on a non-party ( Z) who is or may be liable to the defending party (Y) for all or part of
the claim brought by the plaintiff (X) against the defending party (Y). This is known as
impleader, and it shows up with frequency on the MBE.

It's important to note the limitations of the impleader rule. It only applies if a third-party plaintiff
is alleging that the third-party defendant is or may be liable to the third-party plaintiff for all or
part of the claim originally brought against the third-party plaintiff. In other words, the third-
party complaint must assert some sort of derivative liability. The third-party plaintiff cannot
implead a third-party defendant if the reason for doing so is not for the reason stated above.

Put a bit more simply, a third-party claim may be asserted only when the third party defendant's
liability is in some way dependent on the outcome of the main claim made by plaintiff against
defendant or when the third party defendant is secondarily liable and not primarily liable to the
original plaintiff. So, if the third-party plaintiff wants to claim that the third-party defendant
alone is liable to the original plaintiff, that would not be a proper basis for impleading the third-
party defendant. That would be claiming direct liability, and the nature of third-party claims is
derivative.
Posted by Sean Silverman at 10:07 AM 0 comments
Email ThisBlogThis!Share to TwitterShare to FacebookShare to Pinterest
Labels: Federal Civil Procedure
Thursday, January 24, 2019
Shareholders' Appraisal Rights
There are certain fundamental changes that a corporation might approve for which a shareholder
might dissent. When that happens, a shareholder will not necessarily be required to abide by the
change; instead, shareholders are given rights known as appraisal rights which could require the
corporation to purchase the shares of the shareholder. Not all shareholders are provided this
right, however. The following have such a right: shareholders entitled to vote on a plan of merger
(and shareholders of the subsidiary in a short-term merger); shareholders of a corporation whose
shares are being acquired in a share exchange; and shareholders who are entitled to vote on a
disposition of all or substantially all of the corporation's property.

If this issue shows up on a Corporations essay, there is a procedure with which to analyze it. The
first step is for the corporation to note when providing notice of a shareholders' meeting that
shareholders will be entitled to exercise their appraisal rights at the meeting. And before any vote
is taken at the meeting, a shareholder who plans to exercise that right must deliver written notice
of the intent to demand payment for the shareholder's shares should the corporation choose to
move forward with a proposed action at the meeting. The shareholder, in issuing that intent,
cannot vote in favor of the proposed action.

If the corporation moves forward with the proposed action, the corporation must notify within 10
days of approval of the action all shareholders who filed an intent to demand payment. The
corporation must then pay to the shareholder the amount that the corporation estimates to be the
fair value of the shareholder's shares, plus accrued interest.

It's of course possible that the shareholder will not agree with that estimate. If so, the shareholder
then has 30 days in which to send the corporation the shareholder's own estimate of value and
demand payment for that new amount. (Generally, this will amount to the difference between the
corporation's estimate and the shareholder's estimate.)

When the corporation does not agree with the shareholder's estimate, the decision goes to court.
The corporation must file an action in court within 60 days of receiving the shareholder's
estimate with a request that the court decide on a fair valuation of the shareholder's shares. If the
corporation does not satisfy the filing requirement, the shareholder's estimate governs.

Posted by Sean Silverman at 9:38 AM 0 comments


Email ThisBlogThis!Share to TwitterShare to FacebookShare to Pinterest
Labels: Corporations & Limited Liability Companies
Thursday, January 17, 2019
Zoning Regulations
A recent essay on the Uniform Bar Exam dealt heavily with zoning, and the topic has been
showing up with more frequency on the MBE. This isn't too surprising because when the NCBE
updated its Real Property content recently, a heavier focus on zoning was noted. As such, the
following are all important points to note:

When the state enacts a zoning statute, the purpose is to control the use of land for the health,
safety and welfare of its citizens. Importantly, this power is limited by the Due Process Clause,
the Equal Protection Clause (look out for racial discrimination) as well as the Fifth Amendment's
provision preventing taking private property for public use without providing just compensation
to the property owner. It should be noted that if there is a valid restrictive covenant on the use of
land, the fact that a zoning ordinance would allow that use will not generally prevent the
covenant from being enforced.

A cumulative zoning ordinance creates a hierarchy of uses of land. Under such an ordinance,
land that is zoned for a particular use may be used for the stated purpose or for any higher use.
For example, if a single-family home is a higher use than an apartment building, then the land
may be used for a single family home even if it is zoned for apartment buildings. But in such a
case, use of an apartment building would not be allowable if zoned for single-family homes.
Noncumulative zoning, in contrast, is simpler; land may be used only for the purpose for which it
is zoned.

A nonconforming use is a use that exists at the time that a zoning ordinance is passed but also
does not conform to the ordinance. The use itself is allowable, but any changes that take place
after the ordinance is in effect must comply with the ordinance. One way to analyze whether a
change to a nonconforming use is allowable is to ask whether the change is substantial or
insubstantial; if substantial it is likely that the change will not be allowable. It's safe to assume
for the exam that any doubts should be resolved against the change.

A special use permit relates to uses that are allowed in specific zoning districts provided they
meet certain conditions set out in the ordinance. The conditions are generally intended to prevent
adverse impacts on the neighboring areas. For example, special use permits might be granted to
hospitals, funeral homes, etc.

A variance allows for uses that would otherwise be prohibited by a zoning ordinance. Since a
variance violates a zoning ordinance, to obtain a variance, an applicant must ordinarily show
hardship which might be shown by providing that without the variance the applicant will not be
able to make reasonable use of the land.

Posted by Sean Silverman at 12:20 PM 0 comments


Email ThisBlogThis!Share to TwitterShare to FacebookShare to Pinterest
Labels: Real Property
Thursday, January 10, 2019
Dangerous Wild Animals
Animals show up in a variety of contexts in Torts on the MBE. Many of the strict liability
questions that show up in Torts will involve dangerous defects in products, but you'll also be
questioned on strict liability due to harm caused by animals. And the liability due to animals
often turns on whether the animal is deemed to be wild or domestic. On the MBE it's safe to
assume that domestic animals are commonhousehold pets and livestock. Other animals are wild.
The distinction is, admittedly, not always easy.

Wild Animals:

First to note is that an owner is strictly liable for reasonably foreseeable damage caused by
trespass of the owner's wild animals. An owner is likewise liable to both licensees and invitees
for injuries caused by wild animals, even if that wild animals is kept as a pet. The harm that
results from the wild animal must, however, result from the kind of danger that is to be
anticipated from the animal, which can include harm that results from fleeing from the perceived
danger. In other words, the harm must result directly from the dangerous propensities of the wild
animal.

Domestic Animals:

An owner is not strictly liable for injuries caused by domestic animals. An important exception
to note is that an owner can be held strictly liable for injuries caused by domestic animals if the
owner has knowledge of the dangerous propensities of the domestic animal and if those
dangerous propensities are not common to the species. You should always ensure before finding
liability for harm caused by a domestic animal that whatever dangerous propensities that animal
has is not one that is common to the species.

It's important to note that strict liability will generally not be imposed in favor of trespassers who
trespass on the land and are then injured by an animal (wild or domestic). But always consider
liability based on intentional tort, which might be available even to trespassers.
Posted by Sean Silverman at 10:57 AM 0 comments
Email ThisBlogThis!Share to TwitterShare to FacebookShare to Pinterest
Labels: Torts
Thursday, January 3, 2019
Examining Witnesses
The examination of witnesses will show up frequently in Evidence questions on the MBE. It's
difficult to narrow down this topic in a post but here I'll focus on a few of the issues that could
show up on the exam:

Leading Questions: Leading questions (those questions that suggest the desired answer) are
generally improper on direct examination. Such questions, however, are permitted on cross
examination. They are also permitted to elicit preliminary or introductory matters and when
necessary to assist a witness who might need help due to loss of memory, immaturity, or
physical/mental weakness. An attorney is also permitted to ask leading questions of a hostile
witness.

Improper Questions & Answers: Questions are improper if they are misleading, compound
(requiring a single answer to more than one question), argumentative, conclusionary, cumulative,
or unduly harassing. Also improper are questions that call for a narrative answer or call for
speculation. Questions must also not assume facts that are not in evidence. As to answers, they
must not lack foundation (for example, if the witness has insufficient knowledge but answers
regardless), and must not be unresponsive to the question asked. All of the above will allow for
sustained objections.

Cross Examination: There's an art to cross examination, but on the MBE, we need only know
the basics. Cross examination of adverse witnesses is a matter of right but is limited to the scope
of direct examination (including all reasonable inferences that may be drawn from the direct
examination) as well as testing the credibility of witnesses (through the use of impeachment).
The cross examiner is generally bound by the answers of the witness to questions concerning
collateral matters. In other words, if a question concerning a collateral matter is asked, the cross
examiner generally must accept the answer and may not refute the answer by other extrinsic
evidence. An important exception applies to impeachment: certain recognized avenues for
impeaching a witness such as bias, convictions of crimes, etc., may be developed through
extrinsic evidence beyond cross examination.
Posted by Sean Silverman at 11:58 AM 0 comments
Email ThisBlogThis!Share to TwitterShare to FacebookShare to Pinterest
Labels: Evidence
Monday, December 31, 2018
Available Tutoring Options (February 2020)
I work one-on-one with students through phone and video (Skype) tutoring for both the MBE
and the essays on the uniform bar exam ("UBE") and the Florida bar exam. Here is a bit about
how each component of the tutoring works:

The MBE:

The MBE tutoring consists of assigning an MBE subject to review throughout the week. The
lesson provides you the opportunity to thoroughly review with me any of the substantive law in
that subject. We'll also spend the lesson reviewing practice questions that I've assigned prior to
our meeting.

To perform well on the MBE requires a deep understanding both of the law and of the test. A
misconception is that one can score very well merely by memorizing the law. Rather, in addition
to that foundation, you also must develop skills both for issue spotting and for eliminating wrong
answers. As such, the lesson is geared towards two components: building a solid knowledge
base, and learning the skills required to perform well.

Specifically, there are two purposes to the MBE tutoring. The first is to fully understand the legal
rule tested in any question you answered incorrectly so that you can later apply that rule to a new
fact pattern. And the second is to examine the questions I've assigned to determine where you
could have improved in your legal analysis and to develop strategies for more effectively
answering questions correctly. We work extensively on eliminating the distractors so that you’ll
have a better understanding of the patterns used by the test makers to create incorrect answers.

The Essays:

The intent of the essay tutoring is to learn how to apply the law to the types of essay questions
likely to show up on the exam.

I assign essays and we review your responses to these essays in detail throughout the lesson. We
work together on maximizing the number of points you can score on any given essay through
rigorous and effective legal analysis and I teach my theory on how best to IRAC in order to score
those points.

So much of effective essay writing on a bar exam revolves around the difficult skill of analyzing
legal issues and so our goal throughout is to fine tune those skills before you sit for the exam.
After the lesson, I send to you a model answer written by a candidate who had written the essay
while taking the bar exam as well as my own answer to the question which incorporates my
blueprint for essay writing that I'll be sending to you as well.

Final Thoughts:

I've taught the bar exam for a number of years now. It would be difficult for me to summarize in
such a short post exactly what is required to excel on the exam. But I do strongly believe that at
its core there are two components to doing well.

Imagine you set out to learn chess. Before practicing, you'd want to know the rules of the game
because without knowing those rules your progress would be hindered regardless of how much
you’ve practiced. But you wouldn't want to play in a chess tournament if you had only read the
rulebook. Because to learn the game would require practicing and applying all those rules you
had learned. Memorizing rules is not the same as developing skills.

In short, that's the bar exam. You can not score well if you don't know the law. But to excel on
the exam requires far more than legal knowledge. You must learn the skill of applying the law to
a given set of facts both on multiple choice questions and on essays.

You're welcome to reach out to me @ silvermanbarprep@gmail.com to learn more about the


tutoring!

Posted by Sean Silverman at 9:43 AM 0 comments


Email ThisBlogThis!Share to TwitterShare to FacebookShare to Pinterest
Labels: Tutoring
Friday, December 28, 2018
Imperfect Self Defense
This post will mainly be about a concept called imperfect self-defense, but to understand it
requires an understanding of traditional self-defense. And to understand self-defense requires an
understanding of the differences when one is confronted with deadly force and nondeadly force.
A person not at fault may use such force as the person reasonably believes is necessary to protect
himself from the imminent use of unlawful force against that person. If the person is not
threatened with deadly force, then deadly force cannot be used as a means of protection. There is
no duty to attempt to retreat prior to using nondeadly force for self defense.

In contrast, a person can use deadly force if the person without fault is confronted with unlawful
force and reasonably believes that there is an imminent threat of death or great bodily harm.
Generally, there is no duty to retreat before using deadly force to protect yourself. A minority
view does require an attempt to retreat before using deadly force but not if the attack occurs in
the home or if the attack occurs while making an arrest or during the commission of a robbery.

To be clear, traditional self defense using deadly force requires a reasonable belief that death or
great bodily harm is imminent. And so what should result if you have an honest belief that such a
threat exists, but objectively that belief is not reasonable?

Imperfect self defense is recognized in some states. Under this doctrine, a defense can exist even
if the belief as to the threat is not deemed objectively reasonable or if the person claiming the
defense was the original aggressor. Specifically, imperfect self defense is allowable if the
defendant was at fault in starting the altercation or if the defendant unreasonably but honestly
believed in the necessity of responding with deadly force.

The practical differences between self defense and imperfect self defense are important as well.
If one proves self defense, then that will be a complete defense to the crime charged. In other
words, the person will be deemed not guilty. But imperfect self-defense doesn't reach so far.
Imperfect self defense could lead to a conviction of voluntary manslaughter rather than murder,
but it will not completely exonerate someone.
Posted by Sean Silverman at 9:19 AM 0 comments
Email ThisBlogThis!Share to TwitterShare to FacebookShare to Pinterest
Labels: Criminal Law
Thursday, December 20, 2018
The Statute of Frauds Under the UCC
It seems to me that generally those preparing for the bar exam have a pretty good feel for the
Statute of Frauds. In most instances, oral contracts are valid, but there are some types of
contracts that require a writing to be enforceable. One type of contract that requires a writing is a
contract for the sale of goods for a price of $500 or more.

A contract for the sale of goods for a price of $500 or more generally requires a writing to
evidence that the agreement actually took place. The writing will be sufficient even if it omits
terms or incorrectly states terms, but if quantity is incorrectly stated then the contract will only
be enforceable as to the quantity stated. As a general point, it should be noted that quantity is an
essential term under the UCC.

But it's not the rule itself that trips people up on these questions. There are times when even
though a contract for the sale of goods is for a price of $500 or more, a writing is not required.
The MBE is often a test of exceptions, and so knowing these exceptions may be more important
than knowing the rule.

The first exception is for specially manufactured goods. If goods are to be specially
manufactured for the buyer and are not suitable for sale to others by the seller in the ordinary
course of business, the contract is enforceable even without a writing but only if the seller has
made a substantial beginning in the manufacturer of the goods or commits for their purchase
prior to the buyer repudiating.

Next, a party might claim that the contract should not be enforced because there is no writing but
that party has already admitted in the pleading, testimony, or otherwise, that the contract for sale
was made. Here, the contract will be enforceable without a writing but only for an amount that
was previously admitted.

It's also possible that a buyer might accept and pay for goods, but then later claim that the
contract for those goods is not enforceable because it wasn't in writing. The contract will be
enforceable, but not beyond the quantity of goods accepted and paid for.

Lastly, and an exception that shows up often on the MBE, is the merchant's confirmatory memo
rule. In contracts between merchants, if one party within a reasonable time after an oral
agreement sends to the other party a written confirmation of the oral agreement and if the written
confirmation satisfies the Statute of Frauds, then the contract will be enforceable even though the
original agreement between the parties was not in writing. The party receiving the memo will
now be bound by the contract if that party has reason to know of the confirmation's contents, and
does not object to it within 10 days of receipt.
Posted by Sean Silverman at 9:10 AM 0 comments
Email ThisBlogThis!Share to TwitterShare to FacebookShare to Pinterest
Labels: Contracts/Sales
Thursday, December 13, 2018
The Supremacy Clause and Preemption
I've found when working with students that many understand the purpose of the Supremacy
Clause of the Constitution. But the MBE might test some very fine details that are worth
remembering.

First to note is the idea of express preemption. A federal law may expressly provide that the
states may not adopt laws concerning the subject matter of federal legislation. More difficult
(and subtle) is implied preemption. If a state law conflicts with a federal law, the state law will
be held to be impliedly preempted. Similarly, if a state law prevents the achievement of a federal
objective, it will be held to be impliedly preempted. And this is so even if the state law was
enacted for a valid purpose other than to prevent the achievement of the federal law.

You'll also want to keep in mind the situations in which a federal statute occupies the field. In
those instances, the statute will preclude any state regulation even if the state regulation does not
conflict with a federal law. Determining whether the federal regulation occupies the field will be
the objective of a court which will use a variety of factors in determining congressional intent.

In all of the above (and especially when determining whether the federal court has occupied the
field) the court will start with the presumption that state powers are not to be superseded unless
that was the clear intent of Congress.

Posted by Sean Silverman at 9:25 AM 0 comments


Email ThisBlogThis!Share to TwitterShare to FacebookShare to Pinterest
Labels: Constitutional Law
Wednesday, December 5, 2018
Amendments to Pleadings
In the subject of Civil Procedure, we can't get around the fine details. And there are many that
you'll need to know when answering questions about amending a pleading. Such questions are
implicated when, for example, a complaint is served and then the party that filed the complaint
decides an amendment is needed.

The rule here is that a pleading may be amended once within 21 days of its service. If the
pleading is one to which a responsive pleading (for example an answer to a complaint) is
required, then the original pleading may be amended within 21 days after service of the
responsive pleading or pre-answer motion. After this one amendment, however, leave to amend
is by consent or by leave of court which must be freely given.

And then assume a pleading is amended. The next question is whether the amended pleading will
relate back to the time period of the original pleading. For purposes of statutes of limitation, an
amendment to a pleading that arises from the same conduct, transaction, or occurrence that was
set forth in the original pleading generally is deemed filed on the date that the original pleading
was filed. This is important since it's possible that the date of the amended pleading might occur
after the expiration of the statute of limitations on the claim that is the subject of the complaint.
But by relating back to a time period prior to the expiration of the statute of limitations, that is no
longer an issue.

Note also that an amendment naming a new adverse party will relate back if, within the time for
service of process, the new party had sufficient notice of the action to avoid prejudice and knew
or should have known that but for a mistake concerning the new party's identity, the new party
would have been named in the original pleading.
Posted by Sean Silverman at 8:57 AM 0 comments
Email ThisBlogThis!Share to TwitterShare to FacebookShare to Pinterest
Labels: Federal Civil Procedure
Thursday, November 15, 2018
Formation of a Partnership
There's a lot that can be tested in Partnership law on the UBE. As an initial matter, though, you'll
want to know how a partnership is formed. And to understand that, you'll need to know both
what is required and also what is not required.

A partnership is formed when when two or more people associate to carry on as co-owners of a
business. Interestingly, no formal agreement is required to form a partnership. In other words,
the intent to associate may be implied from conduct. Worth noting though that the Statute of
Frauds will generally require a writing if the partners wish to agree to remain partners for more
than a year.

Because no formal agreement is generally required, it may be difficult to determine whether a


partnership actually exists, and the courts will generally look to the intent of the parties which
can be established by observing whether profits were shared. Sharing of profits raises a
presumption of partnership unless the share was received as payment for a debt, services
rendered, rent payments, etc. Courts will also look to other factors such as whether property is
held in joint tenancy, and whether the parties have designated themselves as partners, but none of
these factors will be as dispositive as a determination that profits were shared.
As to the required capacity, anyone who is capable of entering into a binding contract may enter
into a partnership. If someone lacks such capacity, that person will be liable only to the extent of
his capital contributions. Provided there are at least two others wishing to form a partnership, the
partnership may still exist without that person unless steps are taken to dissolve it.

A partnership formed to achieve an illegal purpose is void, and unless otherwise agreed upon, no
one can become a partner without the express or implied consent of all other partners.

An Explanation of the Erie Doctrine


The Erie Doctrine is definitely among the topics tested on the MBE that students believe to be
the most difficult. It's tough but if analyzed systematically you'll be able to answer these difficult
questions correctly.

The Erie Doctrine aims to clarify whether a federal court in a case that is in federal court based
on diversity of citizenship will apply its own federal law or instead apply state law to an issue in
the case. A federal court in a diversity case will apply its own procedural law, but it must apply
the substantive law of the state in which the court sits.

Step one in the analysis is to ask whether there is a federal law on point (for example, a Federal
Rule of Civil Procedure). If there is, and provided it is valid, the federal rule should be applied.
Oftentimes, the federal court will have to decide whether to apply state or federal law to a given
issue, and that's where the Erie Doctrine provides some guidance.

If the issue is procedural, a judge can decide to apply federal law when determining that issue. If
the issue is substantive, state substantive law rather than federal law will be used to decide the
issue. It's not easy to determine whether the issue is procedural or substantive but the test to
make that determination is called the outcome determinative test. An issue is substantive rather
than procedural if the issue substantially effects the outcome of the case. The court will also
balance the interests so that if the state has a great interest in having its own law applied, the
issue might be deemed substantive and state law will apply. Finally, there is the goal of deterring
forum shopping: if failing to apply state law would increase litigation in the federal courts, and if
the increase would be due to the fact that the federal law is likely to provide a favorable
judgment, the issue might be deemed substantive so that state law applies.

For purposes of the MBE, there are some issues that are clearly substantive. These include
statutes of limitations, rules for tolling statutes of limitations, choice of law rules, and elements
of a claim or defense. On these issues apply state law. It's possible that a state statute or rule may
contain both substantive and procedural elements; in such cases, the federal courts will apply the
state law to the substantive elements, and the federal law to the procedural elements.

If it's determined that a federal court will apply state law, it will apply the substantive law that
would be applied by the highest court in the state.
Posted by Sean Silverman at 8:27 AM 0 comments
Email ThisBlogThis!Share to TwitterShare to FacebookShare to Pinterest
Labels: Federal Civil Procedure
Thursday, October 4, 2018
Purchase Money Resulting Trusts
There are a few different types of resulting trusts that could show up in a trusts essay on the
essay section of the UBE. The one I'd most expect to see is a purchase money resulting trust.

You should consider a purchase money resulting trust whenever a person (call him "x") furnishes
consideration for the acquisition of real or personal property, but with x's consent title is taken in
the name of another person (call him "y"). No trust has actually been created in the traditional
sense, but the idea here is to presume that if the facts are as stated above, then x is the beneficiary
of a trust and that y is a trustee of a trust and is merely holding the property for the benefit of x.
For a purchase money resulting trust to be presumed, the consideration paid by x for the
purchase of the property must be supplied at or before the time that y takes title to the property.

The burden will be on x to prove by clear and convincing evidence that he supplied the
consideration. Once x has met this burden, a resulting trust will be presumed. Y can then rebut
by showing that no trust was intended. Y might, for example. present evidence that the payment
from x was a gift or a loan to y to satisfy a debt that x owed to y.

An exception to keep in mind is where there is a close personal relationship between x and y. In
that case it will be presumed that there was a gift from x to y and no trust will result. As with the
above, this presumption is also rebuttable, this time by x. "Close personal relationship" can be be
tough to define, but the notes indicate that a parent, grandparent or spouse would satisfy the
definition, whereas an uncle, aunt, brother, sister, child, or grandchild would not.

MBE Fast Fact: Statutory vs. Equitable Right of Redemption


This is an easy enough distinction but it's also a question that can easily trip you up if you don't
know the distinction. First a bit about foreclosure:

When a mortgagor (the borrower) fails to pay back the loan borrowed from the mortgagee (the
lender), the mortgagee can foreclose on the mortgaged real estate. Almost all states require
foreclosure by sale whereby the property is sold to satisfy the debt.

Redemption provides a 2nd chance for a borrower who has defaulted, and there are two types of
redemption to keep in mind:

Redemption in Equity:

Redemption in equity allows the mortgagor at any time prior to the foreclosure sale to redeem
the property by paying the amount due. If the note or mortgage contains an acceleration clause,
then in order to redeem, the mortgagor will have to pay the full amount of the note or mortgage.
Importantly, the right of redemption can not be waived in the mortgage itself.

Statutory Redemption:

A less commonly offered right of redemption is statutory redemption. Here, the mortgagor can
redeem the property in a similar manner as stated above for a fixed period of time even after the
foreclosure sale has occurred.
The Attractive Nuisance Doctrine
The attractive nuisance doctrine is sure to show up in your torts class, and also pretty certain to
show up on the bar exam. Most understand that it deals with trespassing children, but there are
some particulars that if you don't know well enough can trip you up on a question.

The policy behind the doctrine is important: it allows for liability in situations where liability
would not otherwise be allowable. Generally, it's difficult for a trespasser to sue for injuries
caused by the landowner or possessor of property. But if the trespasser is a child, it might not be
so difficult.

The attractive nuisance doctrine requires the following:

A possessor of land has a dangerous artificial condition on the land.

and

The place where the condition exists is one in which the possessor knows or has reason to know
that children are likely to trespass.

and

The possessor knows or has reason to know that the hazardous condition on the land involves a
risk of death or serious bodily harm to any child who might trespass.

and

The children because of their age do not discover the dangerous condition or if they do discover
it they do not appreciate the risk.

and

The benefit that the possessor receives from maintaining the condition as well as the burden that
would be required to eliminate it are slight compared to the risk to children.

and

The possessor fails to exercise reasonable reasonable care to eliminate the danger or otherwise to
protect the children.

So, lots of elements. And note that this rule requires that all elements are satisfied for the
doctrine to apply. For example, if it is determined that the child was able to appreciate the risk of
harm then even if all other elements are satisfied, the doctrine will not apply.

Grounds for Divorce


If a Family Law essay shows up on the UBE, there is a strong chance that divorce might be
among the issues tested. Since states can vary as to the grounds required to obtain a divorce, it's
important to know how to approach that issue on the UBE.
Most state statutes provide for divorce without any regard to fault. Since fault is removed from
the equation, it's only required that there is a showing that the marriage has been irretrievably
broken and/or the parties to the marriage have been living apart for a specified period of time. If
one spouse wants to defend against the divorce then that spouse will need to deny existence of at
least one of these grounds.

But some states do require a showing of fault to obtain a divorce. The usual grounds are
adultery, wrongful desertion for a specified time, extreme physical or mental cruelty, voluntary
drug addiction or habitual drunkenness, and a spouse's mental illness.

If analyzing a divorce requiring fault, the final step would be to determine whether any defenses
exist. The following are defenses to a divorce requiring a degree of fault; in other words, these
defenses would be raised after one spouse has proven that the fault required for the divorce has
been satisfied.

Collusion: This is an agreement between the spouses to simulate grounds for divorce or to
forego raising a valid defense. If collusion is proven, this might prevent the granting of a divorce
requiring fault.

Connivance: If one spouse willingly consents to the other spouse's misconduct, the consenting
spouse may be prevented from using that misconduct as a basis for proving that it amounts to the
fault required for a divorce. Should be noted that this is a very limited defense that has been
abolished in most states.

Condonation: This is the forgiveness of marital offenses with full knowledge of their
commission. An indication of forgiveness would be resuming the marital relations after gaining
the knowledge of the offenses.

Recrimination: This defense arises when a spouse seeking a divorce is also guilty of misconduct
that the other spouse could use to obtain a divorce. This defense is no longer recognized.

For the most part, on the UBE it would be enough to analyze no-fault divorce. But knowing a bit
about how various states approach the question doesn't hurt when providing some background
for the analysis.

Apparent Agency
A concept that appears to be showing up on the MBE in the subject of torts originates in the law
of agency. In agency law we learn that even if there is not actual authority granted from the
principal to the agent, there are circumstances in which an agent can contractually bind a
principal. Generally, there are requirements for apparent agency (or sometimes called apparent
authority):

(1): A direct or implied representation of agency by the alleged principal.


(2): Reliance on the representation by the party alleging agency.
(3): Detrimental reliance on the representation by the party alleging agency.

And here's how it's showing up on the MBE. Situations may arise in which a person or company
is performing a job and using a product manufactured by another to perform the job. The person
performing the work, however, is not an employee of the manufacturer. Although the worker is
not an employee of the manufacturer it may still be possible to hold the manufacturer liable if the
worker is negligent, assuming that the manufacturer acted in a way that would have led a
reasonable person (the person injured) to believe that the worker was an employee of the
manufacturer. The basis for holding the manufacturer liable even though the worker was not the
employee of the manufacturer would be on a basis of apparent agency.

Holdover Tenants
A common issue tested within landlord/tenant law on the MBE will present a situation in which
the tenant continues to remain in possession of property after the rightful termination of the
tenant's right to possess. It's then a matter of determining the rights of the landlord.

The landlord has two choices, the first of which is to treat the tenant as a holdover tenant and
evict the tenant under an unlawful detainer statute. But often in the questions, the landlord will
choose not to do so and instead will attempt to bind the holdover tenant to a new tenancy. This
is allowable as well.

The new tenancy will be a periodic tenancy and generally the terms and conditions of the expired
tenancy may govern the new tenancy. But the determination of the terms of the new tenancy will
depend on the nature of the property; if commercial property, then the tenant may be held to a
new year-to-year tenancy provided the original tenancy was for more than a year. If the property
is residential, the hold-over tenant will generally be held to a new month-to-month tenancy
regardless of the terms of that tenant's expired tenancy. It's allowable for the landlord to raise the
rent on both the residential and commercial tenant for the new tenancy but only if the landlord
notifies the tenant prior to the expiration of the original tenancy that if the tenant chooses to
remain, the new tenancy will be at an increased rate.

The landlord may choose at any time to terminate the new periodic tenancy and to do so will
require that the landlord provide notice at least equal to the length of the time period of the
tenancy. For example, if the periodic tenancy is month-to-month then a month notice is required.
One exception to note, though, is that a year-to-year tenancy does not require that the landlord
provide notice a year before termination; six months will suffice.
Posted by Sean Silverman at 11:28 AM 0 comments
Email ThisBlogThis!Share to TwitterShare to FacebookShare to Pinterest
Labels: Real Property
Thursday, June 21, 2018
Disclosure of Expert Testimony
On the topic of discovery within the subject of Civil Procedure, there are 3 types of disclosure
that you should know very well for the MBE. Each can get a bit involved since there are general
rules and exceptions to the general rules for each of them. The 3 types are initial disclosure,
disclosure of expert testimony, and pretrial disclosure. This post will focus on an often-tested
type of required disclosure: disclosure of expert testimony.

Disclosure of expert testimony is required. Specifically, a party must disclose to other parties the
identities of expert witnesses expected to be used at trial. As part of this disclosure, a report is
generally required and the the report must be prepared by and signed by each expert witness
stating his/her qualifications as well as the opinions that the expert intends to express and the
basis for those opinions.

This disclosure must be made either at the time directed by the court or if no time is directed by
the court (and if there is no stipulations among the parties) at least 90 days prior to trial. There
is, however, one exception to note regarding the time requirement: if the evidence to be
presented by the expert is intended solely to rebut another party's disclosure of expert testimony,
then it must be made within 30 days after disclosure of the evidence being rebutted.

Citizenship for Diversity Jurisdiction


Diversity jurisdiction requires that no plaintiff is a citizen of the same state as any defendant.
And so in making this determination it's first required to determine the citizenship of various
parties to the lawsuit. Things can get a bit complicated when the parties are not individuals, but
remembering the following should make the determination of diversity far easier:

Individuals: Determining the citizenship of an individual depends upon the permanent home to
which that individual intends to return. The citizenship of a child is that of the child's parents.

Corporations: A corporation is deemed a citizen of every U.S. state and foreign country in
which it is incorporated and the one U.S. state or foreign country in which it has its principal
place of business. The principal place of business is the U.S. state or foreign country from which
the high-level officers direct or control the activities (often the headquarters).

Unincorporated Associations and Limited Liability Companies: These businesses are


considered to be a citizen of each state in which any member is a citizen.

Legal Representatives: The legal representative of the estate of a decedent, an infant, or an


incompetent, has the same citizenship as the decedent, infant, or incompetent.

Class Actions: Citizenship in class actions lawsuits is determined on the basis of the citizenship
of the named members of the class.
Posted by Sean Silverman at 10:48 AM 0 comments
Email ThisBlogThis!Share to TwitterShare to FacebookShare to Pinterest
Labels: Federal Civil Procedure
Thursday, May 3, 2018
Character Evidence
Along with impeaching witnesses and hearsay evidence, character evidence ranks up there with
the most troubling areas of Evidence for those preparing for the bar exam. It is simpler to
understand character evidence if you distinguish the type of character evidence admissible in
civil cases from the type of character evidence admissible in criminal cases. But first there are
some commonalities:

There are three methods that a person might use to offer evidence of another's character but not
all three are always admissible. The three types are specific acts, opinion testimony, and
reputation testimony.

Civil Cases:
The general rule here is that no form of character evidence is generally admissible in civil cases
regardless of whether the offering is of a specific act, opinion, or reputation testimony. For
example if x sues y for reckless driving, it is generally not admissible for x to offer testimony
that y has driven recklessly in the past if the purpose of offering such testimony is to prove that y
has the character of a reckless driver. Similarly, y could not offer testimony of previous careful
driving to prove that y has the character of a careful driver. An important exception here applies
to civil cases in which character is directly at issue. For example, in a defamation case the
character of plaintiff may be at issue in determining whether the statement made by defendant
about plaintiff was actually false (since falsity is required for a claim of defamation). In these
circumstances, character evidence is admissible.

Criminal Cases:

The defendant can offer (only by opinion or reputation testimony) evidence of the good character
of defendant. And once the defendant has chosen to do so, the prosecution may then cross
examine that defense witness and may even elicit from that witness whether the witness knows
about specific acts that would contradict the testimony previously offered. The prosecution,
however, must limit the rebuttal to cross examination of the defendant's character witness.
Extrinsic evidence to rebut the defendant's character witness is not admissible. The prosecution
may also call up its own witness to testify to the defendant's bad reputation or to give an opinion
about defendant's bad character once the defendant has called up its own character witness. But if
the prosecution chooses this route, asking the witness about specific acts in not allowable.

Victim in a Criminal Case:

It should also be noted that except in rape cases a defendant may introduce reputation or opinion
testimony of a bad character trait of the alleged victim if relevant to show the defendant's
innocence. And once this evidence has been admitted, the prosecution can counter with
reputation or opinion evidence of the victim's good character for the same trait or the defendant's
bad character for the same trait.
Posted by Sean Silverman at 11:08 AM 0 comments
Email ThisBlogThis!Share to TwitterShare to FacebookShare to Pinterest
Labels: Evidence
Thursday, April 26, 2018
Homicide
Homicide is a very important topic to know well within the larger subject of Criminal Law on
the MBE. It's not too complicated but the questions tend to be very tricky. A good start to
breaking this topic down is to understand that there are three ways to commit homicide on the
MBE: murder, voluntary manslaughter, and involuntary manslaughter.

Murder: Murder is the unlawful killing of a human being with malice aforethought. And there
are four ways to satisfy the element of malice aforethought:

--Intent to kill
--intent to inflict serious bodily injury
--reckless indifference to an unjustifiable risk to human life
--intent to commit a felony.
Intent to kill is the most straightforward. But even if x does not intend to kill y, x can be charged
with murder if x intended to inflict serious injury upon y and y ended up dying. And not even
that is necessary; x can instead just act recklessly and if y dies as a result of x's recklessness that
too might be murder. Or x might be committing a felony (bank robbery, for example) and in the
process might negligently kill y. That would be murder as well even though negligence doesn't
rise to the level of recklessness, but only because the negligence occurred during the commission
of a felony.

Voluntary Manslaughter: Voluntary manslaughter is a killing that would be murder but for the
existence of adequate provocation. Provocation is adequate if it would cause a reasonable person
to lose control, if the person does in fact lose control, and if there is not sufficient time between
provocation and the killing for the person to cool off.

Notice that voluntary manslaughter still requires that you analyze murder. In other words,
voluntary manlaughter = murder + adequate provocation.

Involuntary Manslaughter:

A killing is involuntary manslaughter if it was committed with criminal negligence. Some states
also have a rule similar to the felony-murder rule called the misdemeanor-manslaughter rule
which will allow for a charge of involuntary manslaughter if the killing occurs during the
commission of a misdemeanor.

Distinguishing between involuntary manslaughter and murder can be tricky. Murder allows for
recklessness while involuntary manslaughter requires criminal negligence. This is a very fine
line, though recklessness requires a higher degree of fault than does negligence. With
recklessness a person acts with the knowledge that what they are doing is wrong, whereas with
negligence that knowledge need not be present provided that a reasonable person would have
known not to act.

The Final Judgment Rule and Interlocutory Appeals


You've got to be ready for some questions about appeals on the MBE. It shows up in a few
different contexts with one context focusing on whether interlocutory orders are reviewable
immediately or instead whether only final orders are reviewable. The general rule to note
(sometimes referred to as the final-judgement rule) is that only final orders are reviewable, but
the inquiry doesn't end there.

Certain interlocutory orders are reviewable as of right; they include injunctions, appointments of
receivers, and certain admiralty, patent infringement, and property possession cases. In addition,
the Interlocutory Appeals Act allows for discretionary review when the trial judge certifies that
the interlocutory order involves a controlling question of law. Review of interlocutory orders is
more likely if substantial ground for difference of opinion exists, and immediate appeal from the
order may materially advance the ultimate termination of the litigation. It'll also be necessary that
the court of appeals agrees to allow the appeal.

Another basis for immediate appeal prior to a final order is the Collateral Order Doctrine. For
this rule to apply, the claim or issue that is the subject of the appeal must be separable from and
collateral to the main suit and too important of an issue to require deferring appellate review until
a final judgment on the main suit has been rendered.
A few additional points to note: an order granting or denying the certification of a class action
may be reviewed at the court's discretion within 14 days after entry of the order. In addition, in
rare circumstances, the final-judgement rule may be avoided through a variety of appellate writs
such as mandamus (compelling a judge to act) and prohibition (prohibiting a judge from acting).

The Powers and Limitations of the President


The President of the United States as part of the executive branch of government is granted
certain powers by the Constitution, but those powers are far from absolute. For purposes of the
MBE it's important to understand both the powers of the President as well as the limits placed
upon whoever holds the position.

The following are the key points to keep in mind:

~ The President has the power to appoint all ambassadors, other public ministers and consuls,
justices of the Supreme Court, and all other officers of the United States. Congress, however,
vests the appointment of "inferior officers." It should be noted that Congress may not appoint
members of a body with administrative or enforcement powers (often tested).

~ The President can remove high level executive officers (for example, members of the cabinet),
and may do so at will without any interference from Congress. Congress, however, may provide
limitations on the power of the President to remove all other executive appointees. If Congress
itself wants to remove executive officers it may do so but only through the impeachment process.

~ The President may grant pardons for all federal offenses but not for impeachment or for civil
contempt. Congress has no power to limit the pardon power of the President.

~ The President can veto an act of Congress, but that act may still become law if the veto is
overridden by 2/3 vote of each house. Importantly, the veto power only allows the President to
approve or reject a bill in total; there is no right granted to cancel part of a bill and approve other
parts (a line-item veto). The President has 10 days to exercise the veto power. If not exercised in
that time span the bill is automatically vetoed if Congress is not in session. If Congress is in
session, the bill becomes law.

~ The President may act militarily in hostilities against the United States without a formal
declaration of war by Congress. The President, however, has no power to declare war.

~The President has the power to enter into treaties with foreign nations, but such treaties require
the consent of 2/3 of the Senate. If there is a conflict between a treaty and a valid congressional
act, the one that occurred most recently prevails. A treaty, however, never prevails over the
Constitution.

~The President has the power to enter into executive agreements with foreign nations, and these
agreements do not require consent from the Senate. If a state law conflicts with an executive
agreement, the executive agreement prevails, but if a federal law conflicts with an executive
agreement, the federal law prevails.

~ The President is given a privilege to keep certain communications secret. But in criminal
proceedings, communications involving the President will be available where a need for such
information is demonstrated.

~ The President has immunity from civil damages based on any action he took within the official
responsibilities of the office but there is no immunity for acts that occurred prior to taking office.

~ The President is subject to impeachment. Grounds for impeachment include treason, bribery,
and high crimes and misdemeanors. A majority vote in the House is needed to invoke charges
against the President, and a 2/3 vote in the Senate is necessary to convict and remove from
office.
Posted by Sean Silverman at 8:34 AM 0 comments
Email ThisBlogThis!Share to TwitterShare to FacebookShare to Pinterest
Labels: Constitutional Law
Thursday, March 15, 2018
Defective Products
There are a variety of theories upon which one might sue for products liability. Among these
theories are intent, negligence, strict liability, and breach of warranty. One commonality among
every theory is that there must be proof that the product sold was defective and that it was
defective when it left the defendant's control. And because this element applies to all theories of
products liability, it's important to understand well the rules regarding defective products.

There are three types of defects: manufacturing defects, design defects, and inadequate warnings.

Manufacturing Defects: These defects occur when a product emerges from manufacturing
different from and more dangerous than the products that were made properly. Importantly, you
should note that with manufacturing defects, it's not required that all products within a line of
products suffer from the defect; instead, only certain products within the line of products suffer
from the defect, and those products are more dangerous that the products that are not defective.

Design Defects: These defects occur when all products within a line of products have dangerous
propensities that render them all defective based on a design defect in the product itself.

Inadequate Warnings: Along with design defects and manufacturing defects, a product might
be defective as a result of the manufacturer's failure to provide adequate warnings as to the risk
involved in using the product. Be careful with this one; if the risk is apparent to reasonable users
then the manufacturer is not likely to face liability for failing to warn about such apparent
dangers.

Along with understanding the various defects, you should also understand how each defect is
proven. For manufacturing defects, defendant will be liable if plaintiff can show that the product
failed to perform as safely as an ordinary consumer would expect. For a design defect, plaintiff
must show that defendant could have made the product safer and that doing so would not have
been economically or otherwise infeasible. And for inadequate warnings it'll need to be shown
that there was a lack of warning as to a non-apparent danger; for example, manufacturers are not
likely to be held liable for not warning buyers about the dangers of a knife since the dangers are
apparent and for the knives to perform their function requires a danger inherent to the product.

Future-Advance Mortgages (Optional vs Obligatory)


There is no doubt that mortgages is a subject that you'll want to understand very well when
preparing for Real Property on the MBE. If you take a look at the subject-matter outline
prepared by the National Conference of Bar Examiners ("NCBE"), under the topic on mortgages
is listed a sub-topic of "future-advance mortgages." A few important points to keep in mind to
be prepared to answer these questions correctly:

Watch for a situation in which a lender advances funds to a borrower for a fixed amount. The
lender secures a mortgage on property not only for the amount of money that it has advanced, but
also for the amount (or amounts) it agrees to advance in the future.

It's possible that the future advances might be obligatory in the sense that the lender is
contractually obligated to advance the future funds. But it's also possible that the future advances
will be optional in which case the lender will retain the option to later decline to make any
advances. This distinction is the key to the analysis.

If payments under a future-advance mortgage are deemed obligatory then a junior lender's lien is
junior both to amounts advanced to the debtor before the junior lien was recorded and to amounts
advanced after the junior lien is recored.

So, for example, if lender 1 lends money to debtor and secures a future-advance mortgage which
is recorded in which the lender is contractually obligated to advance more funds in the future,
then lender 1 need not worry about lender 2 (a 2nd mortgagee) lending money to buyer after
lender 1's first advancement of funds but prior to the future advancement of later funds. Lender 1
will have priority over not only the first advancement but also over future advancements even if
those advancements occur after lender 2 has recored its interest.

But if lender 1 lends money to debtor and secures a future-advance mortgage which is recorded
but the future advancements are deemed optional then lender 1 will have priority over the 1st
advancement but will be junior to any later mortgagee that advances money to the debtor prior to
any future advancements by lender 1, assuming that the later mortgagee records to put lender 1
on notice.

The policy behind the distinction is that in the case of an obligatory loan, the junior lender can
protect itself by checking the records and ascertaining the maximum amount of the senior
lender's loan. But when those future advances are optional, the junior lender cannot know
whether subsequent advances will be made and the senior lender can protect itself by choosing
not to make those advances if a junior lender will be senior regarding any optional future
advances.
Posted by Sean Silverman at 10:11 AM 0 comments
Email ThisBlogThis!Share to TwitterShare to FacebookShare to Pinterest
Labels: Real Property
Sunday, January 28, 2018
The Rule Against Perpetuities
The dreaded Rule Against Perpetuities. The rule has been described both as a "technicality-
ridden legal nightmare," as well as a "dangerous instrumentality in the hands of most members of
the bar."

And with that said, it's tested on the MBE, so let's try our best to understand it.
First, the rule: No interest is valid unless it must vest, if at all, not later than 21 years after some
measuring life at the creation of the interest. But let's think of this in a more helpful way; if there
is any possibility that an interest in property will not vest within 21 years of someone who is
alive at the time that the interest is granted (a measuring life), then the common law Rule Against
Perpetuities is violated. And so in determining whether the rule has been violated you've got to
come up with a hypothetical situation in which someone who has been granted a non-vested
interest in property might have that property vest more than 21 years after all the measuring lives
have died.

Let's say A grants property to B for "so long as no alcohol is consumed on the property, then to
C." Here B has a fee simple subject to an executory limitation, and C has an executory interest.
But the question is whether the executory interest granted to C is valid. The measuring lives here
are A,B, and C. It's possible that A, B, and C could die and then more than 21 years after the
death of all, B's heirs will violate the condition causing the property to vest in C's heirs. But that
vesting will occur more than 21 years after the death of the measuring lives. And so the
executory interest in C violates the Rule Against Perpetuities, and the grant becomes a fee simple
determinable. If the condition is violated, the interest will revert back to A or A's heirs rather
than shifting to C or C's heirs.

Or let's say that A grants property "to B for life then to B's children for life, then to B's
grandchildren." Assume that at the time of the grant to B, B has two children, C and D, and one
grandchild, E. Again, ask yourself whether it's possible that any of the granted interests might
vest more than 21 years after all the measuring lives have died. The measuring lives here are A,
B, C, D, and E. Assume that a week after the grant C, D, E, and A all die. Sometime later B has
another child, F. Note that F is not a measuring life because F was not alive at the time of the
grant. Then B dies.

So now F is alive and as per the grant, when B dies the property goes to F, B's child. No problem
there since it was certain the property to F would vest within 21 years of a measuring life (here,
B). F lives for another 25 years and then has a child, G. Then F dies. Now the property is
supposed to vest in G, B's grandchild. The problem for G though is that all the measuring lives
(A, B, C, D, and E) have been dead for more than 21 years. If F were a measuring life, there
would be no problem. But F is not a measuring life as discussed above. So, it was possible as
per the grant that a granted interest would vest more than 21 years after the death of all
measuring lives (as has happened here). Because of that possibility, the interest granted to B's
grandchildren violates the rule and should be stricken.

The rule is as horrible as everyone claims it to be. And there are certain details not covered in
this post that could be tested on the MBE. But if you understand the above, you'll begin
to understand how to analyze these problems, and the "legal nightmare" might be a bit less
scary.

Posted by Sean Silverman at 8:33 AM 0 comments


Email ThisBlogThis!Share to TwitterShare to FacebookShare to Pinterest
Labels: Real Property
Thursday, January 25, 2018
UBE Essentials: Jurisdiction in Child Custody Cases
Jurisdiction is not just an issue that shows up in the subject of Civil Procedure on the UBE. It
also shows up in the heavily tested subject of Family Law. Below is a sample from the chapter
on Family Law from my book UBE Essentials covering this topic:

Q: What is the Uniform Child Custody Jurisdiction and Enforcement Act (“UCCJEA”)?

A: The purposes of the UCCJEA are to avoid jurisdictional disputes with courts of other states in
matters involving child custody and visitation. In addition, the Act aims to promote interstate
cooperation, and to facilitate the interstate enforcement of custody and visitation orders.

Q: When does a court have jurisdiction to enter or modify a child custody or visitation order?

A: A court has jurisdiction to enter or modify such an order if the state is the child’s home state.
In addition, jurisdiction is proper even if the state is not currently the home state of the child if
the state was the child’s home state within the past six months and the child is absent from the
state but a parent or person acting as a parent continues to live in the state.

Q: How is "home state" defined?

A: A child’s home state is the state in which the child lived with a parent (or a person acting as a
parent) for at least six consecutive months immediately before the commencement of the
proceeding. Note that if the child is younger than six months old, then the home state will be
where the child has lived since birth, disregarding temporary absences.

Q: Are there other instances in which a court can have jurisdiction over a child custody or
visitation order?

A: If when applying the above rule, no state has or accepts home state jurisdiction, then another
court will have jurisdiction provided that the child and at least one parent (or person acting as
parent) have a significant connection with the state and substantial evidence concerning the child
is available in the state. In addition, a court will have jurisdiction to enter or modify a child
custody or visitation order if no other state has jurisdiction under any other test.

Q: Will a court that has jurisdiction over a child custody or visitation order ever lose that
jurisdiction?

A: A court may lose jurisdiction but it will not lose jurisdiction until the court determines that
neither the child nor the child’s parents (or persons acting as parents) continue to reside in the
state or the child no longer has a significant relationship with the state and substantial evidence
relating to the child is no longer available in the state.

Q: When might a court have emergency jurisdiction over a child custody or visitation matter?

A: A court has temporary emergency jurisdiction if the child has been abandoned or it is
necessary in an emergency to protect the child because the child or a sibling or parent is
subjected to or threatened with abuse.

Preliminary Injunctions vs. Temporary Restraining Orders


There are quite a few similarities between preliminary injunctions and temporary restraining
orders. And like with so many other concepts in Civil Procedure, the differences among similar
concepts are likely to be tested. Here are some points to keep in mind so that you can distinguish
between these different types of injunctions.

Preliminary Injunctions:
It's often stated that a party will seek a preliminary injunction to preserve the status quo while the
suit is pending. The adverse party (the party that will be burdened by the injunction) must be
given notice and an opportunity to be heard. There are some specific elements that must be
satisfied before a preliminary injunction will be granted. It must be true that the plaintiff will
suffer irreparable harm if the injunction is not granted. Further, the harm to the plaintiff must
outweigh the harm to the defendant and the plaintiff must show that s/he is likely to be
successful on the merits of the action. Finally, it must be true that public interest favors granting
the injunction. The granting of a preliminary injunction is within the equitable discretion of the
judge and this is why the element requiring irreparable injury is important to analyze carefully. If
the party seeking the injunction has an adequate remedy at law (for example, if money damages
will suffice), it is unlikely that the preliminary injunction will be granted.

Temporary Restraining Orders:


There are times when irreparable injury might occur if required to wait for the hearing on a
preliminary injunction. In such instances, a party may seek a temporary restraining order to
preserve the status quo until the hearing on the preliminary injunction. Again, there are some
requirements to keep in mind. The adverse party (the party that will be burdened by the order)
must generally be given notice. Importantly, though, a temporary restraining order that will last a
maximum of 14 days may be imposed without notice if the following three elements are
satisfied: the party seeking the order states specific facts in an affidavit or verified complaint of
the irreparable injury that will occur if the order is not granted; the party seeking the order
certifies in writing the efforts made to notify the adverse party and the reasons why notice should
not be required; and the party seeking the order provides security to pay for any damages
incurred by the adverse party if the court later finds the adverse party to have been wrongfully
restrained.

It's worth noting that although a temporary restraining order may be issued without notice of a
hearing under some circumstances, a person must have actual notice of the order before that
person may be held in contempt for violating it.

Trespassers
Lots of trespassers on the MBE. And as such there are some rules to know so that you can apply
them to questions testing the many different angles that the writers of the test can choose to focus
on in the questions.

In these questions the trespasser will be the plaintiff suing the landowner for damages resulting
from harm caused to the trespasser. Step one is to determine whether the trespasser is discovered
or undiscovered. If the trespasser is undiscovered (or unanticipated), then the landowner has a
duty to refrain from willful or wanton misconduct. In other words, not much of a burden here on
the landowner to prevent harm to those who are entering the landowner's land without consent
from the landowner and without knowledge as to their entering. No amount of negligence will
suffice.

On the other hand, there will be times when a landowner knows that trespassers have been on
his/her land in the past and these trespassers are known as discovered trespassers. The burden,
though still rather slight, is increased here since the landowner has knowledge of the entrance. If
the landowner maintains an artificial (as opposed to natural) condition on the land and if that
condition is unlikely to be noticed by a discovered trespasser (for example, if it's concealed),
then the landowner must either make that condition safe or warn the trespassers of the condition
but only if the condition involves a risk of death or serious bodily harm to the trespasser. If the
landowner carries on dangerous activities on the land, then the landowner should exercise
reasonable care in the exercise of such activity.

Lastly, the rules change if the trespasser is a child. A landowner should exercise ordinary care to
avoid foreseeable risk of harm to children caused by dangerous conditions on the property. Note
that here a distinction is generally not made between artificial and natural conditions as it was
above. The landowner here may be liable if the plaintiff can show that there was a dangerous
condition on the property that the landowner should have been aware of, the owner knew or
should have known that children frequent the area, the condition on the land is likely to cause
injury, and the expense of remedying the situation, on balance, would be reasonable when
compared to the magnitude of the risk posed. This specific rule as it relates to trespassing
children is known as the attractive nuisance doctrine.

Posted by Sean Silverman at 10:29 AM 0 comments


Email ThisBlogThis!Share to TwitterShare to FacebookShare to Pinterest
Labels: Torts
Thursday, November 30, 2017
Forum Analysis (First Amendment)
Every subject on the MBE has its hot topics (topics that are heavily tested on the MBE) and one
of those topics within Constitutional Law is the First Amendment. Entire courses in law school
are dedicated to this one amendment and so the depth makes it especially difficult to prepare for
during the short bar prep period. But a very helpful start when tackling this topic is to
understand the difference between content-based restrictions on freedom of speech, content-
neutral restrictions on freedom of speech and restrictions on freedom of speech that primarily
affect conduct related to speech rather than the content of the speech itself. The focus here will
be on the latter but first a bit about content-based restrictions, and content-neutral restrictions.

For the most part (with some exceptions) content-based restrictions on freedom of speech are
presumptively unconstitutional. To be valid, restrictions on content must be necessary to achieve
a compelling government interest. Content-neutral restrictions require a lower level of scrutiny;
namely, such regulations are generally subject to intermediate scrutiny whereby they must
advance important interests unrelated to suppression of speech and must not burden more speech
than necessary.

Unlike with content-based and content-neutral regulations on speech, the government generally
does have the power to regulate conduct associated with speech. But that power is not absolute
and will depend very much on the forum in which the speech is taking place. And that's where
forum analysis comes in.

Public Forums and Designated Public Forums: Public forums are reserved for those areas that
have historically been open to speech-related activities (streets, sidewalks, etc.). In addition,
there are areas that have historically not been open to speech-related activities but which have
been been open by the government for speech-related activities on a limited basis (schoolrooms,
recreations groups, etc.), and these are known as designated public forums. The test for public
forums and designated public forums is the same: the government may regulate speech in these
forums with reasonable "time, place, and manner" regulations that are content neutral, narrowly
tailored to serve important government interests, and leave open alternate channels of
communication.

Limited Public Forums and Nonpublic Forums: Unlike with the forums mentioned above,
there are areas that the government has not historically opened for speech-related activity but
which are opened for a very specific activity (for example, opening up a school gym to host a
debate). These areas are deemed to be limited public forums. There are also areas that are have
not historically been open for speech-related activity and are not open for such on even a limited
basis (jails, military bases, etc.), and these are known as nonpublic forums. There is more leeway
here for government regulation; specifically, the government can regulate speech in such forums
provided that the regulation is viewpoint neutral and reasonably related to a legitimate
government purpose.

**note: There are some outlines that interchange the words "limited" and "designated" public
forums. These forums are quite similar and if there is any distinction to be made it is that with a
designated public forum, the forum has been open more frequently for speech-related activity,
whereas with a limited public forum, the forum is generally open for a very specific purpose.

The Privileges and Immunities Clause (Art. IV vs The 14th Amendment)


As anyone who has or is preparing for the bar exam knows (or will know once it's covered) there
are two privileges and immunities clauses. Although the Privileges and Immunities Clause of
Article IV is tested far more often, it's good practice to understand the differences so that you
don't confuse them in the questions on the exam. Let's focus on those differences:

Privileges and Immunities Clause of the 14th Amendment:

This is the one that students have told me they've been advised not to worry too much about. I
agree with that advice, and yet questions pop up on occasion. States may not deny their citizens
the privileges and immunities of national citizenship. Some examples of these rights are the
right to petition Congress for redress of grievances, the right to vote for Federal officers, and,
importantly, the right to interstate travel. Note that corporations are not protected by this clause.

Privileges and Immunities Clause of Article IV:

This is tested far more often and as such there is more to discuss here. The Privileges and
Immunities Clause of Article IV prohibits intentional discrimination by a state against
nonresidents of the state when the discrimination concerns fundamental rights such as rights
relating to important commercial activities or rights relating to civil liberties. Note well this
qualification, though: if the discrimination applies to a right that is not commercial and does not
involve civil liberties (for example the right of nonresidents to pay the same amount as residents
for a recreational hunting license) then this clause is not the correct one to analyze.

But let's assume that the right does affect an important commercial activity or civil right. In that
case, the state law regulating that right might still be valid but only if the state has a substantial
justification for the different treatment of residents and nonresidents. The state will want to
show that the nonresidents either caused the problem or are part of the problem that the state is
attempting to solve and that there are no less restrictive means by which the state might
undertake to solve the problem.

Here, as in the Privileges and Immunities Clause of the 14th Amendment, corporations are not
protected. In additions, Aliens are not protected.

Shareholders' Lawsuits
Lots of people struggle with the subjects that fall under the category of "business associations."
Corporations is one of those subjects and it's tested frequently on the Multistate Essay Exam. A
large number of the questions dealing with corporations include issues regarding shareholders,
and this post will address one of those issues.

Direct Actions by a Shareholder:

There are times in which a shareholder might feel that a fiduciary duty was breached by either a
director or an officer of the corporation and that the shareholder is affected directly by the
breach. The shareholder (rather than the corporation itself) is affected directly if the shareholder
suffers the most immediate and direct damage and if the defendant's duty of care ran directly to
the shareholder rather than to the corporation. If a shareholder sues in such a situation that is
known as a direct action by the shareholder, and considerations are important because in a direct
action by the shareholder, recovery is for the benefit of the individual shareholder and not the
corporation.

Derivative Actions by a Shareholder:

I see derivative actions tested a bit more frequently than direct actions. In a derivative action, the
shareholder is asserting the corporation's rights (rather than the rights of the individual
shareholder). Recovery here generally goes to the corporation rather than to the shareholder, and
yet interestingly the corporation is still named as the defendant. There is a process here to keep
in mind if a shareholder is to bring a derivative action.

The shareholder must have been a shareholder at the time of the act or omission complained of
or must have become a shareholder through transfer by operation of law from one who was a
shareholder at that time. In addition, the shareholder must adequately and fairly represent the
interests of the corporation. Assuming these requirements are satisfied, the shareholder must
make a written demand on the corporation to take suitable action and the derivative proceeding
may not be commenced until 90 days have elapsed from the date of that demand. The 90-day
requirement will not be enforced, however, if the shareholder has been notified that the
corporation has rejected the demand or if irreparable injury to the corporation would result if
required to wait 90 days.

If a majority of directors (at least 2) who have no personal interest in the controversy find in
good faith after reasonable inquiry that the suit is not in the best interest of the corporation, then
the suit may be dismissed on motion by the corporation. To avoid such dismissal, the
shareholder will have the burden to prove that the decision was not made in good faith after
reasonable inquiry. The burden will shift to the corporation, however, if it's not true that a
majority of directors had no personal interest in the controversy. In that case, the corporation
will have the burden to prove that the decision was made in good faith after reasonable inquiry.

Once the derivative action has ended, the court may order the corporation to pay the plaintiff's
reasonable expenses if it finds that the action has resulted in a substantial benefit to the
corporation. If, however, the court finds that the action was maintained without reasonable cause,
the court may order the plaintiff to pay reasonable expenses to the defendant.

Solicitation
Some of the most difficult questions within the subject of Criminal Law are found within the
topic of inchoate offenses. Within these offenses are the following crimes: solicitation,
conspiracy, and attempt.

Like most aspects of Criminal Law, the elements will guide the analysis. The following are the
aspects of solicitation that often pop up in the questions and therefore should be known well:

Solicitation requires that a person incite, counsel, advise, urge, or command another to commit a
crime with the intent that the person solicited commit the crime. The fact that the person must
intend that the other commit the crime makes this a specific-intent crime. Importantly, it is not
required that the person solicited actually respond affirmatively and commit the crime. So, for
example, if x commands y to commit a crime and x intends for y to actually commit that crime,
the fact that y denies x's command has no bearing on the analysis as to whether x has committed
the crime of solicitation; the crime is in the command itself.

It is also not a defense that the person solicited is not convicted of the offense solicited, nor is it a
defense that the offense solicited could not in fact have been successful. Withdrawal is always
worth considering in Criminal Law, and with solicitation most jurisdictions have held that
withdrawal is not a defense. One important defense to note, however, is a situation in which the
solicitor could not have been found guilty of the completed crime because of a legislative intent
to exempt the solicitor. For example, a minor female cannot be guilty of solicitation for the
crime of statutory rape by urging an adult male to have intercourse with her because the
legislative intent is to not find the minor guilty of statutory rape in such a situation once the
crime is completed.

Let's assume that the person solicited actually completes the solicited crime. In such an instance
both the person solicited and the solicitor can be held liable for that completed crime. Similarly,
if the person solicited commits acts which would qualify for the crime of attempt, both the
person solicited and the solicitor can be liable for attempt. And finally if the person solicited
agrees with the solicitor to commit a crime but then chooses not to commit it, don't forget that
both the solicitor and the person solicited might be liable for conspiracy. In all of these
circumstances, though, the solicitor will not be liable both for the solicitation and the completed
crime. The crime will merge with the solicitation so that the solicitor will be liable either for the
solicitation or the completed crime but not for both.

***Note: Although the default rules on the MBE require a knowledge of the common law, its
important to note that under the Model Penal Code one can renounce (i.e., withdraw from) the
solicitation if the solicitor prevents the commission of the crime such as by persuading the
person solicited not to commit the crime.

Automatic and Temporary Perfection of a Security Interest


To review, there are 5 ways to perfect a security interest as per the rules in Article 9 of the UCC.
The previous 2 posts have discussed 3 of those ways (filing, possession, and control), and this
post will discuss the remaining 2, automatic perfection, and temporary perfection.

Automatic Perfection: The rules you'll need to know about automatic perfection are very
limited. Only a purchase money security interest ("PMSI") in consumer goods is automatically
perfected. A seller of goods has a PMSI when the security interest is retained to secure at least
part of the purchase price of the goods. So, if a seller of consumer goods lends money to the
buyer and retains a security interest to secure that loan (i.e., ensure repayment) then that security
interest will be automatically perfected with no further action required to perfect it. It's
important to note that not all PMSI's are eligible for automatic perfection. If the PMSI is in
inventory or equipment, for example, then you should not apply the rule regarding automatic
perfection; it should only be applied to consumer goods. In addition, there is an exception for
motor vehicles to keep in mind: a security interest in motor vehicles can be perfected only by
notation on the vehicle's certificate of title.

Temporary Perfection: The first place to begin when discussing temporary perfection of a
security interest is with proceeds. A security interest in proceeds from original collateral is
continuously perfected for 20 days from the debtor's receipt of the proceeds. This is automatic
perfection but the security interest will become unperfected after 20 days unless the statutory
requirements are followed. The security interest, however, will continue beyond the 20 days if:

(1): The security interest in the original collateral was perfected by filing a financing statement, a
security interest in the type of collateral constituting the proceeds would be filed in the same
place as the financing statement for the original collateral, and the proceeds were not purchased
with cash proceeds of the collateral.

or

(2): The proceeds are identifiable cash proceeds.

or

(3): The security interest in the proceeds is perfected within the 20-day temporary perfection
period.

In addition to proceeds, there are a few other types of security interests eligible for 20-day
temporary automatic perfection. The first deals with instruments, negotiable documents, and
certificated securities. Where new value is given under an authenticated security agreement for
instruments, negotiable documents, or certificated securities, perfection is valid for 20 days after
attachment; nothing further is required to perfect temporarily.

In addition, where a creditor has perfected a security interest by possession and delivers to the
debtor instruments, negotiable documents, certificated securities, or goods in the possession of a
bailee, perfection will continue for 20 days after which the creditor must re-perfect (since the
creditor no longer has possession).

An example here might be helpful: Assume that the creditor possesses a promissory note as
collateral for a loan given by the creditor to the debtor. The note has been perfected by
possession but at some point the creditor must give the note to the debtor so that the debtor can
present it for payment. Perfection of the security interest will not be lost on that note the
moment that the creditor stops possessing it, but the creditor will need to perfect (for example
file or re-possess) the note within those 20 days or else lose perfection.

Perfection of a Security Interest by Possession or Control


In the last post I discussed how one might perfect a security interest by filing a security
agreement. There are 5 ways in total to perfect a security interest, and this post will be focus on
2 of the remaining 5, possession and control.

Possession:

Security interests in most types of collateral can be perfected by possession. There are some
types of collateral, however, that cannot be perfected by possession and they should be kept in
mind. The types of collateral that cannot be perfected by possession are general intangibles,
non-consumer deposit accounts, non-negotiable documents, electronic chattel paper, certificate
of title goods, and accounts. In other words, to perfect a security interest in these types of
collateral will require perfecting by a method other than possessing the collateral.

If perfecting by possession, the security interest will be perfected from the moment of possession
and will continue as long as possession is retained. When the collateral is being possessed by a
bailee, possession will begin the moment a bailee authenticates a record acknowledging that it is
holding the collateral for the benefit of the secured party.

Control:

As with possession, there are certain types of collateral that can be perfected by control, and here
they are rather limited. They are non-consumer deposit accounts, electronic chattel paper, and
investment property. Regarding non-consumer deposit accounts, a bank in which a non-
consumer deposit account is maintained will automatically have control over the deposit account.
If the secured party is not such a bank, it may obtain control over a non-consumer deposit
account by putting the deposit account in the secured party's name or by agreeing in an
authenticated record with the debtor and the bank in which the deposit account is maintained that
the bank will comply with the secured party's orders regarding the deposit account without
requiring the debtor's consent.

As to electronic chattel paper (chattel paper stored in an electronic medium such as a computer),
it is controlled when a system is put in place to show the transfer of interests in the chattel paper
which reliably establishes the secured party as the assignee.

And finally, there is investment property. One can gain control over a certificated security (such
as a stock or bond represented by a certificate) by taking possession of the certificate if it is in
bearer form. If, however, the certificate is in registered form, the secured party must take
possession and the certificate must be indorsed to the secured party or registered by the issuer in
the name of the secured party.

If the investment property is a securities account rather than a certificated security, then one will
obtain control over that account if the owner of the account instructs the securities intermediary
that the secured party has the same rights in the account as the owner or if the owner instructs the
intermediary that the intermediary may comply with the secured party's orders without the
owner's further consent.

Perfection of a Security Interest by Filing


Once a security interest has attached, it's then important to determine whether the requirements
for perfecting that security interest have been satisfied. By perfecting the interest a party will
have priority over most other third parties (though well known by anyone who has studied this
subject, there are exceptions).

There are five ways to perfect a security interest. Throughout the next five posts I'll be reviewing
each in detail. If an essay question shows up on the bar exam testing secured transactions, the
probability is quite high that an issue dealing with perfection will be included.

The five methods to perfect a security interest are as follows:

(1): Filing
(2): Taking possession of the collateral
(3): Control
(4): Automatic perfection
(5): Temporary perfection

To file a security interest requires filing a financing statement which can be filed either
electronically or in writing. The financing statement must contain the debtor's name and mailing
address, the secured party's name and mailing address, an indication of the collateral covered by
the financing statement and if the financing statement covers real-property-related collateral a
description of the real property, the name of the record owner, and an indication that it is to be
filed in the real property records.

A minor error in the debtor's name will not invalidate a financing statement but seriously
misleading errors will. If the debtor's name becomes insufficient and therefore seriously
misleading (because, for example, the debtor changes his/her name) then the financing statement
will only be effective against collateral acquired by the debtor before the name became
insufficient and within 4 months after. After that point, the secured party must re-file using the
debtor's correct name.

If the filing office chooses to accept a financing statement that does not contain the debtor's
address, the statement is effective despite the lack of address. The financing statement must
indicate the type of collateral covered by the statement but it may do so broadly (for example
"equipment"). It may even include a "supergeneric" description such as "all assets."

The debtor must authorize the financing statement in an authenticated record either before or
after it is filed. Filing should be done centrally in the office of the secretary of state. One
exception to the place of filing deals with filing for security interests in timber to be cut, minerals
and fixtures. Those interests should be filed locally in the county where a mortgage on real estate
is filed.

Filing is valid for 5 years. A continuation statement may be filed which will extend this 5 year
period for another 5 years but may only be filed within 6 moths before the lapse of the first 5
year period.
Next up will be a post on another method for perfecting a security interest: taking control of the
collateral.

And, yes, I know that is the worst cliffhanger ever. :)

Service of Process (Out-of-State Defendants)


A recurring question has been coming up with my tutoring students over the last week, so I
thought it a good idea to address it on the blog. It deals with the federal rules for service of
process of out-of-state defendants and the rule can definitely be a bit complex.

The rule regarding service of process to a party (an individual) served in the same state in which
the court sits is straight-foward enough: any person who is 18 years old and not a party to the
action may serve process at the defendant's usual place of abode with one of suitable age and
discretion who resides there. In addition, service may be upon an authorized agent of the
defendant. There are slightly different rules if dealing with an organization (for example, a
corporation), a minor, etc.

But the rules change when the person to be served is outside of the state in which the federal
court sits. First look to the long-arm statute of the state in which the federal court sits. The long-
arm statute might guide as to the rules regarding serving a person out of state. In addition, there
is a specific rule for serving third-party defendants (impleader, rule 14), and indispensable
parties (rule 19). The rules provide that third-party defendants or parties required to be joined
may be served if served within 100 miles from the place where the action is pending. This is
sometimes referred to as the "bulge provision."

Finally, out-of-state service may be permitted if permitted by federal statute and for cases that
involve a federal question (only a federal question, so this provision does not apply to cases that
are in federal court based on diversity) if a defendant is served with process and is not subject to
general jurisdiction in any state, provided that the defendant also has sufficient contacts with the
United States and that jurisdiction is not prohibited by statute.

MBE Fast Fact: Complaints


It's important to understand the standard for complaints in a civil case. As per the Federal Rules
of Civil Procedure, a complaint must state: (1) the grounds of federal jurisdiction; (2) a short
statement of the claim showing that the pleader is entitled to relief; (3) a demand for judgment
for relief which may be in the alternative. Importantly, the facts stated in the complaint should
support a plausible and not just possible claim. Seems like a rather fine distinction to make, but
the MBE tends to test such things.

One other point to note: if alleging fraud or mistake, then the standard stated above is slightly
modified. Here, a party must state with particularity the circumstances constituting fraud or
mistake. For example, when alleging fraud, the complaint should specifically state the precise
misconduct that constitutes the fraud.

MBE Fast Fact: Diversity Jurisdiction & Limited Partnerships


When determining the citizenship of a limited partnership for purposes of diversity, be sure to
assess the citizenship of all partners, general and limited. For example, assume the plaintiff is
suing a limited partnership in federal court on a state-law claim. Plaintiff is from state A, 4
general partners are from state B, and 3 limited partners are from state A. Because there are
limited partners who are citizens of the same state as the plaintiff, diversity is destroyed and
there will be no subject matter jurisdiction over this claim in federal court. Importantly, it's not
only the citizenship of the general partners that are assessed when making this diversity
determination.
Posted by Sean Silverman at 1:10 PM 0 comments
Email ThisBlogThis!Share to TwitterShare to FacebookShare to Pinterest
Labels: Federal Civil Procedure
Wednesday, June 28, 2017
Electronically Stored Information
Discovery is an area that should be known well when preparing for Civil Procedure on the MBE.
A specific sub-topic within the larger topic of discovery deals with discovery of electronically
stored data. Nothing too complicated here, but certainly worth taking the time to understand the
following:

The parties in a civil case are required to preserve electronically stored data. A party can request
of the other party the form in which the electronically stored data should be produced, and the
responding party must use that particular form unless it objects. If the party does object, the court
will then determine if the objection is valid. It may be the case that the requesting party does not
specify the specific form in which the electronically stored data should be produced; in that case
the responding party may use any form in which the information is maintained or a form that is
reasonably usable by the requesting party.

Issues may arise in which one party claims that another has destroyed electronically stored
information. A party must take reasonable steps to preserve electronically stored information.
Importantly, if the party satisfies that standard of reasonableness then no sanctions may result
from the destruction of the information. In addition, if unpreserved electronically stored
information can be restored, then no other remedial measures (such as sanctions) will be
imposed.

If, on the other hand, the data cannot be restored and the discovering party is prejudiced by the
lack or restoration the court may order remedial measures but such measures can be no greater
than necessary to cure the prejudice.

Note that if the disclosing party acted with intent to deprive the discovering party of the
electronically stored information, the court may take additional remedial measures such as
presuming the lost information was unfavorable to the disclosing party and/or instructing the jury
that it may or may not presume that the information was unfavorable to the disclosing party. In
addition, the court can dismiss the action or enter a default judgment against the party who
destroyed the electronically stored data.

Alienage Jurisdiction
Questions on the MBE come up quite often testing your understanding of diversity jurisdiction.
One of the difficulties of this topic is that there seems to be an endless amount of angles to test.
Here's one more:

As a quick review, diversity jurisdiction requires diversity of citizenship in which the dispute
involves citizens of different states within the United States. No plaintiff can be a citizen of the
same state as any defendant or diversity will be destroyed and jurisdiction in the federal court
will be improper unless there is another basis for claiming that subject matter jurisdiction is
proper (such as federal question jurisdiction.)

A lesser known rule allows for subject matter jurisdiction over alienage cases in which the
dispute is between a citizen of a U.S. state and a citizen of a foreign country. Importantly, this
basis for jurisdiction will not apply if the citizen of the foreign country (an "alien") has been
admitted to the United States for permanent residence and is domiciled in the same state as the
U.S. citizen. In other words, when such occurs, diversity is destroyed just as it would be if the
case involved a plaintiff and defendant, both of whom were citizens of the same U.S. state.

It should also be noted (and this comes up in questions as well) that there is no subject matter
jurisdiction over cases by an alien against an alien; there must be a citizen of a U.S state on one
side of the suit to qualify for alienage jurisdiction.
Posted by Sean Silverman at 2:39 PM 0 comments
Email ThisBlogThis!Share to TwitterShare to FacebookShare to Pinterest
Labels: Federal Civil Procedure
Wednesday, June 14, 2017
Future Interests
Future Interests has got to be one of the most dreaded topics in Real Property. It takes some time
and struggling to get this stuff but as an initial matter it's important to understand which type of
future interest goes with which type of present possessory estate. That's the first step in
understanding this very difficult concept. So, let's begin there:

Bellow #s will be listed as follows:

Present possessory estate--->future interest associated with that estate

(1): Fee simple absolute--->No future interest

(2): Fee simple determinable-->Possibility of reverter

(3): Fee simple subject to condition subsequent--->Right of entry

(4): Fee simple subject to executory interest--->Executory interest

(5): Life estate-->Remainder.

As an explanation if you look at (2) above, that means that if a fee simple determinable is
granted to someone (let's call that person y) then the future interest associated with that grant is a
possibility of reverter. Assume that grantor x grants to y property so long as y uses the property
for farming. Such a grant fits the definition of fee simple determinable, and so y has been
granted a fee simple determinable as a present possessory estate. X, the grantor, retains a
possibility of reverter as a future interest. If y fails to satisfy the stated condition by not using the
property for farming then the possibility of reverter will force the property to revert back to x and
x's future interest (a possibility of reverter) will become a present possessory estate (a fee
simple).

The same logic can be applied for all of the above, though there are some slight differences
depending upon the present estates and future interests.

It's quite complex but it certainly helps to know the above as you begin to delve deeper into this
area.

Appeals: A Quick Summary


Quite a lot to know about appeals when preparing for Civil Procedure questions on the MBE.
I've written a bit about this previously but this post will summarize the important points to keep
in mind:

The federal courts have subject-matter jurisdiction over appeals from all final decisions of the
federal district trial courts. However, as a general matter the federal courts do not have subject-
matter jurisdiction over appeals from interlocutory matters (those matters that have not reached
final judgement) of the federal district trial courts.

But there are exceptions, and one important exception deals with injunctions. The federal courts,
for the most part, do have subject-matter jurisdiction over appeals from interlocutory orders that
grant, deny, or modify an injunction. And in addition to this specific rule regarding injunctions,
the collateral order doctrine provides that a federal trial judge's interlocutory order is reviewable
if it conclusively determines claims of right distinct from and collateral to the rights asserted in
the action and would be effectively unreviewable if the litigant were required to wait for an
appeal.

Regarding the scope of review by a federal court, an alleged error at trial is reviewable on appeal
only if preserved on the record. And even if an error is preserved, it may be unreviewable if a
court determines that the issue preserved constitutes harmless error (an error that does not affect
the substantial rights of the parties).

Finally, as to the standard of review by an appellate court, this will depend upon whether the
alleged error at trial concerned an (1) interpretation of law, (2) an exercise of the court's
discretion, or (3) a finding of fact.

Specifically, the appellate court will exercise de novo review of a trial court's conclusions of law.
In other words, the appellate court will exercise its own judgment on decided legal issues. If
reviewing a trial court's discretionary ruling that do not implicate legal issues, an appellate court
will overturn such discretion by the trial court if the appellate court determines that there has
been an abuse of discretion or plain error. And when reviewing findings of fact by the trial
court, an appellate court will only overturn such factual findings if such findings were made by a
court (not a jury) and if such findings are clearly erroneous.
MBE Fast Fact: The Right to a Speedy Trial
Though not tested all that frequently in Criminal Procedure questions, the right to a speedy trial
is tested enough that I recommend knowing a couple of key points here. This right attaches once
a person has been formally accused of a crime by the government. If the right is violated, the
result will be a complete dismissal of the charges against the accused. There is a balancing test
that courts use to determine whether the right has been violated. Consider the following factors
in any question testing the right to a speedy trial:

--The length of the delay


--The reason for the delay
--The defendant's assertion of his or her right to a speedy trial
--The prejudice to the defendant resulting from the delay.

Regarding that last factor, there are quite a few ways that defendant might be prejudiced by the
delay. Perhaps evidence is lost and the loss is caused by the delay. In addition, an undue delay
might cause witnesses to be less able to recollect facts that will form the basis of testimony.
Even excess anxiety caused to the defendant resulting from the delay will suffice.

MBE Fast Fact: The Taxing Power


I remember my Constitutional Law professor in law school mentioning that Constitutional Law
can be broken down into two main areas: powers and limitations. I think he had a point.

There are quite a few congressional powers and one of those powers is the power to tax.
Specifically, Congress has the power to lay and collect taxes, duties, imposts, and excises to pay
the debts and provide for the common defense and general welfare. And so with this rule in
mind, it follows that a congressional act purporting to be a tax will be upheld if it raises revenue
or if it was intended to raise revenue as raising revenue is deemed to be providing for the
common defense and general welfare.

In addition, as long as Congress has the power to regulate the activity taxed, the necessary and
proper clause can form a basis for Congress to tax that activity.

So, in short, it's best to remember that if Congress has the right to regulate an activity (as per a
power granted to it in the Constitution) then the tax is very likely to be upheld as per the
necessary and proper clause. But even when Congress does not have the power to regulate an
activity taxed, a tax will likely be upheld if its dominant intent is to raise revenue.

Types of Collateral
Article 9 of the UCC (Secured Transactions) is difficult. The complexity makes it daunting in
the relatively short time available for students to learn it for the bar exam. But an important first
step in getting a grip on this subject is to understand the different categories of collateral that can
form the basis of a security interest. The rules throughout this subject will often refer back to
these categories (for example, some rules only apply to consumer goods, a type of tangible
collateral) and building this foundation by simply understanding the types of collateral can set
you up to far better understand the subject as a whole.

In total there are 3 main categories with sub-categories within each one to keep in mind. The
three main categories are tangible collateral, intangible collateral, and proceeds.

I: Tangible Collateral (often referred to as "goods"):

(1): Consumer goods:


These are goods bought or used for personal, family, or household purposes.
(2): Inventory:
These are goods held for sale or lease and goods consumed by a business.

(3): Farm products:


These are goods used or produced in farming that are in the possession of or used by a farmer.

(4): Equipment:
These are goods that do not fit within any of the 3 above categories.

II: Intangible Collateral

(1): Instruments:
Instruments include notes, drafts, and certificates of deposit.

(2): Documents:
Documents include bills of lading and warehouse receipts.

(3) Chattel Paper:


Chattel paper are records evidencing both a monetary obligation and a security interest in
specific goods or a lease of specific goods.

(4): Accounts:
These are rights to payment for goods, services, etc.

(5): Deposit Accounts:


These are savings accounts, passbook accounts, etc.

(6): Investment Property:


These include stocks, bonds, mutual funds, brokerage accounts, etc.

(7): Commercial tort claims:


These are tort claims filed by organizations and tort claims filed by an individual that arose out
of the individual's business and do not involve personal injury.

(8): General intangibles:


These are intangibles not fitting within any of the above 7 types of intangibles.

III: Proceeds

Proceeds include whatever is received upon the sale, exchange, collection or other disposition of
collateral or other proceeds. Insurance payable by reason of loss or damage to collateral is also
deemed to be proceeds unless it is payable to someone other than the debtor or secured party.

Posted by Sean Silverman at 12:03 PM 0 comments


Email ThisBlogThis!Share to TwitterShare to FacebookShare to Pinterest
Labels: Secured Transactions
Sunday, March 26, 2017
The Uniform Child Custody Jurisdiction and Enforcement Act
An area of some complexity that shows up in Family Law is jurisdiction for child custody cases.
And when I'm working with students who are preparing for the MEE, it's an area we spend some
time on because it's important to know it well should it show up on the exam. In other words, it
could amount to a large percentage of the points in any given essay.

The Act to understand well in this regard is the Uniform Child Custody Jurisdiction and
Enforcement Act which is far too many words and so after mentioning it once in an essay, just
call it the UCCJEA. The purpose of the Act is to avoid jurisdictional disputes with courts of
other states in matters of child custody and visitation as well as promote interstate cooperation. It
also aims to facilitate the interstate enforcement of custody and visitation orders.

And so a question may be raised in a fact pattern as to which state has jurisdiction to initially
enter or to modify a child custody or visitation order. First, we look to the home state of the
child. A child's home state is the state in which the child lived with a parent (or a person acting
as parent) for at least six consecutive months immediately before the commencement of the
proceeding. If the child is less than six months old then the home state will be the state where
the child has lived since birth disregarding temporary absences. A court in the home state of the
child will have jurisdiction to enter or modify a custody or visitation order

It's possible, though, that a child had a home state (had been living in the state for at least 6
consecutive months) and within the last six months has moved out of state. A court in the state
that was recently the child's home state will have jurisdiction to enter a custody order if a parent
or person acting as a parent continues to live in that state.

Further, it's also possible that no state will satisfy the home state test as stated above. If no state
has or accepts home state jurisdiction then a court will have jurisdiction to enter or modify a
custody or visitation order if that court sits in a state in which the child and at least one parent (or
persons acting as parents) have a significant connection and if substantial evidence concerning
the child is available in that state.

When analyzing jurisdiction for these purposes, I would first look to apply the home state rules
and then move on to the significant connection test if necessary. And then note that the court
that made the initial custody or visitation determination has exclusive continuing jurisdiction
over the matter until neither the child nor the child's parents (or persons acting as a parent)
continue to reside in the state or the child no longer has a significant connection with the state
and substantial evidence relating to the matter is no longer available in the state.

Even if a court has jurisdiction as outlined above, the court may choose to decline jurisdiction if
it determines that it is an inconvenient forum under the circumstances and that a court in another
state is a more appropriate forum. Finally, note that under some extreme circumstances, a court
may have temporary emergency jurisdiction even if the general jurisdiction rules as outlined
above are not satisfied. This should be applied sparingly, though; a court will have temporary
emergency jurisdiction if the child has been abandoned or if it is necessary in an emergency to
protect the child because the child, the child's sibling(s), or the child's parent is threatened with
or subjected to abuse.

Posted by Sean Silverman at 9:55 PM 0 comments


Email ThisBlogThis!Share to TwitterShare to FacebookShare to Pinterest
Labels: Family Law
Tuesday, March 21, 2017
Actual, Apparent, and Inherent Authority
If you happen to get a Partnerships essay on the MEE, the chances are quite high that Agency
issues will be mixed within. And a very common Agency issue deals with the authority that an
agent (for example a partner in a partnership) has to act for the principal (for example, the
partnership). This post will review three types of authority that should be known well going into
the exam:

Actual Authority: Actual authority is the authority that the agent reasonably believes s/he
possesses based on the principal's dealings with the agent. This type of authority may be express
or implied. Express is pretty straight-forward; look for an agency agreement and if the authority
is stated within that agreement then express authority has been granted to the agent. Implied
authority is that which the agent reasonably believes s/he has as a result of the principal's actions
even if there is no express agreement stating as such.

Apparent Authority: Apparent authority arises from the reasonable beliefs of third parties. In
other words, if a principal directly or indirectly holds out another as possessing certain authority
and such holding out induces reasonable reliance by another that the agent has such authority, the
agent will have apparent authority to act on behalf of the principal even if as between the agent
and principal the definition of actual authority (either express or implied) has not been satisfied.
One way to think about this is that the principal will be estopped from denying that the agent has
authority to act for the principal if the principal has acted in a way that would lead a reasonable
person to believe that such authority was in fact granted.

Inherent Authority: Even if the agent has no actual or apparent authority, the agent might still
have the inherent authority to act on behalf of the principal. Sometimes, courts wish to protect
innocent third parties rather than the principal when an agent acts on behalf of the principal. As
such, the law will hold the principal liable for the acts the agent. A very common example of
inherent authority is respondeat superior in which an employer will be held liable for the acts of
an employee that occur within the scope of the employee's employment.

There is more to say on this topic for sure (for example, how authority is terminated.) Future
posts will delve deeper into this commonly tested area.
Posted by Sean Silverman at 7:16 PM 0 comments
Email ThisBlogThis!Share to TwitterShare to FacebookShare to Pinterest
Labels: Partnerships & Agency
Sunday, March 12, 2017
Limited Liability Companies
Limited Liability Companies ("LLC's") is a trendy topic on the MEE. So much so that the most
recent update to my book MEE Essentials will contain a more in-depth discussion of this topic.
A foundational area to understand here is the differences between member-managed LLC's and
manager-manager LLC's.

Member-Managed LLC's:
With this type of management structure, each member owes to each other member as well as to
the LLC duties of care and loyalty. In addition, each member must discharge their obligations
consistent with good faith and fair dealing. Most importantly to remember here is that there are
obligations that members have that they do not necessarily have under other management
structures such as will be listed below (under manager-managed LLC's).

Regarding the duty of loyalty, a member must account to the LLC for any benefit derived in
connection with the LLC's business and must refrain from dealing adversely with the LLC
(unless the transaction is deemed fair by the LLC). Further, the members must refrain from
competing with the LLC's business. These acts, however, may be authorized even though they
would generally violate the duty of loyalty if authorization is provided by the other members
after full disclosure has been provided to the other members.

Under the duty of care, members must act with the care that a person in a like position would
exercise under similar circumstances, in a manner reasonably believed to be in the best interest
of the LLC. This standard shows up elsewhere and is known as the 'business judgement rule.'

Manager-Managed LLC's:
Here, the duties of loyalty and care are different for managers and members. Although both
retain the obligation of good faith and fair dealing, only the managers are subject to the duties of
loyalty and care as discussed above for member-managed LLC's. Further, only the members
may authorize or ratify an act by a manager that would otherwise violate the duty of loyalty.

Lastly, it should be noted a member-managed LLC is presumed unless the operating agreement
of the LLC provides otherwise.

Spendthrift Provisions (Trusts)


A common question I often get in the final weeks before the bar exam deals with study strategy.
As the exam approaches it becomes much harder to cover everything that might potentially show
up on the exam. I think it's important to be sure not to go into the test having not reviewed a
subject hoping that subject won't show up. But it is also important, especially when time is
short, to focus your studying on the subjects that are most likely to be tested.

Trusts has shown up with some real frequency on the MEE. A spendthrift clause in a trust
precludes a beneficiary from voluntarily or involuntarily transferring his/her interest in the trust.
As such, creditors are precluded from reaching the trust to satisfy their claims. Importantly, note
that once income has been distributed to the beneficiary, the creditors can then reach that
income; it is only the income interest that the creditors cannot reach!

There are some exceptions to note: A spendthrift clause cannot be used shield the beneficiary
from his/her own creditors where the beneficiary is also the settlor. In other words, a settlor
cannot create a trust whereby the settlor is the beneficiary of the trust with a spendthrift clause if
the purpose was merely to shield him/herself from creditors Further, claims for support,
alimony, and necessities as well as claims by the government are valid and will withstand a
spendthrift provision. Lastly, any creditor can reach a mandatory distribution of income or
principal if the trustee did not make that distribution within a reasonable time.

MEE Fast Fact: Attachment (Secured Transactions)


Not only is secured transactions a very complex subject but it is tested with some frequency on
the MEE. There's plenty to say about this subject but an initial place to begin is with the
concept of attachment. A security interest will not be enforced unless it has first attached.
Essentially, this means that the secured party will have no rights against the debtor unless there
has been an attachment of a security interest. And attachment requires the following:

(1): The parties (the secured party and the debtor) must agree to create a security interest.
Evidence of this agreement can be accomplished in a few ways, one of which is the creditor
taking possession of the collateral (personal property of the debtor). In addition, the interest can
be evidenced by the debtor authenticating a security agreement describing the collateral. Finally,
the agreement can be evidenced by the secured party taking control of certain types of collateral.

(2): In addition to the evidence outlined above, value must be given by the secured party to the
debtor.

(3): Lastly, the debtor must have ownership rights in the property used as collateral.

If all three elements above are satisfied, a security interest has attached. That's just the
beginning, but it's a good place to start.
Posted by Sean Silverman at 9:10 AM 0 comments
Email ThisBlogThis!Share to TwitterShare to FacebookShare to Pinterest
Labels: Secured Transactions
Monday, January 30, 2017
MEE Posts Coming Soon
These days, I tutor almost as much for the Multistate Essay Exam as I do for the Multistate Bar
Exam. As such, I'd like to start posting on the essay subjects tested on the MEE (that are not
tested on the MBE) as well as continue to post on the MBE subjects. The subjects covered in
these posts will be the following:

Agency & Partnerships


Corporations & LLCs
Family Law
Conflict of Laws
Secured Transactions
Wills
Trusts

As always, I hope that you find the posts helpful in your preparation for the exam!
Posted by Sean Silverman at 11:58 AM 0 comments
Email ThisBlogThis!Share to TwitterShare to FacebookShare to Pinterest
Labels: Miscellaneous
Monday, January 23, 2017
Use Immunity and Impeachment
Came across an interesting issue in a tutoring session that I thought worth posting about:

Assume a situation in which a witness at a grand jury proceeding refuses to answer a question by
claiming the 5th Amendment privilege against self-incrimination. Use immunity is granted and
the witness then answers the questions asked. Later when the case goes to trial, the witness
states something contradictory to what was stated at the grand jury proceeding, and the
prosecution wants to use the previous statement at the grand jury proceeding as a prior
inconsistent statement for purposes of impeaching the witness at trial. Will the fact that the
witness was granted use immunity at the grand jury proceeding prevent the prosecution from
using that statement during the trial to impeach the witness?

The use immunity will not prevent the opportunity for impeachment by the prosecution. A
witness who testifies under a grant of use immunity is protected against use of that testimony in
any subsequent criminal trial against that witness. But since in the above hypothetical the
witness was not the defendant in the subsequent trial, use immunity will not prevent offering the
statement made at the prior grand jury proceeding for the purposes of impeaching that witness at
trial.

Posted by Sean Silverman at 5:03 PM 0 comments


Email ThisBlogThis!Share to TwitterShare to FacebookShare to Pinterest
Labels: Evidence
Saturday, January 21, 2017
MBE Fast Fact: Remand
There are a few posts here on the blog focusing on the ability of a defendant to remove a case to
federal court if the plaintiff has chosen to bring the case in state court. And no doubt, knowing
the details of removal is very important for the MBE. There are also times when after the
defendant has removed the case to federal court, the plaintiff will have the opportunity to remand
the case back to state court.

In general it's important to note that the first step to get the case remanded back to state court is
for the plaintiff to make a motion to have the case remanded. A case will be remanded if there is
no federal jurisdiction. But even if there is federal jurisdiction and even if the prior removal to
federal court by the defendant was entirely proper, the federal court has discretion to remand a
case to state court if federal jurisdiction was based on the fact that there were federal claims at
the time of removal and the only claims that remain are state claims over which there is no
diversity jurisdiction.

And if the case is remanded, the order remanding the case may require payments of costs and
expenses including attorney fees incurred as a result of the removal of the case to federal court.
Posted by Sean Silverman at 7:39 AM 0 comments
Email ThisBlogThis!Share to TwitterShare to FacebookShare to Pinterest
Labels: Federal Civil Procedure
Sunday, January 15, 2017
Expert Witnesses
Expert witness questions can show up both in Evidence questions and in Civil Procedure
questions. In Evidence, the questions are likely to focus on such topics as the qualifications to be
an expert witness, the bases of testimony for an expert, and the proper subject matter for expert
testimony.

For Civil Procedure, it's important to go into the exam understanding the obligations that a party
has towards the opposing party if a party intends to use an expert witness at trial. Knowing the
following should put you in a good position to answer these questions correctly:

A party using an expert witness must automatically supply to the opposing side a written report
containing all of the following:
(1): A complete statement of all opinions the expert witness will express and the basis and
reasons for those opinions.

(2): The facts or data considered by the expert witness in forming the opinions that the witness
will express.

(3): Any exhibits that will be used to summarize or support the opinions that the expert witness
will express.

(4): The expert witness's qualifications, including a list of all publications authored in the
previous 10 years.

(5): A list of all other cases in which, in the previous 4 years, the witness testified as an expert
either in trial or by deposition.

(6): A statement of the compensation to be paid to the expert.

Posted by Sean Silverman at 7:34 AM 0 comments


Email ThisBlogThis!Share to TwitterShare to FacebookShare to Pinterest
Labels: Federal Civil Procedure
Sunday, January 8, 2017
Accretion vs. Avulsion
Even though I've been teaching the MBE for many years now, occasionally I'll come across a
distinction in a question that surprises me. It may seem rather insignificant with so much else to
know in the area of Real Property, but a testable (and tested) distinction is that between avulsion
and accretion. Here's what's to know:

Both of these concepts deal with the fact that the natural effects of water upon land has the
potential to change the nature and extent of the land in a way that might affect real property
rights.

Accretion: A key point to keep in mind regarding accretion is that the changes to the land
happen over a lengthy period of time. For example, if you were to view over a long period of
time the bank of a river or the shoreline of a lake you would notice that the water from the river
or lake was depositing soil on the bank or the shoreline. And this deposit of soil is known as
accretion. Importantly for the MBE, land formed by accretion is generally recognized to be
owned by the owner of the bank or the shoreline upon which the accretion occurs.

Avulsion: An important distinction between avulsion and accretion is one of time. Whereas
accretion takes place over a long period of time, avulsion is a sudden change in land (for
example by depositing soil far downstream) brought about by water. It may result in addition or
removal of land from a bank or shoreline. Importantly, the law generally provides that soil
removed by avulsion remains the property of the original owner.

There is one other point worth noting about avulsion. There may be a situation in which a
stream acts as a boundary between two parcels of land. Avulsion might cause that boundary to
shift, and when it does the law generally holds that the landowners will retain the property that
they owned before the shift in the course of the stream if the shift is due to avulsion.
Posted by Sean Silverman at 1:18 PM 0 comments
Email ThisBlogThis!Share to TwitterShare to FacebookShare to Pinterest
Labels: Real Property
Tuesday, December 20, 2016
MBE Fast Fact: Modification of Contracts
Whenever the rules differ between the common law of contracts and the UCC, an added level of
difficulty exists within the question. One topic that stands out in this regard deals with
modification of contracts, and two types of questions that show up quite a lot within this topic
deal with the need for consideration to modify a contract and the need for a writing to modify a
contract

Consideration: Under the common law of contracts, a contract cannot be modified unless the
modification is supported by new consideration. The UCC, however, differs. Under the UCC,
good faith promises of new and different terms than were stated in the original contract between
the parties are valid even if those new and different terms are not supported by any additional
consideration.

Writing: Generally, and under the common law of contracts, a written contract can be modified
orally. And furthermore, under the common law, even if a written contract expressly provides
that a contract may be modified only by a writing, the parties may still orally modify the
contract. Under the UCC, though, any modification must be in writing if the contract as modified
falls within the Statute of Frauds. What this means is that if the contract as modified is for $500
or more (thereby falling within the UCC Statute of Frauds), the modification must be evidenced
by a writing. If, however, the contract as modified is for less than $500, then no writing is
required.

One other point worth noting is that the parole evidence rule will not prevent the admissibility of
evidence dealing with the modification of a contract. This is because the parole evidence rule
only applies to exclude from evidence written or oral statements made prior to the writing as well
as oral statements made contemporaneously with the writing if those statements are going to
contradict the writing and if the writing was intended as the final expression of the bargain
between the parties. But modifications fall outside the scope of the parole evidence rule because
by their nature they occur after the written agreement has already taken place (as opposed to
prior to or contemporaneously with the writing).
Posted by Sean Silverman at 5:11 PM 0 comments
Email ThisBlogThis!Share to TwitterShare to FacebookShare to Pinterest
Labels: Contracts/Sales
Friday, December 9, 2016
Equitable Conversion
I've written a bit about equitable conversion here on the blog. The general rule is important to
note but an additional point that comes up in the practice questions is worth noting as well. As is
usual, the more practice questions you work through, the more angles you'll see as to each issue
that could be tested, and the less likely it'll be that you'll face too many 'curve balls' on the exam.
There are two components to doing well on the MBE: knowing the law and knowing how to
apply the law. As difficult as this test may seem, when you improve at these two components
you will see your scores go up quickly and significantly.

Under the doctrine of equitable conversion, the risk of loss of realty passes to the buyer as soon
as a contract of sale is executed between the buyer and seller. But there is an additional point
worth noting. Even in those jurisdictions that choose not to follow the doctrine of equitable
conversion (so that risk of loss would remain on the seller until title or possession is transferred
to buyer), if there is destruction of an immaterial part of the realty prior to the time that the seller
transfers title or possession to buyer, the seller can still enforce the contract but the price will be
abated to account for the damage. In other words, even though the risk of damage to the
property remained with the seller, the buyer will not be able to use the fact that the property was
damaged as a way to get out of the contract--the seller will still be able to enforce the contract
because the damage was immaterial.

Finally, a growing number of states have adopted the Uniform Vendor and Purchaser Risk Act.
Importantly, this Act negates the doctrine of equitable conversion as it relates to risk associated
with loss. In other words, the risk of loss is retained by the seller in jurisdictions that follow this
Act.
Posted by Sean Silverman at 7:12 PM 0 comments
Email ThisBlogThis!Share to TwitterShare to FacebookShare to Pinterest
Labels: Real Property
Thursday, December 1, 2016
Podcast
Very good advice on a podcast from the Bar Exam Toolbox regarding the MBE can be found @

http://barexamtoolbox.com/podcast-episode-52-bar-exam-tips-mastering-mbe/.

In general, these podcasts provide some really great advice and information.
Posted by Sean Silverman at 9:56 AM 0 comments
Email ThisBlogThis!Share to TwitterShare to FacebookShare to Pinterest
Labels: Miscellaneous
Tuesday, November 29, 2016
MBE Fast Fact: Privileges in Federal Court
I'm sometimes asked by students about how to apply rules regarding privileges in Evidence
questions on the MBE since the Federal Rules of Evidence have no specific privilege provisions.
Privilege in the federal courts is governed by common law principles as interpreted by the courts.
There are some privileges, however, that are currently recognized and should always be applied
on the MBE. These include the attorney-client privilege, spousal immunity, the privilege for
confidential marital communications, the psychotherapist(or social worker)-client privilege, and
the clergy-penitent privilege.

Further guidance is provided by rule 501 of the Federal Rules of Evidence. That rule provides
that in the trial of a civil proceeding in which state law provides the rule of decision, the rules of
privilege shall be determined in accordance with state law. In other words, in such an instance
(where state law governs the rule of decision), if a state law recognizes a given privilege, the
federal court must recognize the privilege as well.
Posted by Sean Silverman at 6:18 PM 0 comments
Email ThisBlogThis!Share to TwitterShare to FacebookShare to Pinterest
Labels: Evidence
Monday, November 14, 2016
The Right to Travel
In the most recent post, I wrote about the fundamental right to vote. Another fundamental right
that shows up in MBE questions (not all that frequently) is the fundamental right to travel. And
to reiterate, these fundamental rights are important both for substantive due process and for equal
protection. In either case, if a law impairs such a right, the law will only be valid if it passes strict
scrutiny.

An individual has a fundamental right to travel from state to state and to be treated equally after
migrating to the new state. It's important to note, though, that not every restriction on the right to
cross state lines is an impairment on the fundamental right to travel. An example to illustrate a
restriction that would not be an impairment on the fundamental right to travel would be a penalty
incurred by a parent who has abandoned his/her children by leaving the state and not paying
child support. In addition, often states will impose durational residency requirements for
receiving certain benefits within the state. It's questionable whether strict scrutiny is necessary
here (in other words, it's questionable if such a residency requirements is an impairment on the
fundamental right to travel). It's probably best to keep the following in mind: A one-year
residency to receive full welfare benefits is invalid, as is a one-year residency to receive state-
subsidized medical care. Likewise, a one-year residency to vote in state is invalid. On the other
hand, a 30-day residency requirement to vote in state is valid, as is a one-year residency to obtain
a divorce.

Note also that international travel is not a fundamental right. It's best instead to apply rational
basis scrutiny to ensure that any regulations impairing international travel are not arbitrary.
Posted by Sean Silverman at 6:06 PM 0 comments
Email ThisBlogThis!Share to TwitterShare to FacebookShare to Pinterest
Labels: Constitutional Law
Thursday, November 3, 2016
The Right to Vote
Fundamental rights are important both in terms of the Equal Protection Clause and in terms of
substantive due process. If these rights are denied to everyone the correct analysis is substantive
due process; if, on the other hand, they are denied to some individuals rather than everyone, then
the proper analysis is equal protection. In either case, though, whenever a fundamental right is at
issue, the proper analysis is strict scrutiny. In other words, any government action that is going
to affect a fundamental right must be necessary to serve a compelling government interest.

One such fundamental right is the right to vote, and there are a few key things to keep in mind
regarding this specific fundamental right:

Residency Requirements: Reasonable time periods for residency are valid. Though
reasonableness can be tough to evaluate, 30 days residency has been held to be valid for
purposes of voting.

Property Ownership: Conditioning the right to vote on ownership of property is almost always
invalid. One exception deals with special purpose elections (voting for officials who do not
exercise normal governmental authority but instead deal with matters of special interest in the
community).

Poll Taxes: Poll taxes are unconstitutional.

Primary Elections: States may require early registration to vote in primaries. States cannot,
however, prohibit political parties from opening their primary elections to anyone, whether or
not registered with the party.
One Person One Vote: This principle applies whenever any level of government decides to
select representatives to a governmental body by popular election from individual districts. An
exception, again, is for special purpose elections.

Gerrymandering: Race and other suspect classifications cannot be the predominant factor in
drawing the boundaries of voting districts unless the plan satisfies strict scrutiny.

Posted by Sean Silverman at 6:58 PM 0 comments


Email ThisBlogThis!Share to TwitterShare to FacebookShare to Pinterest
Labels: Constitutional Law
Tuesday, November 1, 2016
Helpful Link
Not too long ago the Federal Rules of Civil Procedure were updated and these updates are now
tested on the MBE. A good percentage of the testable content deals with discovery (especially
that of electronically stored information), but the Amendments span a wider area than just
discovery. A very helpful summary of the information can be found at the following link:

https://www.akingump.com/images/content/3/9/v2/39946/Amendments-to-the-Federal-Rules-of-
Civil-Procedure-Effective-Dec.pdf
Posted by Sean Silverman at 11:22 AM 0 comments
Email ThisBlogThis!Share to TwitterShare to FacebookShare to Pinterest
Labels: civil procedure
Saturday, October 22, 2016
The Top Five: Eminent Domain and Leasehold Estates
Eminent domain questions come up with some regularity, especially so because this topic can be
tested both in Property and in Constitutional Law. A subtopic that comes up requires an
understanding as to what happens to a leasehold if leased realty is taken by eminent domain. The
following are some points to remember:

(1): If leased property is taken by eminent domain, the leasehold and the reversion merge in the
taker (often the state) and so the leasehold is terminated.

(2): Once the leased property is taken by eminent domain there is no longer an obligation for the
lessee to pay rent.

(3): Both the lessor and lessee will be entitled to "just compensation" once the property has been
taken by eminent domain.

(4): The measure of "just compensation" provided is as follows: The lessor is entitled to receive
the value of the leased property (including the value of rent to be received) minus the value of
the leasehold interest that has already been conveyed to the lessee. The lessee is entitled to
receive the value of the leasehold.

(5): In a sense, the lessee benefits from the eminent domain as the obligation to pay rent ceases.
As such, the rent that the lessee otherwise would have been required to pay is generally deducted
from the value of the leasehold when determining the amount at which to compensate the lessee.

UCC Acceptance and Accommodation


A common question that tends to trip up students when we're working together on practice
questions deals with the concept of accommodation under the UCC. To understand these rules
it's important to first understand that there are multiple methods of accepting an offer when the
contract is for the sale of goods under the UCC. If the seller chooses to accept in a method other
than by shipment of the goods (for example, by directly notifying the buyer that the seller
accepts) then the rules regarding accommodation do not apply. And if the goods are later sent
and do not conform to the contract then that is a breach of contract and the buyer will have a
variety of options, one of which is to sue for breach.

If the seller chooses to ship the goods without first notifying the buyer that the seller has
accepted prior to shipment then that, too, might constitute an acceptance of the offer. But if the
seller seasonably notifies the buyer that the shipment is offered only as an accommodation to the
buyer, such a shipment will not be treated as an acceptance. Rather, the shipment will be treated
as a counter-offer.

The buyer will then have a few options if the seller has shipped nonconforming goods. The
buyer can keep the non-conforming goods in which case there is a contract for the goods as they
are at the price the seller has indicated. The buyer can also reject the shipment thereby
preventing a contract from forming. Importantly, however, the option is not available here for
the buyer to hold the seller in breach because without an acceptance of an offer by the seller
there can be no breach of contract.

Conflicts of Law: Real Property


Changes are coming to the MBE in February of 2017. One such change is stated by the NCBE
in its MBE subject-matter outline as "conflicts of law related to disputes involving real property."
This is rather vague and perhaps as we get closer to the exam we'll know more about the
specifics that need to be known for these questions but there are a few points that I'd think should
be known even as we wait to learn more.

The general rule governing conflicts of law in the subject of real property is that such property is
subject to the laws of the place where the property is situated (sometimes called the "law of the
situs"). The law of the place where the property is situated governs the disposition of real
property and this will be true whether the disposition is by descent, deed, or any other method.

In addition, contracts related to real property are often governed by the law of the place where
the property is situated. It's important to note, though, that if the real property is merely
incidental to the contract and if the contract is of a personal nature, then the traditional conflicts
of law rules regarding contracts will apply rather than the specific rules regarding real property.

More or on this change as well as the other changes to the content of the test in the months to
come......
Collateral Estoppel
Collateral estoppel applies to prevent a person in a civil case from re-litigating an issue. Once a
court has decided an issue of fact or law necessary to its judgment, that decision will preclude re-
litagation of that issue in a suit on a different cause of action involving a party to the first case.
A topic within this area that has been showing up in practice questions is that of mutuality.

Traditionally, collateral estoppel only applied where the party seeking to employ collateral
estoppel and the party against whom collateral estoppel was being employed were both parties to
the previous lawsuit. If this were true it was said that there was mutuality among the parties.
This requirement has been abandoned by most courts in most circumstances. For purposes of the
MBE, you should understand 4 ways in which collateral estoppel may be used:

Defensive mutual collateral estoppel: This is when collateral estoppel is used by the defendant
from the first suit against the plaintiff from the first suit regarding issues that were previously
litigated against the defendant in the first suit.

Defensive non-mutual collateral estoppel: This is when collateral estoppel is used by a new
defendant in a later suit regarding issues that were previously litigated against the plaintiff by a
former defendant in a previous suit.

Offensive mutual collateral estoppel: This is when collateral estoppel is used in a later suit by
the plaintiff from the first suit against the defendant from the first suit regarding issues that had
already been litigated in that first suit.

Offensive non-mutual collateral estoppel: This is when collateral estoppel is used by a new
plaintiff in a later suit against the defendant from a previous suit regarding issues that were
previously litigated against that defendant in the previous suit.

Posted by Sean Silverman at 7:49 AM 0 comments


Email ThisBlogThis!Share to TwitterShare to FacebookShare to Pinterest
Labels: civil procedure
Sunday, July 3, 2016
MBE Fast Fact: Trespass to Chattel
Intentional torts tend to be relatively straight-forward but there is one aspect of trespass that can
be counter-intuitive and so I thought it worth mentioning. Trespass to chattel is the intentional
interference with the plaintiff's chattel resulting in damages. The intent requirement can cause
some problems here. For this purpose, intent consists of a desire or knowledge that the chattel
will be involved without regard to whether the defendant actually knows that the chattel belongs
to the plaintiff, or that the plaintiff's rights are being violated.

In other words, if defendant takes a coat thinking that it belongs to the defendant but in fact
belongs to plaintiff, that mistake alone is not enough for the defendant to claim that the
defendant did not have the requisite intent to commit this tort. Defendant intended to take the
coat, and the intent to take the coat is enough. It's worth noting that these same types of questions
can come up in questions testing trespass to land. If someone walks onto the property of another
thinking that the land is his own, the requisite intent for trespass to land is still satisfied if there
was an intent to enter the land.
Posted by Sean Silverman at 5:40 PM 0 comments
Email ThisBlogThis!Share to TwitterShare to FacebookShare to Pinterest
Labels: Torts
Sunday, June 26, 2016
Past Recollection Recorded
One of the *many* hearsay exceptions that must be remembered is the one involving past
recollection recorded. But before understanding the particulars you'll need to be able to read the
fact pattern and understand from reading the facts that this exception is implicated. Look for a
situation where a witness states that there is insufficient recollection of an event to enable
testifying to the event fully and accurately.

And this is true even after the witness on the stand has had an opportunity to consult a writing
that might aid in remembering the event. The writing may then be read into evidence but because
it is an out of court statement offered for the truth of the matter asserted there is a hearsay
problem. And the way to get around this hearsay problem is to satisfy the elements of the
exception for this particular hearsay problem. The elements here amount to the foundation that
must be laid to offer this kind of statement into evidence, and the foundation is as follows:

(1): The witness at one time had personal knowledge of the facts in the writing.

(2): The writing was made by the witness or under his/her direction, or it was adopted by the
witness.

(3): The writing was timely made when the matter was fresh in the witness's mind.

(4): The writing is accurate.

(5): The witness now has insufficient recollection to testify fully and accurately.

Note that whenever a witness has used a writing to refresh his/her memory as stated above, an
adverse party is entitled to have the writing produced at trial. An adverse party is also entitled to
cross-exaimine the witness regarding the writing and to introduce portions relating to the
witness's testimony into evidence.
Posted by Sean Silverman at 11:28 AM 0 comments
Email ThisBlogThis!Share to TwitterShare to FacebookShare to Pinterest
Labels: Evidence
Thursday, June 9, 2016
The Top Five: Diversity Jurisdiction
There is so much material to cover regarding jurisdiction in Civil Procedure that it's easy to
overlook some of the finer points. Here are 5 potentially testable aspects of jurisdiction that may
not stand out when reading the outlines:

(1): Just as citizens of different states will satisfy the diversity requirement for diversity
jurisdiction so too will a citizen of the United States and an alien. Jurisdiction is denied,
however, if the alien has been admitted to the United States for permanent residence and is
domiciled in the same state as the US citizen.

(2): Diversity of citizenship must exist as of the time that the suit is instituted. In other words,
diversity need not exist at the time that the cause of action arose, and diversity is not defeated if
after commencement of the action a party later becomes a citizen of the same state as his/her
opponent.
(3): When determining the citizenship (for purposes of diversity) of a corporation, you must
consider both the states in which the corporation is domiciled (can be more than 1), and the state
in which the corporation has its principal place of business (can be only 1). A factor to strongly
consider when determining where the corporation has its principal place of business is to look to
where it has its headquarters. In contrast, for unincorporated associations (such as partnerships),
the association is considered for purposes of diversity jurisdiction to be a citizen of each state of
which any member is a citizen.

(4): For class action lawsuits, diversity is determined on the basis of the citizenship of the named
members of the class only.

(5): The legal representative of the estate of a decedent, an infant, or an incompetent is deemed
to have the same citizenship as the decedent, infant, or incompetent for purposes of determining
if diversity exists.
Posted by Sean Silverman at 3:52 PM 0 comments
Email ThisBlogThis!Share to TwitterShare to FacebookShare to Pinterest
Labels: Federal Civil Procedure
Saturday, June 4, 2016
State Taxation of the Federal Government
As you're preparing for the exam you're certain to come across some questions in Constitutional
Law testing the concept of state taxation of the federal government. The general rule to note is
that a state may not tax federal instrumentalities without the consent of Congress. The key point
here is that a state cannot tax the federal government or its agents in a manner that is going to
prevent the federal government from performing its federal functions. But then there are those
nondiscriminatory taxes (a very common issue that shows up is when the state requires federal
employees to pay state income taxes) that are perfectly valid provided that they do not
unreasonably burden the federal government.

But the reason for this post is that a less common rationale for providing the federal government
with a basis for avoiding state taxation comes up in the questions and it should be considered as a
correct answer choice.

The Necessary and Proper Clause in the Constitution gives Congress the power to make all laws
necessary and proper for executing any power granted to the federal government. Though it
seems like a bit of a stretch, the right to make all laws necessary and proper for executing any
power granted to the federal government has been held to include the power of the federal
government to grant itself immunity from state taxation.

Posted by Sean Silverman at 7:13 AM 0 comments


Email ThisBlogThis!Share to TwitterShare to FacebookShare to Pinterest
Labels: Constitutional Law
Wednesday, May 25, 2016
MBE Fast Fact: Negative Evidence
A term I encounter often in the practice questions is "negative evidence." It's likely to show up in
an evidence question involving products liability and the following example illustrates how the
concept comes up:

Assume that x is in a car accident and asserts a personal injury claim against the car
manufacturer claiming that a defect in the car caused the accident. At trial, the defendant calls a
witness to testify that although he is in charge of hearing all complaints for the car company
regarding product safety, there has never been a complaint similar to the one that plaintiff is
claiming caused the accident, and many thousands of cars identical to plaintiff's have been sold.
Is the absence of similar complaints admissible evidence to prove that the plaintiff's complaint in
this case is unwarranted? In other words, is this "negative evidence" admissible?

If a proper foundation is laid, evidence that a particular product has been used many times
without accident is admissible as circumstantial evidence that its condition is not dangerous.
You'll want to ensure that the conditions under which the product was previously used are
identical to those that exist at the time of the accident that is the subject of the current case and
that the witness would have heard if there had been any previous accidents.

Posted by Sean Silverman at 8:26 PM 0 comments


Email ThisBlogThis!Share to TwitterShare to FacebookShare to Pinterest
Labels: Evidence
Sunday, May 15, 2016
MBE Fast Fact: "Then Existing State of Mind" Exception
One of the many exceptions to the rule against hearsay is the "then existing state of mind." How
this generally plays out in questions is that statements of the declarant's intentions are deemed to
fit within this exception. In addition, these statements may be relevant to establish that the
declarant acted in a way that was consistent with the stated intentions.

It's important to keep in mind, however, that these statements are relevant only to establish the
conduct of the declarant him/herself; they are not relevant to establish the conduct of others. So,
for example, assume that two defendants are being tried together and a witness testifies on behalf
of both defendants. If the witness testifies to a conversation he had with defendant 1 and in doing
so testifies to a statement made by defendant 1 that both defendant 1 and defendant 2 intended to
be in another state the day that the crime was committed, the statement will be admissible insofar
as it helps to establish that defendant 1 was in the state that he claimed to have intended to be in
on the day of the crime, but the statement will not be admissible to infer the same about
defendant 2.

Illusory Promises
What is an illusory promise? In Contract Law consideration must exist on both sides of a
contract if the parties are to be bound to perform their promises. And so if two people promise to
perform for each other but only one party is actually bound to perform, then there is an illusory
promise involved which will prevent the formation of the contract. So, for example, assume x
tells y that x will purchase a product from y if x decides to first apply for a loan from a bank.
Because x is not obligated to apply for the loan, x has taken on no additional burden in making
that promise and as such the promise is illusory.

As important as it is to recognize illusory promises, it's also important to be able to spot promises
that may seem illusory but are not. The following is not an exhaustive list but contains examples
that are not illusory and therefore satisfy the mutuality requirement necessary to form a valid
contract:
--Requirement and output contracts under the UCC. Claiming that you will purchase all the
widgets that you require (not illusory) is different than claiming that you will purchase all the
widgets that you want (illusory).

--Conditional promises unless the condition is entirely within the promisor's control.

--Contracts where a party has the right to cancel if that right is somehow restricted (for example
if it is stated that cancellation requires notice).

--Voidable promises (such as those made by an infant).


Posted by Sean Silverman at 7:45 PM 0 comments
Email ThisBlogThis!Share to TwitterShare to FacebookShare to Pinterest
Labels: Contracts/Sales
Tuesday, April 19, 2016
5 Things to Remember About Adverse Possession
Adverse possession is one of those topics that seems to pop up quite a lot on the MBE. There are
many different angles to test, and if you work through enough practice questions, you'll see them
all. Here are five points that I'd remember on this topic as you prepare for the exam:

(1): Assume that co-tenants own property. Possession by one co-tenant is usually not adverse to
his/her co tenants because each co-tenant has the right to possess all of the property. And
because the requirement for hostile possession will therefore not be satisfied, a co-tenant will
generally not gain ownership rights to another co-tenant's property through adverse possession.

(2): Assume now that y has ownership in real property but the ownership rights amount to a
future interest in the property. If x attempts to adversely possess that property, the statute of
limitations will not begin to run against y until y's future interest becomes a present possessory
estate.

(3): What if an adverse possessor uses the land in violation of a restrictive covenant in the
owner's deed. In such an instance, the adverse possessor takes the property free of the restriction
(kind of counter-intuitive so worth noting).

(4): Title to government-owned land and land registered under a Torrens system cannot be
acquired by adverse possession.

(5): The statute of limitations triggered by adverse possession will not begin to run if the true
owner of the property was under some disability to sue when the cause of action first accrued
(when the adverse possessor first took possession). The only disability that is relevant here is the
disability of the owner of the property, and that disability must exist at the time that the cause of
action arose.
Posted by Sean Silverman at 4:47 PM 0 comments
Email ThisBlogThis!Share to TwitterShare to FacebookShare to Pinterest
Labels: Real Property
Wednesday, March 16, 2016
MBE Fast Fact: Foreclosure and Priorities
Priority of mortgages is tested in a variety of ways on the MBE. An important point to keep in
mind in order to keep things straight with this difficult topic is that when a mortgage is
foreclosed the buyer at the sale takes title as it existed when the mortgage that is now being
foreclosed was placed on the property. What this means, in effect, is that if there is more than
one mortgage on the property then foreclosure of one mortgage will only terminate those
interests that are junior to the mortgage being foreclosed. In other words, senior interests will not
be affected by the foreclosure of a junior interest.

But the complications don't end there. Although the foreclosure of a senior mortgage can
terminate mortgages that are junior to that senior mortgage, those junior mortgages are still
necessary parties to the foreclosure action. Failure to include those necessary parties will result
in the preservation of that junior party's interest despite foreclosure and sale. So, for example, if a
senior mortgagee fails to name a junior mortgagee to a foreclosure action by the senior
mortgagee, the buyer at the foreclosure and sale will take the property subject to the interests of
the junior mortgagee.
Posted by Sean Silverman at 8:23 PM 0 comments
Email ThisBlogThis!Share to TwitterShare to FacebookShare to Pinterest
Wednesday, March 9, 2016
MBE Fast Fact: Real Property Warranties
An interesting issue that pops up on the MBE deals with the differences between warranties
allowable under the UCC and warranties for the sale of real property. Under the UCC for the sale
of goods there are both express and implied warranties to consider carefully but for the sale of
real property no implied warranties of quality or fitness for a particular purpose are generally
allowable.

But there is an important exception to keep in mind. This exception allowing for implied
warranties only applies for the sale of real property if the seller of the real property is also the
builder of the property. There is an implied warranty that the new property is designed and
constructed in a reasonably workmanlike manner and suitable for human habitation. In general,
for purposes of this issue, it's enough to understand that a buyer will have no claim based solely
on implied warranties against a seller of real property unless the seller was also the builder of the
property, in which case the buyer can make a claim that the house was not constructed in a
reasonably workmanlike manner.
Class Action Lawsuits
When working with students to prepare for Civil Procedure on the MBE, a topic that comes up
frequently is class action lawsuits. This post will summarize some aspects that I believe are
important to know for purposes of the MBE:

When is a class action proper?

There are some requirements here. First, the class of plaintiffs must be so numerous that joinder
of all members is impracticable. In addition, there must be questions of law or fact common to
the class, named parties' interests must be typical of the class, and named parties must adequately
represent the interests of the absent members of the class. In addition, one of the following must
be true: separate actions would create a risk of inconsistent results or impair the interests of
unnamed parties; the defendant has acted or refused to act on grounds applicable to the class, and
injunctive or declaratory relief is appropriate for the class as a whole; or common questions of
law or fact predominate over individual issues and a class action is superior to alternate methods
of adjudication.

When should the court decide whether to certify the class?


This should be decided at an early practicable time, but a court can determine that a class action
is not appropriate at any time. When the court certifies a class, the court must define the class,
issues, and defenses. Further, the court must appoint class counsel who must fairly and
adequately represent the class.

What effect does a class action judgement have?

All members of a class will be bound by the judgement rendered in a class action except those in
a common question class action who notify the court that they do not wish to be bound. For this
reason, notice to all members of a class is required in a common question class action lawsuit.

How is jurisdiction analyzed in a class action lawsuit?

In class actions founded on diversity only the citizenship of the named representatives of the
class is taken into account in establishing whether the diversity requirement is satisfied. One
class representative's claim generally must exceed $75,000. Class members with claims not
exceeding this amount may invoke supplemental jurisdiction provided that doing so does not
destroy diversity.

Must the court approve dismissal of a class action?

Yes, the court must approve dismissal or settlement. In a common question class action lawsuit,
the court may provide the parties with a second opportunity to opt out.

Larceny vs Embezzlement
Some distinctions come up time and again on the MBE. One such distinction is that between two
theft crimes: larceny and embezzlement. You'll see quite a few of these questions as you work
through Criminal Law, and here I'll outline the important differences to note:

Larceny is the taking and carrying away of tangible personal property of another by trespass with
the intent to permanently deprive the other of his/her property.

Determine first whether the person charged with larceny had possession of the property at the
time it is claimed that the person took and carried away the property. Because if the person had
legal possession of the property at that time, then larceny is not the proper charge. The reason for
this stems from the definition above; larceny requires taking and carrying away the property of
another and if a person is deemed to have possession of the property that he takes, then it can't be
said that the person is taking the property of another, and as such not all essential elements of the
crime have been satisfied.

Further, larceny requires that at the time of the taking and carrying away of the property of
another, the person taking the property intends to permanently deprive another of the property.
For purposes of the MBE you should also have an understanding of "continuing trespass larceny"
in which the intent to permanently deprive is not formed until after the property has been taken.
This too is deemed larceny, provided that all other elements have been satisfied.

Embezzlement is the fraudulent conversion of personal property of another by a person in lawful


possession of that property. Unlike with larceny, to be charged with embezzlement, one needs to
have been in lawful possession of property prior to misappropriating that property.
In determining if a person is in possession of property for purposes of analyzing embezzlement
consider the following definition for guidance: A person has possession of property when he has
sufficient control over it to use it in a reasonably unrestricted manner (in which case consider
embezzlement). A person lacks possession if he has physical control over it, but his right to use it
is substantially restricted by the person in lawful possession of the property (in which case
consider larceny).

This distinction outlined here is a tough one as the line between the two crimes is rather thin. But
like so many legal concepts on the MBE, the best way to understand the type of distinction that
might be tested is to work through many practice questions. The MBE should be thought of as a
game, and like many games the best way to improve is through dedicated practice and training.
The outlines will start you on your way, but practice will take you to where you need to be.

MBE Fast Fact: Supplemental Jurisdiction


There are quite a few avenues the MBE can take when testing supplemental jurisdiction in Civil
Procedure. Supplemental jurisdiction gives authority to the federal courts, under certain
circumstances, to hear additional claims substantially related (or of the same "common nucleus
of operative fact") to the original claim even though the court would lack the subject-matter
jurisdiction to hear the additional claims independently.

But one point to keep in mind is that a court may not exercise supplemental jurisdiction over
claims by plaintiffs against parties added as third parties if doing so would destroy diversity of
citizenship among the parties. Thus, for a court to have subject-matter jurisdiction over a claim
by a plaintiff against a party added by plaintiff as a third party, the prerequisites for traditional
subject-matter jurisdiction must be satisfied.
Posted by Sean Silverman at 8:55 PM 0 comments
Email ThisBlogThis!Share to TwitterShare to FacebookShare to Pinterest
Labels: Federal Civil Procedure
Saturday, November 21, 2015
Malice Aforethought
Oftentimes when the MBE is testing the crime of murder, a statute is provided in the fact pattern
and you'll be required to interpret that statute and draw a conclusion based upon your
interpretation. You'll see terms like "first-degree murder," "second-degree murder," etc. But
that's not the default rule on the MBE; rather, the default rule on the MBE is the common law,
and common-law murder does not incorporate distinctions such as first-degree.

Murder under the common law is defined as the killing of another human being with malice
aforethought. As with all legal rules, you'll want to parse the rule to ensure that all elements of
the rule have been satisfied before drawing your conclusion. The first few elements are easy
enough; you'll want to ensure that the victim has died and that the victim was a human being.
The question will likely be testing the element of malice aforethought, so it's essential to
understand what constitutes malice aforethought.

The 4 mindsets that will satisfy the malice aforethought requirement are:

(1): An intent to kill: If x intends to kill y (in other words, x actively desires the prescribed
criminal consequences to follow his act), then murder is applicable.
(2): An intent to cause serious bodily injury: Note here that a person can be charged with murder
under the common law even if it is not the desire of that person to kill another. Provided that x
desires to cause y serious bodily injury, then if y dies as a result of the injury inflicted by x, x can
be charged with murder.

(3): Reckless indifference for the life of another: This is sometimes referred to on the MBE as
depraved heart murder. Even if there is no intent to kill, and there is no intent to cause serious
bodily injury to another, one can be charged with murder if he acts recklessly, and the reckless
act causes the death of another. An important distinction to note here is between recklessness and
negligence. Causing the death of another through one's own negligence is not sufficient for a
charge of murder (the more appropriate crime is involuntary manslaughter), but causing the
death of another through one's own recklessness is sufficient. The key difference here is that a
person acts recklessly if he is aware of a substantial risk that a certain result will occur as a result
of his actions whereas he acts negligently if he should have been aware (even if not actually
aware) of a substantial and unjustifiable risk that a certain consequence would result from his
actions.

(4): Felony Murder: You'll want to look for a situation in which a death occurs during the
commission of an inherently dangerous felony. If a death occurs during the commission of an
inherently dangerous felony, and there is no intent to kill, no intent to cause serious bodily
injury, and no reckless disregard for the life of another, then consider felony murder, as it will
provide a basis for a charge of murder even if the highest degree of fault on the part of the person
committing the felony is negligence.

Posted by Sean Silverman at 9:45 AM 0 comments


Email ThisBlogThis!Share to TwitterShare to FacebookShare to Pinterest
Labels: Criminal Law
Thursday, November 12, 2015
MBE Fast Fact: First Amendment: Overbroad vs. Void for Vagueness
I've stated on this blog in the past that the First Amendment is an important topic to know well
when studying Constitutional Law. The terms "overbroad" and "void for vagueness" have
similarities that make them very testable as small differences can materially distinguish one
answer on the exam from another.

A statute is overbroad if the statute proscribes activities that may be constitutionally forbidden
but in doing so also sweeps within its coverage speech that is protected by the First Amendment.
A law limiting First Amendment rights must be crafted to limit only those rights not protected by
the Constitution. If not crafted as such then it might well be overbroad and therefore
unconstitutional.

In contrast, a statute may be void for vagueness if the conduct forbidden is so unclearly defined
that people would have to guess at its meaning and would differ as to its application. Oftentimes
if a question is testing the concept of vagueness a specific term within a law will be provided, but
that term will not be defined. By not defining that term its meaning is left ambiguous and
ambiguity leaves open the possibility that people will define the term differently thereby limiting
their ability to correctly apply the law. If a reader of a statute is left to guess the meaning of a
material term within a statute, consider whether that statute is void for vagueness.
Posted by Sean Silverman at 9:51 PM 0 comments
Email ThisBlogThis!Share to TwitterShare to FacebookShare to Pinterest
Labels: Constitutional Law
Wednesday, November 4, 2015
Summary Judgment vs Motion for Judgment as a Matter of Law
The distinction between a summary judgment and a judgment as a matter of law (formerly
known as a directed verdict) was showing up on the MBE even before Civil Procedure became a
testable subject. Now that Civil Procedure is tested, it's even more likely to come up in one way
or another.

A motion for summary judgment is a pretrial procedure to determine whether genuine issues of
material fact exist within the case. A decision in favor of the moving party will resolve a lawsuit
before there is a trial. Summary judgment is awarded if the undisputed facts and the law make it
clear that it would be impossible for the opposing party (the non-moving party) to prevail if the
matter were to proceed to trial.

A motion for judgment as a matter of law, on the other hand, is requested at the end of a
plaintiff's case or after all the evidence has been completed. The moving party when making a
motion for judgment as a matter of law is arguing that the evidence clearly reveals that s/he must
prevail and that there is therefore no reason to send the case to the jury. In deciding whether to
grant a motion for judgment as a matter of law, all the evidence is viewed in the light most
favorable to the non-moving party. A motion for judgment as a matter of law if granted is usually
granted because the judge concludes that the non-moving party has failed to offer the minimum
amount of evidence to prove the case. In other words, no reasonable jury could decide in favor of
the non-moving party.

Lien Theory/Title Theory/Joint Tenancies


Questions on the MBE may require understanding the distinction between a lien theory state and
a title theory state. A typical example has a joint tenant mortgaging his interest in the property
held in joint tenancy. According to the lien theory, the mortgagee is considered the holder of a
security interest only and the mortgagor is deemed the owner of the land until foreclosure. In
contrast, according to the title theory, legal title is in the mortgagee the moment that the property
is mortgaged by the joint tenant.

As an example, let's assume that x and y are joint tenants with rights of survivorship. x decides to
take out a loan from a bank, and in exchange for the loan, x gives the bank a mortgage on the
property that he owns with y as joint tenants. Determining the interest that the bank receives will
depend entirely on the rules above; namely, in a lien theory state the bank merely has a security
interest and x is still deemed the joint owner of the property with y until foreclosure. In a title
theory state, however, the mortgage severs the joint tenancy so that the the bank receives legal
title to the property and becomes a tenant in common with y at the time of the mortgage.

Let's also assume that after the mortgage but before x defaults on the loan x dies. In a lien theory
state, x and y remained joint tenants and as per the rights of survivorship, the moment that x
died, y became the sole owner of the property. As such, there would be no property for the bank
to foreclose upon. In a title theory state, however, the bank is a tenant in common with y so the
death of x has no effect on the interest of the bank.
Posted by Sean Silverman at 8:01 PM 0 comments
Email ThisBlogThis!Share to TwitterShare to FacebookShare to Pinterest
Labels: Real Property
Monday, August 31, 2015
MBE Fast Fact: Future Interests
In the many years I've taught the bar exam, it would be tough for me to think of a topic that
students dislike more than future interests. Certainly, there is more to know about future interests
than that contained in this post, but the following are three types of future interest retained by the
grantor that all students should understand well when preparing for the exam:

Reversions: A reversion is a future retained by the grantor who conveys less than he owns. For
example, let's say that X, the grantor, owns property in fee simple, and he conveys that property
to Y, the grantee, for life. X has only conveyed a life estate (a present possessory estate) even
though X owned the property for an infinite duration. When Y dies, the property will not die
along with him. Rather, the property will revert back to X.

Rights of Entry: Similar to a reversion, a right of entry is a future interest reserved by the
grantor. However, the right of entry is reserved when the grantor grants to another a fee simple
subject to condition subsequent (a present possessory estate). A fee simple subject to condition
subsequent has the potential to last indefinitely, so it's not certain, as it is with a reversion, that
the grantor will ever regain an interest in the property. For example, let's say that X grants
property to Y but if the property is not used for farming purposes then X may reenter and retake
the property. The future interest in X is not certain to vest; it will only vest if Y fails to use the
property for farming purposes. At that point, X will use his right of entry to reenter and retake
the property. In other words, a right of entry is always contingent.

Possibilities of Reverter: A possibility of reverter, like in the two previous examples, is a future
interest reserved by the grantor. Unlike with the right of entry, the grantor here need not take any
affirmative action by reentering the property in order to retake the property. For example, X
grants property to Y for as long as Y uses the property for farming purposes. At that moment, Y
has a fee simple determinable (a present possessory estate). Y is subject to the same obligation as
in our previous example; namely, to use the property for farming purposes. But if Y fails to do
so, X has no obligation to reenter the property; rather, the possibility of reverter will kick in, and
the property will automatically revert back to X.
Posted by Sean Silverman at 7:11 PM 0 comments
Email ThisBlogThis!Share to TwitterShare to FacebookShare to Pinterest
Labels: Real Property
Monday, August 24, 2015
The Commerce Clause
Questions testing the Commerce Clause on the MBE are quite common. The Commerce Clause
provides an avenue for Congress to pass regulations but it's important not only to understand the
power granted to Congress but the limitations as to that power as well. Congress can pass laws
that regulate interstate commerce provided the laws regulate the channels of interstate
commerce, or the instrumentalities of interstate commerce. In addition the Commerce Clause
stretches even further by allowing Congress the power to pass laws that regulate activities that
have a substantial effect on interstate commerce.

When the activity that is the subject of the regulation is intrastate (does not cross state lines)
rather than interstate (crosses state lines), any regulation by Congress attempting to regulate such
activity will be upheld only if the activity is deemed economic or commercial, and there is a
rational basis for concluding that the activity will have a substantial effect on interstate
commerce. Stated otherwise, if the activity is noncommercial and intrastate, the Commerce
Clause is unlikely to provide an appropriate avenue for Congress to regulate that activity.

And even if Congress has not passed a law regulating interstate commerce, a state law that
discriminates or unduly burdens interstate commerce will likely be held in violation of the
Commerce Clause. This is often called the Dormant Commerce Clause, and it limits the state's
ability to pass laws discriminating against interstate commerce, even if Congress has passed no
conflicting law, unless the law furthers an important non-economic state interest and there are no
reasonable nondiscriminatory alternatives, the state is acting as a market participant, or the law
involves government action regarding the performance of a traditional state-government
function. Note that if a non-conflicting state law does not discriminate against interstate
commerce, it may still be a violation of the Commerce Clause (as per the Dormant Commerce
Clause) if the law burdens interstate commerce, and the state's interest in passing the law does
not outweigh that burden.
Posted by Sean Silverman at 7:49 PM 0 comments
Email ThisBlogThis!Share to TwitterShare to FacebookShare to Pinterest
Labels: Constitutional Law
Wednesday, August 12, 2015
MBE Fast Fact: Misrepresentation
In Contracts there are two types of misrepresentation to keep in mind, and the fine distinctions
between them are exactly the type that the writers of the MBE are likely to test:

Fraudulent Misrepresentation: In this type of misrepresentation one party to the contract induces
another to enter into the contract by asserting information that s/he knows is untrue. Here, if a
contract is formed, the contract is voidable by the innocent party provided that the innocent party
justifiably relied on the fraudulent misrepresentation. This is sometimes referred to as fraud in
the inducement.

Nonfraudulent Misrepresentation: There can be a misrepresentation even if that


misrepresentation is not fraudulent. In other words, a false statement is made by a party to the
contract but that party does not know that the statement is false when the statement is made and
there is not necessarily an intent to defraud. Here, again, the contract is voidable by the innocent
party but only if the innocent party justifiably relied on the misrepresentation, and the
misrepresentation was material. A misrepresentation is material if it would induce a reasonable
person to enter into a contract, or if the person who made the misrepresentation knew that due to
special circumstances the statement would be likely to induce the innocent party, even if it would
not induce a reasonable person.

MBE Fast Fact: Notice of Assignments


Let's take a common situation where x enters into a contract with y whereby x will be providing
a product to y, and y will be paying x for that product. Prior to payment x assigns his right to
payment to z, and further provides y with notice of the assignment to z. Y submits payment to x
rather than to z, and then claims its obligations under the contract have been fulfilled. Z, who
never receives payment because payment went to x, sues y for payment. What should result?

The key to understanding such an issue is to understand that (provided the right was assignable,
and it almost always is), the assignee (here, z) becomes the real party in interest and is therefore
entitled to performance under the contract. Further, once the obligor (here, y) has notice of the
assignment he is bound to render performance to the assignee. If after receiving adequate notice
the obligor pays the assignor (here, x) instead of the assignee, then the obligor will be required to
pay the assignee even though payment has already been submitted to the assignor.

All that said, here y will be required to submit payment to z as the right to payment was assigned
to z, and y was on notice of the assignment.
Posted by Sean Silverman at 6:26 PM 0 comments
Email ThisBlogThis!Share to TwitterShare to FacebookShare to Pinterest
Labels: Contracts/Sales
Sunday, April 19, 2015
MBE Fast Fact: The Mailbox Rule
The mailbox rule (under the common law of contracts) has the potential to be very confusing.
But if you note the following, then questions testing this concept shouldn't be too bad:

The rule is meant to set forth a determination as to whether acceptance of an offer is effective.
The simplest situation is where the acceptance is by mail, and only an acceptance is mailed. In
such a case, the acceptance becomes effective at the moment of dispatch, unless the offeror has
expressly stated that acceptance will become effective only once it is received. (And on the
MBE, the offeror does sometimes state that!) As is often the case in contract law, the offeror has
the final word, so if the offeror states that the mailbox rule does not apply (by claiming
acceptance is only valid upon receipt,) then don't apply the rule for that particular question. Note,
also, that the mailbox rule does not apply in the case of an option contract (when consideration
has been provided to hold the offer open). In an option contract, acceptance is effective upon
receipt by the offeror rather than dispatch by the offeree.

A second scenario involves the offeree dispatching a rejection in the mail, and then changing his
mind and dispatching an acceptance before the rejection is received by the offeror. Here, the
rejection will be effective, unless the acceptance is received first.

Lastly is the situation in which the offeree sends an acceptance and then sends a rejection.
Because acceptance was first dispatched you should merely apply the general mailbox rule that
acceptance is effective upon dispatch. But there is one twist: If the rejection arrives prior to the
acceptance (unlikely), then the rejection will be effective.
Posted by Sean Silverman at 4:17 PM 0 comments
Email ThisBlogThis!Share to TwitterShare to FacebookShare to Pinterest
Labels: Contracts/Sales
Monday, April 13, 2015
MBE Fast Fact: Buyer's Acceptance Under the UCC
When dealing with a question implicating the UCC rather than the common law, it's important to
remember that the doctrine of substantial performance does not apply, and instead the Perfect
Tender Rule applies. This means that if the goods delivered by the seller fail to conform to the
contract in any way, the buyer may reject all the goods delivered, reject some of the goods and
accept some of the goods, or accept all of the goods.

As such, it is equally important to understand exactly what constitutes acceptance in such


circumstances because the moment of acceptance is also the moment that the buyer loses the
ability to reject the goods. A buyer accepts under the UCC under 3 circumstances:

--After a reasonable opportunity to inspect the goods, the buyer indicates to the seller that the
goods conform to the requirements of the contract or that the buyer will keep the goods even
though they do not conform as required.

--The buyer fails to reject the goods within a reasonable time after tender of delivery or fails to
seasonably notify the seller of the rejection.

--The buyer acts in a way that indicates that the buyer is the owner of the goods, or, stated
otherwise, acts inconsistent with the seller's ownership of the goods.

One further note here: If none of the 3 above apply, and buyer rightfully rejects the goods, the
buyer still has an obligation to hold the goods with reasonable care at the seller's disposition and
to follow any reasonable instructions given by the seller as to the rejected goods. If the seller
fails to give instructions, the buyer may reship the goods to the seller, store them for the seller's
account, or resell them for the seller's account and retain any expenses incurred in following the
instructions of the seller.
Posted by Sean Silverman at 9:35 PM 0 comments
Email ThisBlogThis!Share to TwitterShare to FacebookShare to Pinterest
Labels: Contracts/Sales
Sunday, April 5, 2015
Buyer's Insolvency
Assume that the seller has entered into a contract with the buyer, but after entering into that
contract the seller learns that the buyer is insolvent and will not be able to pay for the goods. A
question becomes whether seller at that point must deliver the goods to the buyer and then later
sue when buyer fails to pay, or instead whether seller can avoid delivering the goods entirely.
UCC 2-702 addresses this question.

Where the seller discovers the buyer to be insolvent he may refuse delivery, or if necessary, stop
delivery of goods that have already been delivered to the carrier service. (For the specifics on
stopping delivery see rule 2-705)

Sometimes, however, the seller will not discover the insolvency until the buyer has already
received the goods. In such instances, the seller may reclaim the goods upon demand made
within 10 days after the receipt. Note that if misrepresentation of solvency has been made to the
seller in writing within three months before delivery, then the ten day limitation does not apply.

Also very important for purposes of the MBE is that the seller's right to reclaim goods is subject
to the rights of a buyer in ordinary course or other good faith purchaser for value. In other words,
if the buyer has sold the goods to a bona fide purchaser ("BFP"), then seller will be required to
seek other remedies from buyer, as the ability to reclaim the goods will have been lost upon that
sale.
Posted by Sean Silverman at 5:36 PM 0 comments
Email ThisBlogThis!Share to TwitterShare to FacebookShare to Pinterest
Labels: Contracts/Sales
Sunday, March 29, 2015
Trusts
Trusts is a subject tested on the state essay portion of many bar exams. It's not thought of as an
MBE subject, but it is testable in the area of Real Property. Fortunately, for purposes of the
MBE, the material is very basic. It's enough to know the following:
A trust is a relationship whereby a settlor conveys property to a trustee, and the trustee holds
legal title to that property. The beneficiaries hold equitable title with equitable rights attached. At
the time that the settlor conveyed the property, he/she must have owned the property and had the
intent to create a trust.

A trust can be created in one of two ways: by will or by an inter vivos transfer of the property. It
should be noted that the settlor may choose to act as trustee in which case the trust will be
created if the settlor declares that he is holding the property in trust. As is the case with any
transfer of real property, if the property transferred is real property, then the trust agreement must
be in writing to satisfy the statute of frauds. If a settlor creates a trust during his lifetime, and
then transfers property in his will to that lifetime trust, it is known as a pour over trust.

Occasionally questions will test charitable trusts. A charitable trust must have a charitable
purpose, and must have indefinite beneficiaries (as opposed to a private trust in which the
beneficiaries must be definite). Further, it may be of indefinite duration (the Rule Against
Perpetuities does not apply). Occasionally, a trust will be set up for a charity and at some point
the purpose of the trust will become impracticable or impossible. In such instances, the court will
apply the cy pres doctrine which allows the court to select an alternative charity. Generally,
charitable trust are enforced by the attorney general of the state.
Posted by Sean Silverman at 7:22 PM 0 comments
Email ThisBlogThis!Share to TwitterShare to FacebookShare to Pinterest
Labels: Real Property
Thursday, March 19, 2015
MBE Fast Fact: Recapture of Chattels
Within the area of intentional torts, the topic of recapture of chattels tends to cause some
difficulty. And with good reason: it's somewhat of a gray area of law with different outlines
stating different types of analysis. Here's what should be known for purposes of the MBE:

It's important when analyzing recapture of chattels to determine exactly how the chattels came
into the possession of the person from whom the chattel is now being recaptured. Assume that X
is the owner of the chattel, and that Y is now in possession of X's chattel.

If Y's possession of the chattel began lawfully, then X may only use peaceful means to recapture
the chattel. To the contrary, X may only use force to recapture the chattel from Y if X is in hot
pursuit of Y, and if Y has obtained the chattel unlawfully.

How might this play out in a question. Assume that Y has come into possession of X's chattel
lawfully. For example, assume X has given a piece of property to Y, and that now Y refuses to
return it. If X attempts to recapture the property and uses force in doing so, Y may have a claim
against X for battery, even though X was merely attempting to recapture property that belonged
to X. This is because the use of force on the part of X would not be allowed in such a situation,
so that if X claimed recapture of chattels as a defense to a claim of battery, the defense would
fail.
Posted by Sean Silverman at 5:15 PM 0 comments
Email ThisBlogThis!Share to TwitterShare to FacebookShare to Pinterest
Labels: Torts
Monday, March 9, 2015
MBE Fast Fact: Interference with Business Relations
A tort that shows up with some regularity on the MBE is the tort of interference with business
relations. As is the case with all intentional torts, the key to answering questions correctly that
are testing this tort is to know the elements of the tort. They are as follows:

--A valid contractual relationship between plaintiff and a third party, or a valid business
expectancy of plaintiff with the third part

--Defendant's knowledge of that relationship or expectancy

--Intentional interference of the relationship or expectancy by defendant that induces a breach of


the relationship or expectancy

--Damages to plaintiff.

But the analysis does not end there. Defendant's conduct may be privileged where it is a proper
attempt to obtain business for defendant. Defendant's conduct is far more likely to be deemed
privileged in this respect if the conduct is one that merely interferes with plaintiff's expectancy of
business and not an already-existing business relationship. In addition, the analysis should run in
favor of finding a privilege if defendant used commercially acceptable means of persuasion
rather than illegal or threatening tactics, or if defendant is a competitor of plaintiff seeking the
same prospective clients. It's also important to note that if the third party has contacted defendant
seeking business advice, and the defendant provides such advice, providing such advice is
unlikely to be deemed actionable by plaintiff as an interference of plaintiff's relationship with the
third party, even if the the advice ultimately leads to the third party forming a business
relationship with defendant rather than plaintiff.
Civil vs. Criminal (Assault and Battery)
The fact that both assault and battery show up both in civil law (Torts) as well as in Criminal
Law makes these topics tricky if the distinctions are not known well. Based upon what I've seen
in practice questions, the following should be known about each:

Assault:

In Torts, assault is an intentional tort that requires an act by the defendant creating a reasonable
apprehension in plaintiff of an immediate harmful or offensive contact to plaintiff's person. In
addition, causation is required, but damages are not required.

In Criminal Law, there are two ways to commit the crime of assault. One way is nearly identical
to the definition set forth above in Torts. A criminal assault is an intentional creation of a
reasonable apprehension in the mind of the victim of imminent bodily harm. But there is another
definition of criminal assault to note carefully. Assault is also an attempt to commit a battery.
This is a specific-intent crime so be sure to apply all the rules you've learned regarding specific-
intent crimes to criminal-assault. In addition, note that a person can be guilty of criminal assault
even if the plaintiff was not put in apprehension of harm because all that is required is that
defendant intended to commit a battery upon plaintiff.

Battery:

In Torts, battery is harmful or offensive conduct to plaintiff's person with both intent and
causation. Once again, damages are not a required element.
In Criminal Law, battery is an unlawful application of force to the person of another resulting in
either bodily injury or an offensive touching. Importantly, unlike in Torts, criminal battery need
not be intentional. Unlike criminal assault, criminal battery is a general intent crime.
Posted by Sean Silverman at 4:58 PM 0 comments
Email ThisBlogThis!Share to TwitterShare to FacebookShare to Pinterest
Labels: Criminal Law
Sunday, June 7, 2015
The Top Five: Tenants in Common
There are a lot of angles the MBE can take when testing tenants in common. This post
summarizes the areas I see them go most often in the questions:

The distinguishing factor between a tenancy in common and a joint tenancy is that a tenancy in
common has no right of survivorship. Tenants can hold different interests in the property, (for
example tenant 1 owns 75% while tenant 2 owns 25%), but all tenants have a right to possession
of the entire 100%. Interests are entirely alienable, devisable, and inheritable. The following are
five specific rules to know regarding co-tenancies:

(1): Though each tenant has the right to possess all of the property, no tenant has the right to
exclusive possession of any part of the property. Therefore, if one co-tenant claims exclusive
control (and only if one co-tenant claims exclusive control), then the other co-tenant can claim
s/he has been ousted and bring a possessory action against the tenant claiming exclusive control.

(2): A co-tenant in possession has the right to retain profits from his/her own use of the property.
However, net rents from third parties and net profits gained from exploitation of the land must be
shared with co-tenants, even those who are not possessing the land at the time those rents or
profits are generated.

(3): A co-tenant may encumber his/her interest (for example, by a mortgage), but may not
encumber the interests of another co-tenant. The mortgagee can only foreclose on the mortgaging
co-tenant's interest.

(4): Any co-tenant has a right to judicial partition either in kind (physical division of land among
co-tenants), or by sale and division of the proceeds from the sale.

(5): A co-tenant who pays more than his/her pro-rata share of necessary repairs is entitled to
contribution from the other co-tenants, provided the other tenants were notified as to the need for
repairs. However, in contrast, there is no right of contribution from other co-tenants for the cost
of improvements to the property that do not meet the standard of necessary repairs.

Posted by Sean Silverman at 3:28 PM 0 comments


Email ThisBlogThis!Share to TwitterShare to FacebookShare to Pinterest
Labels: Real Property
Tuesday, May 19, 2015
MBE Fast Fact: Fixture Filings
Article 9 of the UCC (Secured Transactions) can creep its way into Real Property questions on
the MBE. Thankfully, if it does show up, it'll show up in a very limited context. The questions
will play out with a seller of personal property lending money to a purchaser to purchase the
personal property, and the seller taking out a Purchase Money Security Interest ("PMSI") as
security for the loan. The item of personal property is then affixed by the purchaser to the
purchaser's real property which is subject to a mortgage. The purchaser defaults on all loans, and
then it must be determined whether the seller or the mortgagee has priority as to the sale of the
fixture.

The important rule to keep in mind is the following: Even if the seller records his security
interest after the mortgagee records his mortgage on the real property, the seller can still gain
priority over the mortgagee as to the fixture, if the seller files what is called a fixture filing. The
seller must record his security interest within 20 days after the item of personal property is
affixed to the land.

If seller made that UCC filing in the required time period, seller may remove those fixtures that
were the subject of the filing; if not, seller's security interest in the fixture is subordinate to the
earlier mortgage on the property so that if the mortgagee forecloses, seller's interest in the item of
personal property may be lost in the foreclosure sale.
Posted by Sean Silverman at 5:02 PM 0 comments
Email ThisBlogThis!Share to TwitterShare to FacebookShare to Pinterest
Labels: Real Property
Wednesday, May 13, 2015
MBE Fast Fact: Citizenship
When analyzing diversity jurisdiction for purposes of Civil Procedure, along with the required
jurisdictional amount (more than $75,000) you'll need to ensure that every plaintiff is a citizen of
a different state from every defendant. This requirement is known as complete diversity, and the
analysis requires that you understand how to determine the citizenship of individuals and entities
to ensure that the requirement has been satisfied. There are 3 categories to keep in mind here:

(1): Individuals: The state of citizenship of a natural person depends on the permanent home to
which s/he intends to return. Note also that the citizenship of a child is that of his/her parent(s).

(2): Corporations: A corporation is a citizen of every US state and and foreign country in
which it has been incorporated as well as the one US state or foreign country in which it has its
principal place of business. The principal place of business is the US State or foreign country
from which its high level officers control and coordinate the activities of the corporation.

(3): Unincorporated Associations and Limited Liability Companies: Those businesses in this
category (which includes partnerships) are deemed citizens of each state of which any member of
the association is a citizen.

It's important to remember that complete diversity is required. So that if analyzing the citizenship
of an unincorporated association, for example, be sure to note if there are multiple states of
which the association will be deemed to be a citizen. If any of those states is the same as the
citizenship of the opposing party, complete diversity has not been satisfied.

MBE Fast Fact: Wrongful Birth vs Wrongful Life


Let's be sure to distinguish these concepts as they do show up on the MBE:

In short, wrongful life means the the child sues the mother or others for being born. Wrongful
birth, on the other hand, means the mother sues others (very often a doctor who fails to inform
the mother of the potential for a disability) for being burdened monetarily with a disabled child.
Wrongful birth cases are nearly always medical malpractice tort cases.

For purposes of the MBE, it's most important to remember the following: Most states reject a
claim of wrongful life, and of the few states that do permit it, some would limit recovery to the
special damages attributable to the disability.

In contrast, a cause of action for wrongful birth will be permitted in many states. Though
allowable, damages will generally be limited to expenses incurred as a result of the child's
disabilities.
Posted by Sean Silverman at 7:13 PM 0 comments
Email ThisBlogThis!Share to TwitterShare to FacebookShare to Pinterest
Labels: Torts
Wednesday, February 4, 2015
MBE Fast Fact: The Appointments Clause
The Appointments Clause in the Constitution provides that the President shall nominate and by
and with the advice of the Senate shall appoint ambassadors, Supreme Court Justices, and all
other officers of the United States. This issue, though not as common on the MBE as many
others, has appeared in the past.

It is a violation of this clause if the President, when appointing an ambassador, is required to pick
from among names appearing on a congressionally-generated list. Though as per the clause the
President's pick is subject to confirmation by the Senate, when picking the ambassador (or other
principle federal officer), the President may act alone.

In addition, the rule regarding confirmation by the Senate is specific; namely, the appointment
must be affirmatively approved by a vote of the Senate. If questions are presented in which the
Senate attempts to avoid its obligation by a claim (for example by means of a statute) that a lack
of disapproval of the appointment will be deemed as approval, such action will violate the
Appointments clause, and any statute to that effect should be deemed unconstitutional.
Posted by Sean Silverman at 8:06 PM 0 comments
Email ThisBlogThis!Share to TwitterShare to FacebookShare to Pinterest
Wednesday, January 21, 2015
Requests for Jury Instructions
With Federal Civil Procedure now tested, there are two things you should know going into the
MBE next month regarding jury instructions (instructions to the trial jury on the law that governs
the verdict). The first relates to the rules concerning the request for jury instructions, and the
second relates to the rules regarding objections to those request. Let's look at each aspect
individually:

At the close of the evidence or at any earlier reasonable time that the court orders, a party may
file and furnish to every other party written requests for the jury instructions it wants the court to
give. Note the specifics here; namely that every other party must be furnished with the written
requests. In addition, if the requests are not made at the close of evidence or prior, then after the
close of the evidence, a party may file requests for instructions on issues that could not
reasonably have been anticipated by an earlier time. Finally, with permission by the court, a
party may file untimely requests for instructions on any issue.

The court itself has obligations in regards to jury instructions as well. The court must inform the
parties of its proposed instructions and proposed action on the requests before instructing the jury
and before final jury arguments. Further, it must give the parties an opportunity to object on the
record and out of the jury's hearing before the instructions and arguments are delivered. The
court may instruct the jury at any time before the jury is discharged.

If a party is to object to jury instructions, the party who objects to an instruction or the failure to
give an instruction must do so on the record, stating distinctly the matter objected to and the
grounds for the objection. The objection will be timely if made either when the court provides
the opportunity to do so, or if the party was not provided the opportunity to object, if the party
objects promptly after learning that a jury instruction has been given or refused.
Posted by Sean Silverman at 8:20 AM 0 comments
Email ThisBlogThis!Share to TwitterShare to FacebookShare to Pinterest
Labels: Federal Civil Procedure
Wednesday, January 7, 2015
MBE Fast Fact: Equitable Conversion
In all questions concerning the sale of real property there are two important dates to keep in
mind: The first is the date of the contract between buyer and seller, and the second is the date of
closing when legal title is delivered from seller to buyer. But there will also be time between
those two dates, and on the MBE many terrible things will occur during that time that will effect
the analysis.

Specifically, the doctrine of equitable conversion supplies the rules for dealing with those events
that occur between the contract and the date of closing. The doctrine holds that once the parties
have entered into a contract, the buyer's interest in the contract is converted into real property,
and the seller's interest is converted into personal property.

And that conversion greatly affects the analysis. For example, assume X is the seller, and Y is
the buyer. X dies after the contract is entered into with Y but before the date when legal title was
actually transferred to Y. Because legal title has not actually changed hands, it might be expected
that X's heirs would receive the real property upon the death of X. But, rather, due to the doctrine
of equitable conversion, by the time of seller's death, seller was no longer deemed to have held
an interest in the real property. Instead, seller held an interest in the personal property,
specifically the proceeds from the sale of the real property.

Seller's heirs may be entitled to those proceeds, but buyer will be entitled to the real property.
And on that note, if buyer were to die during the period prior to closing, buyer's heirs would have
the right to the real property.
Posted by Sean Silverman at 10:07 PM 0 comments
Email ThisBlogThis!Share to TwitterShare to FacebookShare to Pinterest
Labels: Real Property
Sunday, December 28, 2014
MBE Fast Fact: Abstention
In prior posts I've written about some limitations on the exercise of federal jurisdiction. Some
examples of such limitations are a lack of standing, the rule against advisory opinions, and cases
deemed moot or not yet ripe. Another limitation to keep in mind occurs when a federal court
abstains from resolving a constitutional claim. For MBE purposes, keep in mind that a federal
court will temporarily abstain from resolving a constitutional claim where the disposition rests
on an unsettled question of state law.

Further, federal courts will not enjoin pending state criminal proceedings (and sometimes state
civil proceedings) even if the federal court has jurisdiction over the case. A case is generally
deemed to be pending the moment it is filed. Rather, the federal court will wait until after the
state prosecution has ended to hear the case.

There is an important exception here, however, to keep in mind. A federal court will hear an
action to enjoin a pending state court proceeding if is being conducted in bad faith (for example,
a case moving forward in state court merely to harass the defendant).

Motions
Distinguishing between a variety of motions is sure to be required for those answering civil
procedure questions on the MBE. Below are five important motions made during trial to keep in
mind and the distinguishing characteristics among them:

Motion for Judgment on the Pleadings: This motion may be granted if on the face of the
pleadings (but without considering matters outside of the pleadings) the court determines that the
moving party is entitled to judgment.

Judgment as a Matter of Law: This motion was formerly known as a motion for a directed
verdict. Granting this motion is allowable for either party if the evidence is such that reasonable
people could not disagree in favor of the moving party. The evidence must be viewed in the light
most favorable to the non-moving party. If one is to later make a renewed motion for judgment
as a matter of law, one must first make this particular motion.

Renewed Motion for Judgment as a Matter of Law: This motion was formerly known as a
motion for judgment notwithstanding the verdict ("JNOV"). The motion is filed after judgment
but not later than 28 days after entry of the judgment. The motion should be granted if the verdict
returned could not have been reached by reasonable persons. A party is generally limited to those
issues raised in the motion for judgment as a matter of law.

Motion for a New Trial: As with the renewed motion for judgment as a matter of law, this
motion is filed after judgment but not later than 28 days after judgment. The court has the
discretion to grant a new trial because of an error during trial, or because the verdict is against
the weight of the evidence. Occasionally, the court will issue a new trial if the verdict is
excessive, though another option is to offer the plaintiff remittitur, which allows plaintiff to
choose between a lesser award or a new trial.

Motion for Summary Judgment: Summary judgment shall be granted if, from the pleadings,
affidavits, and discovery material on file, it appears that no genuine issue of material fact exists
and that the moving party is entitled to judgment as a matter of law. This motion is very similar
to the a motion for judgment on the pleadings, though more material is considered here in
determining whether to grant the motion. A party may file a motion for summary judgment at
any time until 30 days after close of discovery.
Posted by Sean Silverman at 9:19 PM 2 comments
Email ThisBlogThis!Share to TwitterShare to FacebookShare to Pinterest
Labels: Federal Civil Procedure
Monday, December 1, 2014
MBE Fast Fact: The Firm Offer Rule
The Firm Offer Rule is definitely one those distinctions between the common law of contracts
and the UCC that you'll want to keep in mind when preparing for the MBE. Under the common
law of contracts an option contract is available in which the offeree gives consideration for a
promise by the offeror not to revoke the offer. Because the offeree provides consideration, the
promise not to revoke is binding on the offeror. Such offer may not be revoked for the specific
time period stated in the offer.

But under the UCC, the rules change in a very significant way. Under Article 2 of the UCC, if a
merchant offers to sell goods in a signed writing and the writing gives assurances that it will be
held open, then the offer may not be revoked for the period specified by the merchant.
Consideration is not required. Watch for situations in which no time period is stated; under such
circumstances, the offer may not be revoked for a reasonable period of time, but in no event may
that time period exceed 3 months.

That last part is important; even if the merchant claims that the offer will remain open for 4
months, the merchant making the offer will only be bound to keep the offer open for 3 months.
But the merchant can always limit the time period in which he is required to keep the offer open
by specifically stating a time period less than 3 months.

MBE Fast Fact: Interpleader


Occasionally, situations will arise in which a person (often deemed a "stakeholder") will want to
determine who among adverse claimants has a valid claim to an item of property ("the stake").
As such, both the Federal Rules of Civil Procedure and federal statutes allow for that stakeholder
to institute an interpleader suit to make this determination. It applies only if separate actions
might result in double liability against the stakeholder.

There are two interpleader procedures in the federal courts, and both procedures should be
known for purposes of the MBE:

(A): Rule 22 Interpleader: Rule 22 requires either that (1): There is complete diversity between
the stakeholder and all adverse claimants and that the $75,000 amount in controversy is satisfied,
or (2): federal question jurisdiction applies. The regular rules for service & process and venue
apply.

(B): Section 1335 interpleader (sometimes referred to as "statutory interpleader") requires only
minimum diversity between the claimants. This minimum diversity will be satisfied if one
claimant is diverse from one other, even if all claimants are not diverse from all others), and that
at least $500 is at issue. Service may be nationwide and venue is proper where any claimant
resides.

MBE Fast Fact: Joinder of Claims


The issue of joinder of claims is implicated when plaintiff has sued defendant on one claim and
wants to join other claims to the same lawsuit. Joinder of claims in such an instance is allowable,
though if there are multiple plaintiffs or multiple defendants, then it is necessary that at least one
of the claims arise out of a transaction in which all were involved.

A few jurisdictional issues to keep in mind here. If jurisdiction is based on diversity of


citizenship, then plaintiff may aggregate all claims against a single defendant in order to satisfy
the jurisdictional amount. Further, once the jurisdictional amount has been satisfied in respect to
one claim, other claims that do not satisfy the jurisdictional amount may invoke supplemental
jurisdiction if they arise from the same common nucleus of operative facts as the claim that did
satisfy the amount.

If, instead, jurisdiction is based on a federal question claim, a nonfederal claim may be joined
provided it is part of the same case or controversy as the federal claim.

Joinder of Parties
A lot to know about joinder as you study Civil Procedure for the MBE. This post will focus
entirely on joinder of parties, and on a future post I'll write about joinder of claims. In regards to
joinder of parties, it can further be subdivided into two main categories: compulsory joinder of
parties, and permissive joinder of parties.

Compulsory Joinder:

There are instances in which a party should be joined to an already-existing lawsuit. A party
should be joined if complete relief cannot be granted to the existing parties in the absence of the
party to be joined. In addition, a party should be joined if the absence of that party may impair
that party's ability to protect his own interests in the controversy. And finally, a party should be
joined if his absence would expose the existing parties to a substantial risk of double or
inconsistent obligations. If any of the above are satisfied, then that party must be joined provided
the party is amenable to service of process, and joinder of that party will not destroy diversity or
venue.

But the question is then raised as to what is the procedure if such a party should be joined but
cannot be joined. When joinder is not feasible the court must decide as to whether the inability to
join that party renders the action dismissed, or instead whether the action should continue
without the party. Once again, the court will have some considerations to attend to when making
this decision. The court will determine whether a judgment in the absence of the party would
prejudice that party or the existing parties to the lawsuit. The court will also consider whether
any prejudice can be effectively reduced, and whether a judgment in the party's absence would
be adequate. Lastly, the court will consider the interests of the plaintiff in the action; if dismissal
of the action will result in depriving the plaintiff of an adequate remedy, the dismissal will be
less likely.

Permissive Joinder:

Other instances present when joinder of parties is permissive even though the requirements for
compulsory joinder are not satisfied. Specifically, parties may join an existing lawsuit if some
claim is made by each plaintiff and against each defendant relating to or arising out of the same
series of occurrences or transactions, and there is a question of fact or law common to all parties.
It's important to note here that although this rule is broad, the requirements as they relate to
diversity jurisdiction are not altered. In other words, if the action is based on diversity of the
parties, then there must be complete diversity among the parties after additional parties have
been joined
Posted by Sean Silverman at 5:05 PM 0 comments
Email ThisBlogThis!Share to TwitterShare to FacebookShare to Pinterest
Labels: Federal Civil Procedure
Monday, September 29, 2014
MBE Fast Fact: Rule 11
Rule 11 of the Federal Rules of Civil Procedure is implicated whenever an attorney (or
unrepresented person) presents to the court a pleading, written motion, or other paper. The rule
provides that whenever such a paper is presented, the presenter certifies that to the best of his/her
knowledge the paper has not been presented for any improper purpose (for example, harassment,
delay, etc.) In addition, it is certified that that the legal contents therein are warranted by existing
law or contain an argument for the modification of existing law. It is further certified that the
allegations and factual contentions therein have evidentiary support and that denials of any
factual contentions are warranted by the evidence or are reasonably based on a lack of
information or belief.

It's important to understand the consequences of violating the rule. Violations of Rule 11 can
lead to sanctions, and the court has discretion to impose sanctions against a party who presents a
paper in violation of the above requirements. The matter may be raised by the court on its own
initiative or the opposing party may serve a motion for sanctions with the court. Sanctions may
consist either of nonmonetary directives or monetary penalties, and the monetary penalties may
include payment of expenses or attorney's fees incurred as a result of violating the rule.
Posted by Sean Silverman at 9:10 PM 0 comments
Email ThisBlogThis!Share to TwitterShare to FacebookShare to Pinterest
Labels: Federal Civil Procedure
Sunday, September 21, 2014
The Top Five: The Complaint and Pre-Answer Motions
The topic of pleadings is sure to be a heavily tested topic in Civil Procedure on the MBE. Here
are five things to know about the complaint and pre-answer motions under the federal rules:

(1): The federal rules require notice rather than fact pleadings. Notice pleadings require less
specificity than fact pleadings, though a certain level of specificity will still be required.

(2): A complaint must state the following: the grounds of federal jurisdiction, a short statement
of the claim showing that the pleader is entitled to relief, and a demand for judgement for relief
(which may be in the alternative).

(3): An important motion to keep in mind is a rule 12(b) motion. Here, the defendant files a
motion raising any or all of the following defenses: lack of subject-matter jurisdiction, lack of
personal jurisdiction, improper venue, insufficiency of process, insufficiency of service of
process, failure to state a claim upon which relief can be granted, or failure to join an
indispensable party. If the defendant files a pre-answer motion prior to filing an answer then
these defenses must be raised in that pre-answer motion. If the defendant files an answer prior to
filing a motion, then these defenses must be filed in that answer. If not filed as specified, then
defendant waives these defenses, except for failure to state a claim upon which relief can be
granted and failure to join an indispensable party, both of which may be raised at any time prior
to trial or at trial. In addition, lack of subject matter jurisdiction may be raised at any time until
all appeals have been exhausted.

(4): If a party deems a pleading to be vague, that party may raise a 12(e) motion for a more
definite statement before responding to the pleading.

(5): Rule 12(f) allows a party to move to have stricken any insufficient defense or any redundant,
immaterial, or scandalous matter in a pleading. This motion must also be made prior to
responding to the pleading.

Defenses to Negligence
The default defense to negligence on the MBE is pure comparative negligence. But often enough
the questions will direct you to apply a different standard, so it's important to understand how the
analysis changes when the standard changes. Three defenses you'll want to know are pure
comparative negligence, modified (or partial) comparative negligence, and contributory
negligence.

Let's assume the following fact situation in discussing the analysis of each. Plaintiff ("P") is
injured by 3 defendants "X," "Y," and "Z." The fault (in dollar amount) of plaintiff's injury is
allocated as follows:

P's fault is valued at $50,000

X's fault is valued at $30,000

Y's fault is valued at $20,000

Z's fault is valued at $10,000

Total: $110,000

Contributory Negligence: If directed in any given question to apply contributory negligence,


the analysis is very straight-forward. If plaintiff is determined to have been at fault (by even the
slightest amount), then plaintiff is prevented from recovering anything from any other defendant.
In our example, plaintiff is $50,000 at fault, so plaintiff will recover nothing. Note that plaintiff
can then raise his own defense of the "last clear chance doctrine," claiming that defendant(s) had
the last clear chance to avoid the act that caused the injury. If proven, then the fact that plaintiff
was contributorily negligent will not entirely prevent plaintiff from recovering.

Modified (partial) comparative negligence: Here the analysis changes. Plaintiff can recover
provided that plaintiff was less than 50% at fault when taking the total dollar amount of damages
into account. Here, plaintiff was determined to have been $50,000 at fault, and total damages
were valued at $110,000. Therefore plaintiff was 45% percent at fault (50,000/110,000), and
because that is less than 50%, plaintiff will recover for his injuries. In determining the amount of
recovery, subtract the amount of damages allocated to plaintiff ($50,000) from the total amount
($110,000). Plaintiff will therefore recover $60,000. Plaintiff can recover this amount from either
x, y, or z (as per the default rule of joint and several liability), and then each defendant will have
a right of contribution from the other defendants if any defendant pays an amount that exceeds
his own fault. So, for example, if plaintiff recovers the $60,000 from z, z will have a right of
contribution of $50,000 from x and y.

Pure Comparative Negligence: This is the standard to apply if you are not told to apply
otherwise. The analysis is the same as in modified comparative negligence, except that under
pure comparative negligence, plaintiff can recover even if plaintiff is determined to have been
more than 50% at fault. So, assume that instead of damages valued at $110,000, the total
damages were valued at $90,000 and further assume that plaintiff was still determined to have
contributed $50,000 to his own injuries. In that case plaintiff would be deemed 56% at fault
(50,000/90,000). Even so, plaintiff would be able to recover for the remaining fault allocated to
defendant(s). In this example, plaintiff would be able to recover $40,000, and once again,
assuming the default rule of joint and several liability applies, plaintiff may choose among x, y,
or z, when recovering that amount, and then each defendant will have a right of contribution
against the others.

**Note:: If told not to apply joint and several liability, the analysis changes slightly. Plaintiff can
no longer recover the full amount from any given defendant, and can only recover an amount
from a defendant equal to that defendant's fault.
Posted by Sean Silverman at 6:48 PM 0 comments
Email ThisBlogThis!Share to TwitterShare to FacebookShare to Pinterest
Labels: Torts
Sunday, August 17, 2014
The Top Five: Venue
It's important when studying Civil Procedure to understand well the difference between subject-
matter jurisdiction and venue. Subject-matter jurisdiction is the power of the court to adjudicate
the matter before it, while venue relates instead to a determination as to which district court
should hear the case. Here are five things to know about venue when preparing for the MBE.

1. The general rule regarding venue is that venue is proper in any judicial district where any
defendant resides if all defendants reside in the same state. In addition, venue is proper in a
judicial district in which a substantial part of the events or omissions giving rise to the claim
occurred, or a substantial part of the property that is the subject of the action is situated.

2. There will be times when the standards above are not satisfied. In such instances, venue will
be proper in a judicial district in which any defendant is subject to personal jurisdiction, but only
if the action is based solely on diversity. If the action is not based solely on diversity, then venue
will be proper in a judicial district in which any defendant may be found.

3. In determining if venue is proper it is sometimes necessary to determine the defendant's


residency (such as in 1, above). For purposes of venue, an individual's residence is determined
by that individual's domicile. A corporation resides in any jurisdiction in which it is subject to
personal jurisdiction, and an unincorporated association (such as a partnership) resides wherever
it does business.

4. Improper venue may be waived by the parties. And importantly, venue is considered waived
unless timely objection is made to improper venue.

5. There are situations in which, although venue is proper, the court may still wish to transfer the
case for the convenience of the parties. This is allowable under the federal rules provided that the
case is transferred to a court where it could have originally been filed. The standard here is the
"interest of justice," and the court to which the case is transferred must have subject-matter and
personal jurisdiction over the defendant; otherwise, venue in the transferee court is improper.
Posted by Sean Silverman at 8:33 PM 0 comments
Email ThisBlogThis!Share to TwitterShare to FacebookShare to Pinterest
Labels: Federal Civil Procedure
Wednesday, August 6, 2014
Federal Question Jurisdiction
In the most recent post, I outlined the important points concerning diversity jurisdiction.
Diversity jurisdiction is one avenue for getting a case into federal court. The other is federal
question jurisdiction.

The district federal courts have original jurisdiction over all civil actions arising under the
Constitution, laws, or treaties of the United States. It's important to note that the federal question
for which jurisdiction is based must appear as part of plaintiff's cause of action. In other words,
the fact that there is an asserted defense that implicates a federal question is not sufficient to
confer jurisdiction if plaintiff has not also asserted a federal question.

It gets a bit more complex when there are multiple claims, some of which assert a federal
question, and some of which do not. In such instances the federal court will have discretion (as
per its pendent jurisdiction) to hear all claims that derive from a "common nucleus of operative
fact." That's a rather odd way of stating the idea, but what it means is that if the claims all arise
from the same set of facts, then the the fact that some of the claims are based on state law will
not prevent a federal court from hearing those state-law claims provided there is at least one
federal claim to warrant jurisdiction. And even if the federal claim on which jurisdiction is based
ends up getting dismissed after trial has commenced, the federal court may still exercise pendent
jurisdiction over the state claims.

It's also possible that questions may arise in which plaintiff sues more than one defendant, and
there is a federal claim against only one of the defendants. Here, too, pendent jurisdiction
(specifically, pendent-party jurisdiction) will allow the federal court to hear the non-federal
claims against all defendants provided that the claims once again derive from a "common
nucleus of operative fact."

***note that the term pendent jurisdiction is interchangeable with the term supplemental
jurisdiction.

Posted by Sean Silverman at 9:13 PM 0 comments


Email ThisBlogThis!Share to TwitterShare to FacebookShare to Pinterest
Labels: Federal Civil Procedure
Monday, August 4, 2014
Diversity Jurisdiction
The MBE is about to change. Beginning in February of 2015, a new subject, Civil Procedure,
will be added to the testable list of subjects. As such, I'll be posting quite a lot on this subject
prior to the exam. Today's post is on the topic of diversity jurisdiction.

Subject-matter jurisdiction is the requirement that the court have power to hear the specific kind
of claim that is brought to that court. The two primary sources of subject-matter jurisdiction of
the federal courts are diversity jurisdiction and federal-question jurisdiction. If jurisdiction is
predicated on diversity, then every plaintiff must be of diverse citizenship from every defendant.
Stated differently, if one defendant and one plaintiff are co-citizens of the same state, there is no
diversity jurisdiction. Though there are exceptions to this rule, which will be discussed in later
posts, it's always important to first understand the general rule.

In addition, diversity of citizenship as outlined above must exist as of the time that suit is
instituted. As such, it need not exist at the time the cause of action arose, and diversity will not
be defeated if after the suit is instituted, a plaintiff and defendant become citizens of the same
state.

In determining whether there is complete diversity, it's often necessary to determine the
citizenship of each party. For individuals, it's fairly simple: an individual is a citizen of his/her
domicile, or the permanent home to which s/he intends to return. A corporation is a citizen of
every state of its incorporation and also of its principal place of business. If the corporation
performs its business in various states, look to see where the executive office is located, as that
state will be deemed the principal place of business for determining citizenship. It's a bit different
for unincorporated businesses such as partnerships. There, you'll want to determine the
citizenship of each member, as the business will be deemed a citizen of each state of which any
member is a citizen.

When analyzing diversity, in addition to determining that the citizenship requirement has been
satisfied, it's also necessary to determine that the jurisdictional amount has been satisfied. When
subject-matter jurisdiction is based on diversity, actions must be in excess of $75,000 exclusive
of interests or costs. The amount is entirely determined by plaintiff's good faith allegations. In
other words, if the plaintiff alleges an amount in excess of $75,000, the complaint can be
dismissed only if it appears there is no legal possibility for the plaintiff to recover the amount
alleged. And importantly, if the plaintiff ends up not recovering an amount in excess of $75,000,
this has no effect on jurisdiction, as jurisdiction is not retroactively defeated. For purposes of
satisfying the jurisdictional requirement, plaintiff may aggregate all claims against a single
defendant (and against multiple defendants if those defendants are jointly liable to plaintiff).

MBE Fast Fact: Priority of Mortgages


Mortgage questions are some of the most difficult questions on the MBE. Some of them are just
brutal. One issue that comes up often enough that it's worth knowing well involves a senior
mortgagee who fails to include a junior mortgagee in a foreclosure sale.

As a general rule, the priority of a mortgage is determined by the time it was placed on the
property (Exceptions do apply, such as purchase-money mortgages which can have priority over
prior mortgages placed on the property.) Generally foreclosure will terminate interests junior to
the mortgage being foreclosed, but will have no effect on mortgages senior to the mortgage being
foreclosed.

Watch carefully, however, for situations where a senior mortgagee fails to include a junior
mortgagee as a party to the foreclosure action. Such failure on the part of the senior mortgagee
will preserve the rights of a junior mortgagee so that the general rule that the junior mortgagee's
rights are extinguished will not apply.

The policy behind the above analysis is that the junior mortgagee has the option to pay off any
liens senior to the junior mortgagee that are in default, and that right makes the junior mortgagee
a necessary party to any foreclosure action on the part of a senior mortgagee. By not including a
necessary party in the foreclosure action, the senior mortgagee effectively preserves the rights of
the junior mortgagee. Anyone who then purchases the property at the foreclosure sale will
purchase subject to the interests of the junior mortgagee.

The Top Five: Permissive Waste by a Life Tenant


Permissive waste occurs when a life tenant either allows land to fall into disrepair, or fails to take
reasonable measures to protect the land. This concept will nearly always be tested in the context
of someone holding a remainder interest suing a life tenant for waste. As such, it's important to
understand the obligations of a life tenant in determining whether such a lawsuit will be
successful. Most importantly you should understand the following five rules in regards to
obligations:

(1): A life tenant has an obligation to repair the property. The obligation, specifically, is to keep
the property in a reasonable state of repair. The life tenant is under no obligation to make
permanent repairs (as the tenant only holds the property for the duration of his life), but is under
an obligation to make repairs to the extent of income or profits from renting the property, or if
the tenant is not renting the property, to the extent of the reasonable rental value of the land.

(2): A life tenant is obligated to pay interest on any encumbrances on the land. A common
encumbrance in this respect would be a mortgage. Note that the tenant, however, is not required
to the pay the principal on the encumbrance; that obligation is on the future interest holder (most
likely someone either holding a remainder or a reversion).

(3): A life tenant is obligated to pay all ordinary taxes on the land.

(4): If there is a public improvement on the land, and such improvement is shorter than the
expected duration of the life estate, then the life tenant is obligated to pay all of the assessment
for the public improvement. If, however, the improvement is likely to outlast the estate, then the
analysis changes; namely, the taxes and assessment for such an improvement will be apportioned
equitably between the life tenant and the holder(s) of any future interest.

(5): Importantly, note that a life tenant is not responsible for insuring the property for the benefit
of any future interest holder, and is similarly not responsible to the future interest holder for any
damages to the land caused by a third party.

MBE Fast Fact: Warranty of Habitability


The warranty of habitability seems to pop up quite a lot in Real Property questions. It's easy
enough to understand that the warranty places burdens on the landlord, and those burdens are
imposed regardless of whether the landlord agrees to them in the contract between the landlord
and tenant. But the questions will require that you understand specifically which obligations this
warranty covers, as well as how it affects the other rights and obligations of both the landlord
and tenant. Be sure to know the following:

The warranty generally only applies to residential tenancies (as opposed to commercial
tenancies). Essentially, the warranty is an unstated guarantee that a rental property meets basic
living and safety standards. So, when a tenant rents an apartment, for example, an implied
warranty of habitability requires that the unit will have hot water, a working electrical system,
heat in the winter, lockable doors and windows, etc. And what happens if the landlord breaches
this warranty?

The tenant has a few options here. One option is to terminate the lease, but if termination is not
desired, the tenant can make repairs and then offset the cost against future rent obligations.
Another option is to forgo making the repairs and to abate the rent to an amount equal to the fair
market value of the defective property. (Though this option is far less desirable depending upon
the specific defects.) Finally, the tenant has the option to remain in possession, pay full rent, and
then seek damages directly from the landlord.

Specific Intent Crimes


One of the first distinctions to focus on when studying Criminal Law for purposes of the MBE is
the distinction between specific intent crimes and general intent crimes. There are two primary
reasons for ensuring you understand this distinction; the first is that the mere existence of a
specific intent crime cannot be conclusively imputed simply by learning that a person has done
an act. In other words, there is a common element that must be proven when analyzing all
specific intent crimes, and that element is the existence of the specific intent. As an example, a
classic specific intent crime is first-degree premeditated murder. It's not enough for the
prosecution to prove that x performed the act of shooting y; the prosecution will have to prove as
well that x shot y with the intent to kill him.

In addition to the above, another reason for identifying specific intent crimes is because some
defenses (the often tested question in this context deals with voluntary intoxication), apply only
to specific intent crimes.

There is a lot to memorize when preparing for the MBE. Here is yet another list you should
know. The following lists the specific intent crimes for purposes of the MBE:

Solicitation
Attempt
Conspiracy
First Degree Premeditated Murder
Larceny
Robbery
Assault (intent to commit a battery)
Burglary
Forgery
False Pretenses
Embezzlement
Posted by Sean Silverman at 5:38 PM 0 comments
Email ThisBlogThis!Share to TwitterShare to FacebookShare to Pinterest
Labels: Criminal Law
Sunday, March 23, 2014
Marketability of Title
When preparing for the MBE, you're sure to come across questions testing the concept of
marketable title. Because the examiners test this concept in a variety of ways, it's important to
know it well. And the first thing to understand is that there is an implied warranty in every land-
sale contract that at closing the seller will provide the buyer with marketable title.

Questions often require a determination as to whether title is marketable, and the best way to
address such questions is to understand what types of defects cause title to be deemed
unmarketable. Defects in the chain of title can render title unmarketable, as can adverse
possession. The adverse possession questions show up often, so note carefully that on the MBE,
title acquired by adverse possession is not marketable, despite the fact that modern law is
generally to the contrary. Though you should always look for twists in the questions in which the
examiners tell you to apply modern law, the default rule is that because a purchaser of land
acquired by adverse possession may be forced to defend in court his rights to the land, the title is
deemed unmarketable.

Further, a variety of encumbrances on land will deem the title to that land unmarketable. These
encumbrances include mortgages, liens, easements, and covenants. In regards to mortgages,
you'll come across questions in which there is a mortgage on the property that is the subject of
the sale, and buyer claims that title is unmarketable. Seller will have the right to satisfy the
mortgage up until the time of closing, and can use the proceeds of the closing to satisfy the
mortgage. Therefore, oftentimes the closing itself will render title marketable.

An easement that reduces the value of the property renders title unmarketable. Distinguish this
from an easement that benefits property (for example a utility easement allowing others to
service the property) which generally will not render title to that property unmarketable provided
the easement was visible and known to the buyer when the seller and the buyer agreed to the
sale.

Finally, note the rule regarding zoning regulations and their effect on marketability of title.
Generally, zoning restrictions do not effect the marketability of title, as the regulations are not
considered encumbrances. An existing violation of a regulation, however, is far different, and if
there is an existing violation of a zoning regulation, then title to the property on which the
violation is taking place will not be deemed marketable.

MBE Fast Fact: Judicial Notice


In criminal cases, the trial judge may not instruct the jury to accept a judicially noticed fact as
conclusive. This is because to do so would impermissibly limit the defendant's right to a jury
trial. The distinction to note here is that the court must take judicial notice of a fact if the court is
supplied with the necessary information to indicate that the fact is not subject to reasonable
dispute; however, in a criminal case the court must instruct the jury that it may or may not
accept the noticed fact as conclusive.
Posted by Sean Silverman at 8:36 PM 0 comments
Email ThisBlogThis!Share to TwitterShare to FacebookShare to Pinterest
Labels: Evidence
Monday, February 3, 2014
MBE Fast Fact: Robbery (The Requirement of Force)
Though it's true that robbery requires the taking of personal property through the use of force, if
a victim is rendered unconscious or helpless, and then property is taken from the victim while
unconscious, the act of rendering the victim unconscious (even though it occurred prior to the
taking of the property) will suffice to satisfy the force requirement for robbery. It's these kinds of
"curveballs" that the writers of the test often add to the questions. The more practice questions
you work through, the less likely you'll get tripped up on such oddities.
Posted by Sean Silverman at 11:21 PM 0 comments
Email ThisBlogThis!Share to TwitterShare to FacebookShare to Pinterest
Labels: Criminal Law
Wednesday, January 29, 2014
The Top Five: Adoptive Admissions
Admissions by a party opponent (an exemption to the rule against hearsay) are relatively easy to
spot, but a subtle twist on this rule deals with situations in which silence itself can constitute an
admission by a party opponent. When silence constitutes an admission by a party opponent it's
known as an adoptive admission, and below are five important points to keep in mind regarding
adoptive admissions:

(1): An adoptive admission occurs when a person expressly or impliedly adopts or acquiesces to
an accusatory statement of another by remaining silent in response to the accusation.

(2): For silence to constitute an admission, it must be true that a reasonable person would not
have remained silent in response to the accusatory statements.

(3): When analyzing adoptive admissions, be certain that the party claimed to have made an
admission heard and understood the statement. Further, it must be true that the party was
physically and mentally capable of denying the statement.

(4): In addition, only if a reasonable person would have affirmatively denied the statement can a
statement be deemed an admission when one remains silent in response to the statement.

(5): Importantly, note that silence in response to an accusation by the police in a criminal case is
almost never considered an admission of the crime, as the 5th Amendment provides for the right
to remain silent in such situations.
Posted by Sean Silverman at 9:05 PM 0 comments
Email ThisBlogThis!Share to TwitterShare to FacebookShare to Pinterest
Labels: Evidence
Monday, January 20, 2014
Impeachment by Extrinsic Evidence
A common question I've been asked while tutoring students as of late relates to extrinsic
evidence. Most importantly is to understand what is meant by extrinsic evidence and how it plays
into the rules regarding the impeachment of a witness. The simplest method for understanding
the meaning of extrinsic evidence is to note that extrinsic evidence is evidence elicited through
methods other than cross examination of a particular witness. For example, if witness X testifies
about witness Y, the testimony by witness X is deemed extrinsic because the testimony regarding
witness Y was not elicited by cross examination of witness Y.

And for purposes of the MBE, it's important to note that certain grounds for impeachment
require that a foundation be laid during cross examination before extrinsic evidence can be
introduced. For example, impeaching a witness by proving the witness has made a prior
inconsistent statement may be offered through either cross examination of the witness or by
extrinsic evidence. To prove the prior inconsistent statement by extrinsic evidence, however,
requires that at some point the witness is given an opportunity to explain or deny the statement.
Note, however, that this rule requiring an opportunity to explain the statement will not apply
where the interests of justice require. In addition, extrinsic evidence is not allowable on prior
inconsistent statements unless those statements are material. Otherwise the statements are
deemed collateral, and the "collateral matter rule" prevents proving such statements by extrinsic
evidence.

Similarly, evidence that a witness is biased may be offered both by cross examination and by
extrinsic evidence. Here, however, the rules require that before a witness can be impeached
through extrinsic evidence, the witness must first be asked, on cross examination, about the facts
that show bias.

In regards to impeachment by conviction of a crime, both cross examination and extrinsic


evidence are allowable. The extrinsic evidence in such instances will consist of a record of the
judgment.

Impeachment by specific bad acts diverts from the above, and the diversion is worth noting.
Cross examination to elicit evidence of the bad acts of the witness is allowable, but extrinsic
evidence of such acts is not permitted. In other words, if on cross examination a witness is asked
about a prior bad act (probative of truthfulness), and the witness denies have engaged in the act,
a second witness cannot be called to dispute the denial as calling a second witness would be an
attempt to prove the prior bad act by extrinsic evidence which is not allowable here.

Finally, note that sensory deficiencies (for example, claiming the witness was not wearing
glasses at the time the witness claims to have witnessed a crime) can be proven both by cross
examination of that witness, and by extrinsic evidence.

Posted by Sean Silverman at 8:59 PM 0 comments


Email ThisBlogThis!Share to TwitterShare to FacebookShare to Pinterest
Labels: Evidence
Tuesday, December 17, 2013
The Top Five: The Rule Against Restraints on Alienation
It seems that the Rule Against Perpetuities gets all the attention. A similar sounding doctrine,
The Rule Against Restraints on Alienation, is also likely to show up on the MBE. Here are five
things to know about it:

(1): There are three types of restraints on alienation that will implicate this rule. They are
disabling restraints (any attempted transfers of property will be ineffective,) forfeiture restraints
(any attempted transfer of property will forfeit the interest of the transferor in that property,) and
promissory restraints (any attempted transfer of property breaches a covenant).

(2): It's important to note that all three types of restraints on fee simple estates are invalid. Thus,
the grantee of a fee simple estate may freely transfer the property without worry.

(3): Unlike absolute restraints, limited restraints on fee simple estates may be valid. In other
words, though it is invalid to require that an owner of a fee simple estate never transfer the
property, it may be valid to require that same owner not to transfer the property for a reasonable
time, if there is a reasonable purpose for the restraint.

(4): Some restraints on alienation are always valid. The following fall within that category:
forfeiture and promissory restraints on life estates; forfeiture restraints on transferability of future
interests; reasonable restrictions in commercial transactions; rights of first refusal (but consider
the Rule Against Perpetuities), and restrictions on assignments and subleases of leaseholds.

(5): Only apply the Rule Against Restraint on Alienation to legal interests. The rule does not
apply to equitable interests in property. For example, consider a case in which a spendthrift
clause is placed into a trust by the settlor. Such a clause does restrain the beneficiary from
alienating his interest, but such clauses are valid as the beneficiary has only an equitable interest
in the property (the trustee owns the legal interest).

Posted by Sean Silverman at 5:25 PM 0 comments


Email ThisBlogThis!Share to TwitterShare to FacebookShare to Pinterest
Saturday, November 30, 2013
Hearsay Exceptions Requiring Unavailability
Compared to the number of hearsay exceptions in which unavailability is not required, the
number requiring unavailability is minimal. Yet, there is quite a lot to know about them. Each
one is outlined below, and include the important points:

Former Testimony:

Let's assume a witness (let's call him X) is unavailable at trial 2, but was available at a previous
trial, hearing or deposition. (For simplicity, let's call that previous trial "trial 1".) If X gave
testimony in trial 1, that testimony is admissible in trial 2, provided that X was a party in the
former action. Further, the former action must have involved the same subject-matter as the
current action. In addition, the testimony that X gave must have been under oath, and X had to
have had the opportunity to develop his testimony in the previous action (by direct, cross or re-
direct examination).

Statements Against Interest:

Let's assume X is unavailable at trial but X has made a previous statement outside of court that
the opposing party wants to admit into evidence. The statement is admissible if the statement that
X made was against his pecuniary, proprietary, or penal interest when made. In addition,
collateral facts contained in the statement are likewise admissible. Ensure when analyzing this
rule, that X had personal knowledge of the facts contained within the statement, and that X was
aware that the statement was against his interest when made.

Dying Declarations:

On the MBE, it's important to note that dying declarations only apply to a homicide prosecution,
or a civil action. A statement made by a now unavailable witness is admissible if the declarant
believed that his death was imminent when the statement was made (the belief is what matters,
not the end result), and the statement concerned the cause or circumstance of what he believed to
be his impending death.

Statements of Personal or Family History:

Statements by a now unavailable witness are admissible if those statements concern births,
marriages, divorces, relationships, genealogical status, etc., provided that the witness is a
member of the family in question or intimately associated with the family, and the statements are
based on the declarant's personal knowledge of the facts or knowledge of family reputation.

Statements Offered Against Party Procuring Declarant's Unavailability.

The statement of a person (let's call him X) who is now unavailable as a witness, may be offered
against a party (let's call him Y) who engaged or acquiesced in wrongdoing that intentionally
procured X's unavailability. In other words, if the actions of Y prevent X from attending the trial,
Y will not be able to use that to his advantage by claiming previous statements made by X are
inadmissible hearsay.

Posted by Sean Silverman at 1:42 PM 0 comments


Email ThisBlogThis!Share to TwitterShare to FacebookShare to Pinterest
Labels: Evidence
Sunday, November 24, 2013
How It's Tested: Holdover Tenants
It's not common, but occasionally you'll be asked on the MBE to distinguish between the
"English Rule" and the "American Rule" as these rules relate to holdover tenants.

Specifically, assume landlord leases to tenant 1, and then leases to tenant 2 when tenant 1's lease
is up. Tenant 1 refuses to vacate the premises (he is a holdover tenant), and when tenant 2 arrives
on the day that he is entitled to take possession and finds tenant 1 still possessing the property,
tenant 2 attempts to sue the landlord. Who wins?

The answer depends upon the jurisdiction. Most American courts follow the "English rule" under
which the landlord has a duty to deliver actual possession to the tenant. Under this rule, the
landlord in the above hypothetical would lose, because when tenant 2 arrived and found tenant 1
holding over, tenant 2 was unable to obtain actual possession of the property, and the landlord
had therefore not fulfilled his obligation.

But a minority of jurisdictions follow the "American Rule," under which the landlord is only
required to give the tenant legal possession. In other words, provided the landlord has not given
anyone other than the tenant a conflicting legal right to possess the property, the tenant has no
right to sue the landlord. In the above hypothetical, the only person who had the legal right to
possess was tenant 2 (tenant 1 was holding over but did not have the legal right to do so), and
therefore under the American rule the landlord had fulfilled his obligation, and would be immune
from suit by tenant 2.

Note this important distinction, because although rare, it does appear.

Posted by Sean Silverman at 9:38 PM 0 comments


Email ThisBlogThis!Share to TwitterShare to FacebookShare to Pinterest
Labels: Real Property
Tuesday, November 19, 2013
The Top Five: Miranda Warnings
Five things you should know regarding the 5th Amendment right to Miranda warnings for
purposes of the MBE:

(1): Miranda warnings are only required once a person is in custody and knows that he is being
interrogated by a government agent. You should aim to understand the custody requirement well.
Whether a person is in custody depends on whether the person's freedom of action is denied in a
significant way, and this is an objective standard. In other words, the question to ask is whether a
reasonable person would believe that he is not free to leave.

(2): Miranda warnings are only required prior to interrogation by a government agent.
Interrogation includes both words and conduct that the agent should know would likely is likely
to elicit a response by the accused.

(3): The right to receive Miranda warning can be waived, but the waiver must be knowing,
voluntary, and intelligent.

(4): If after receiving Miranda warnings, a suspect claims that he would like to remain silent, the
police must honor that request, but may later question the suspect on an unrelated crime,
provided the suspect has not requested to remain silent regarding that unrelated crime as well.

(5): If the police fail to provide warnings and the suspect makes a statement, the 5th Amendment
prevents that statement from getting admitted into evidence for substantive purposes. But note,
importantly, that the statement is not excluded for all purposes in court. Specifically, the
statement may be offered to impeach a witness (For example, if the statement given in violation
of the 5th Amendment contradicts a previous statement made a witness, the later statement can
be offered into evidence to impeach based on prior inconsistent statements).

Posted by Sean Silverman at 10:01 PM 0 comments


Email ThisBlogThis!Share to TwitterShare to FacebookShare to Pinterest
Labels: Evidence
Sunday, November 3, 2013
The Top Five: Fixtures
Five things to know about fixtures for purposes of the MBE:

(1) A fixture is a piece of personal property that has been so affixed to land that it is no longer
considered personal property. The significance of a fixture is that it passes along with ownership
of the land.

(2) It's often difficult to determine whether an item of personal property should be considered a
fixture. The test to apply is an objective one; namely, whether the objective intention of the party
who made the annexation was to make the item a part of the real property. When deciding
whether this objective test has been satisfied, consider the nature of the article, the manner of
attachment, and the amount of damage that would be caused by removal of the object.

(3) A commonly tested MBE area concerns constructive annexation. Constructive annexation
occurs when an article, though not physically annexed to the property, is still considered a fixture
if it is a necessary aspect of the realty. (For example if removing custom-made curtain rods
would disrupt the design of a room, the curtain rods may be deemed a fixture even though they
have not been affixed to the property.)

(4) Between landlord and tenant, first determine if there has been an express agreement
regarding fixtures, as that will control. Absent an agreement, a tenant is deemed to lack the intent
to permanently improve the property, and may therefore remove any personal property that has
been attached to the real property provided that doing so will not damage the real property. Note
that the personal property must be removed by the end of the lease term (or if the tenancy is
indefinite, within a reasonable time after termination of the tenancy). Any damage caused by
removal of the property must be paid for by the tenant.

(5) Situations arise in which a landowner affixes a piece of personal property to real property,
and the seller of the personal property retains a security interest on that property while at the
same time a mortgagee retains a security interest (a mortgage) on the real property. If the
landowner defaults on both loans, it must be determined who, among the mortgagee and the
seller of the personal property, has priority. In such a situation, the seller has priority (and can
therefore collect first on his security interest), provided he records his security interest with a
"fixture filing" within 20 days after the personal property is affixed to the land. The seller must,
however, compensate the mortgagee if removing the personal property causes damage to the real
property.

The Top Five: The Best Evidence Rule


The Best Evidence Rule is a rule of evidence that requires that to prove the terms of a writing,
the original writing must be offered into evidence. Only upon proof that the original (or a
duplicate of the original) is unavailable will secondary evidence be admissible.

Five things to know about The Best Evidence Rule:

(1): The term writing is far broader than what might be expected. Writing here includes a
document (for example, a contract), a recording, a photograph or an X-ray.

(2): The rule does not apply whenever one wants to testify about a document; rather, it applies
only in two situations. The first is when the document is a legally operative or dispositive
instrument, and the second is when knowledge of a witness concerning a fact of consequence
results from having read it in the writing.

(3): On a similar note, the rule does not apply where a fact to be proved has an existence
independent of the writing. For example, if a witness wants to testify for the purpose of proving
that a contract exists (rather than for the purpose of proving the contents of the contract), then the
Best Evidence Rule does not apply, and secondary evidence may be admitted even though the
contract itself is available.

(4): A photocopy is deemed to be a duplicate, and therefore treated the same as an original.
Copies made by hand, however, are deemed secondary, and may only be admitted if the original
is unavailable.

(5): If the original (or duplicate) cannot be produced, secondary evidence is allowable.
Secondary evidence can consist of anything from oral testimony to handwritten copies of an
original document. It's required, however, that a satisfactory explanation is provided as to why
the original is unavailable. Some potential explanations are that the original has been destroyed,
is outside the jurisdiction of the court, or is in the possession of an adversary who refuses to
produce it for purposes of evidence.

Posted by Sean Silverman at 4:04 PM 0 comments


Email ThisBlogThis!Share to TwitterShare to FacebookShare to Pinterest
Labels: Evidence
Saturday, October 12, 2013
Procedural Due Process
The following question was e-mailed to me by a reader of the blog:

"I have a good handle on the procedural due process, but I have a tough time memorizing which
process is required for each violation. Do you have any suggestions on how to approach this?"

Response:

It's true that the number of potential interests (liberty and property) that might be violated is large
and so rather than attempting to memorize the specific interests, it's probably best to understand
the rule that is used to determine which process is required, and then apply that rule to whatever
interest is provided in a given question. Memorizing the most common violations (such as
committing a person to a mental institution, or terminating a person's welfare benefits) can
certainly be beneficial if a question happens to address those specific situations, but as a backup,
definitely understand the following rule:

The type and extent of required procedures for violations of an interest are determined by
balancing the following three factors:

(1): The importance of the interest to the individual: The greater the importance of the interest,
the higher the burden that will be placed on the government to provide procedures that will
safeguard that interest.

(2): The value of specific procedural safeguards to that interest: The higher the value, the more
likely that such procedural safeguards will be required before a person's interest is terminated.

(3): The government interest in fiscal and administrative efficiency: This third factor should be
balanced against the previous two. Some procedural safeguards will put such a burden on the
government that they will hamper the ability of the government to function efficiently. The
greater the burden to the government, the less likely it is that such safeguards will be required.

If you are given a question on the MBE and you do not recall the process required when a
specific interest is violated, use the above rule to guide you in determining the process required.

The Top Five: Negligent Infliction of Emotional Distress


The top five things to know about negligent infliction of emotional distress:

(1): The duty of care is breached when defendant creates a foreseeable risk of physical injury to
plaintiff either by causing a threat of physical impact that leads to emotional distress or by
directly causing severe emotional distress that by itself is likely to result in physical symptoms.
Both will suffice.

(2): If liability is predicated on threat of physical injury, plaintiff must prove that he was in the
zone of danger when the threat occurred. This means that it was reasonable for plaintiff to
believe that he was in fact threatened with physical injury.

(3): Even a bystander who witnesses another being physically injured can recover if the
bystander is in the zone of danger when witnessing the injury. The bystander-plaintiff must
prove that he and the person injured are closely related, that he (the bystander) was present when
the injury occurred, and that he (the bystander) observed or perceived the injury.

(4): To recover for this tort, physical symptoms are usually required (unlike in intentional
infliction of emotional distress). But even though emotional distress alone will not suffice, if that
distress causes, for example, a shock to the nervous system, that alone will be sufficient.

(5): There are a few examples when physical symptoms in addition to emotional distress is not
required. These include when plaintiff receives an erroneous report of a relative's death, and
when the corpse of the plaintiff's relative is mishandled.
Posted by Sean Silverman at 11:01 AM 0 comments
Email ThisBlogThis!Share to TwitterShare to FacebookShare to Pinterest
Tuesday, September 10, 2013
The Duty of Care
After determining that a foreseeable plaintiff is owed a duty of care, the next step in any
negligence question is to determine whether defendant breached the duty of care. The standard
shifts under certain circumstances, making this a tricky element to analyze, but keeping the
following in mind should prove helpful.

There is a general standard of care, and then there are some exceptions to the general standard.
The general standard is the reasonable person standard. This is an objective standard, and
measures one’s conduct against how the average person would act under similar circumstances.
The “reasonable person” is considered to have the same physical characteristics of defendant, so
that physical characteristics are taken into account when determining if defendant did not act
reasonably, thereby breaching his duty of care. Generally, however, mental deficiencies or
inexperience is not taken into account.

Then there are the exceptions. Professionals are held to a higher standard of care in regards to
any act that is connected to their specialty. They are required to possess the knowledge and skill
of a member in good standing of the profession. Children are held to the standard of children
with like age, education, intelligence, and experience. Children involved in adult activities,
however, are generally held to the standard of a reasonable adult. In addition, common carriers
and innkeepers are held to a very high standard of care, so that the slightest breach will suffice.
In an emergency situation, all people are expected to act as a reasonable person would act under
those same emergency circumstances, but note that if defendant causes the emergency, the fact
that he is then placed into such a situation will not change the general standard of care (the
reasonable person standard).

Analyze as stated above, and once you've determined that a breach has occurred, start thinking
about causation.

MBE Fast Fact: Necessity


The defense of necessity in Torts is all about balancing harms, and on the MBE, it's a safe bet to
assume it'll show up in a question that requires you to determine if one had the privilege to enter
another's land. (Because if one has the privilege, then there is no trespass.)

A person has a privilege to interfere with the real or personal property of another when it is
reasonably and apparently necessary to avoid threatened injury from a natural or other force, and
when the threatened injury is substantially more serious than the invasion that is undertaken to
avert it. That last part is quite important and plays into the balance of harms. In order for
necessity to be a valid defense it must be true that the harm you were trying to avoid is
substantially greater than the harm you might cause in trying to avoid the other harm.

The two types of necessity are public and private necessity. Public necessity provides for the
defense when harm to the public is threatened whereas private necessity allows for the defense
when an act is solely to benefit a private individual or property belonging to that individual. An
important difference here is that even if one has the privilege of private necessity, the actor must
pay for any damages caused by the act resulting from the necessity. So, if X while flying his
private airplane is threatened with the possibility of a substantial injury, and to avert that injury,
X lands his plane on the property of Y, then, even if X had the privilege to land on Y's property,
X may still be liable if landing the plane causes any damage to y’s land.

Posted by Sean Silverman at 11:13 AM 0 comments


Email ThisBlogThis!Share to TwitterShare to FacebookShare to Pinterest
Labels: Torts
Saturday, August 24, 2013
Invasion of Privacy
There are four types of invasion of privacy in the subject of Torts that show up on the MBE.
Each of the four tend to test specific aspects, and these are the points that should be kept in mind
going into the exam:

Appropriation of Plaintiff's Picture or Name: The use of the picture or name must be
unauthorized and must be for defendant's commercial advantage. In other words, defendant must
use plaintiff's picture or name intending to benefit commercially from the use (for example, by
promoting a product).

Intrusion upon Plaintiff's Seclusion: Here, you should examine whether the intrusion would be
objectionable to a reasonable person. Note carefully that this is an objective test so that if
plaintiff is excessively sensitive but a reasonable person would not have been offended, then the
elements of this tort have not been satisfied. In addition, photographs taken of plaintiff in a
public place will not form a basis for this tort; rather, the place into which there is an intrusion
must be private.

Publication of Facts Placing Plaintiff in False Light: False light exists where one attributes to
plaintiff views he does not hold or actions he did not take. As with intrusion into seclusion, this
type of invasion of privacy requires that the false light be something objectionable to a
reasonable person. As with defamation, for liability there must have been publication
(communication of the facts constituting false light) to someone other than plaintiff.

Publication Disclosure of Private Facts about Plaintiff: As with the above, the public
disclosure must be objectionable to a reasonable person. It's important to note that public
disclosure of private facts, even if those facts are true, can form the basis for a valid cause of
action, making this an option when the elements of defamation have not been satisfied.

Posted by Sean Silverman at 8:45 PM 2 comments


Email ThisBlogThis!Share to TwitterShare to FacebookShare to Pinterest
Labels: Torts
Monday, August 19, 2013
The Top Five: False Imprisonment
The tort of false imprisonment requires an act or omission on the part of defendant that confines
plaintiff to a bounded area with intent to confine and causation. The top five things to remember
about the tort of false imprisonment for MBE purposes are as follows::

(1): It is irrelevant if the period of confinement is extremely short. The length of confinement
may need to be analyzed for another purpose (for example to determine if plaintiff was harmed)
but the time, in itself, is not a consideration in the analysis of the tort.

(2): Plaintiff need not know that he/she is confined. But if plaintiff does not know of the
confinement then a valid cause of action requires that plaintiff is harmed by the confinement. If
plaintiff does know of the confinement, then that alone will suffice, even if the confinement
causes no other harm.

(3): Moral pressure and/or future threats are insufficient acts of restraint. In other words, these
alone will not provide a basis for a claim of false imprisonment.

(4): Sufficient acts of restraint include the following: physical barriers, physical force (as
opposed to future threats of physical force), present threats of force, failure to release plaintiff
from a bounded area, and invalid use of legal authority.

(5): For purposes of this tort, a bounded area requires that freedom of movement must be limited
in all directions. There must be no reasonable means of escape.

Posted by Sean Silverman at 6:50 PM 0 comments


Email ThisBlogThis!Share to TwitterShare to FacebookShare to Pinterest
Labels: Torts
Tuesday, August 13, 2013
The Rescue Doctrine
According to the subject matter outline created by the National Conference of Bar Examiners,
half of the Torts questions on the MBE will be on the topic of Negligence. It's definitely a topic
you'll want to know very well when taking the exam. An initial consideration when analyzing
negligence is to determine whether a duty of care is owed to the plaintiff (the injured party in the
fact pattern).

The key concept to note is that a duty of care is only owed to foreseeable plaintiffs. Determining
whether a plaintiff is foreseeable requires asking whether a reasonable person would have
foreseen a risk of injury to the plaintiff at the time that plaintiff was injured. In other words if an
act by x causes an injury to y, but a reasonable person in the position of x would not have
foreseen that y would be injured as a result of the actions by x, then x will not be liable for y's
injuries on a theory of negligence.

On the MBE, one area in which you'll often be required to apply the above is when rescuers enter
the scene. For example, assume that x breaches a duty of care and injures y as a result. Z,
spotting an injured y, attempts to help y, and in the process z is injured. Determining whether x is
liable to z requires a determination as to whether a reasonable person in the position of x would
have foreseen at the time that he injured y, that someone might attempt to assist y and in the
process get injured.

It's been famously stated by Justice Cardozo that "danger invites rescue." This rescue doctrine
applies on the MBE, and, as such, because danger invites rescue, if one places another in danger,
one should foresee that such danger will invite a rescuer to the scene, and if the rescuer is
subsequently injured as a result of attempting to assist, the original tortfeasor may be liable both
to the original victim, and to the rescuer who attempted to assist.

Posted by Sean Silverman at 7:06 PM 0 comments


Email ThisBlogThis!Share to TwitterShare to FacebookShare to Pinterest
Labels: Torts
Saturday, August 10, 2013
Felony Murder: A Checklist
Of the 4 types of malice aforethought that will form the basis for a valid charge of murder on the
MBE, the intent to commit an inherently dangerous felony (ie, felony murder) tends to be the
trickiest. Keep the following important points in mind when faced with a felony murder fact
pattern.

--The felony forming the basis of a charge of felony murder must be one that is inherently
dangerous. The common felonies to look for are burglary, arson, robbery, rape and kidnapping.

--The defendant must be guilty of the underlying felony to be guilty of felony murder.

--The felony must be distinct from the killing itself. So, if during the commission of a felony,
defendant points a gun at a store clerk and kills the clerk, that should be analyzed under intent to
kill murder, rather than felony murder.

--The death must have been caused before the defendant's immediate flight from the felony.
Once the defendant has reached a place of temporary safety, you should no longer analyze
subsequent killings under felony murder (unless, of course, defendant commits a later felony).

--The agency theory states that defendant is not liable for felony murder when an innocent party
is killed during the commission of the felony unless the killer is the defendant or an agent of the
defendant. (So, for example, defendant would not be liable if, while robbing a bank, a victim
shoots at defendant and accidentally kills an innocent party.)

--The proximate cause theory broadens the scope of liability, making defendant liable even if an
innocent party kills another, if the killing is a proximate cause of the felony.

--The Redline theory states that defendant is not liable if a co-felon is killed during the
commission of the felony. (So, for example if defendant 1 and defendant 2 rob a bank, and a
victim kills defendant 2 during the commission of the felony, defendant 1 will not be charged
with the murder of defendant 2.

**Note that the three above theories vary by jurisdiction, so apply the theory stated in the fact
pattern if any theory is provided within a given question.
Posted by Sean Silverman at 12:51 PM 0 comments
Email ThisBlogThis!Share to TwitterShare to FacebookShare to Pinterest
Labels: Criminal Law
Monday, August 5, 2013
MBE Fast Fact: Mistake of Law
One of the most difficult areas of Criminal Law is the defenses. It's easy to confuse mistake of
fact with factual impossibility, and mistake of law with legal impossibility. One way to avoid
this confusion is understand that factual impossibility and legal impossibility will be raised as a
defense only if defendant fails to complete the crime. In contrast, mistake of fact and mistake of
law will be raised as a defense only after defendant has completed the crime.

There is a saying that ignorance of the law is no excuse. For the most part, that is correct, but
there are exceptions. (It seems as though there always are.) Specifically, in regards to the defense
of mistake of law (where the defendant claims he believed his actions were legal though they
were in fact illegal), it's important to note that this is generally not a defense even if the belief
was reasonable and based on the advice of an attorney.

There is an important exception here, however. Mistake of law will be valid if the statute
proscribing defendant's conduct was not published or made reasonably available prior to the
conduct for which defendant has been charged, or if defendant reasonably relied on a statute or
judicial decision that indicated his conduct was legal. The key here is certainly the word
"reasonable." This is an objective test, so that it won't be enough for defendant to claim he
subjectively relied on a statute resulting in his mistake; it must be true that a reasonable person
would have made the same mistake.

MBE Fast Fact: Judgment Liens


Questions concerning judgment liens have been showing up more often on the MBE. If you are
sued in court for a sum of money and lose the case, the prevailing party will be granted a
judgment. That party may then file a judgment lien, which is a lien that attaches to your real
estate.

Once a judgment lien is placed on the real estate, this acts as assurance that the money that was
the subject of the judgment will be paid to the creditor. On the MBE, these liens are placed on
property already owned, and sometimes also on property acquired within a given period of time
in the future. (These after-acquired clauses are quite common.)

Generally, the priority of a judgment lien is determined by its recording date. So, if the judgment
lien is filed on property, and later there is a mortgage taken out on that same property, the
judgment lien will be senior to the mortgage, and therefore the judgment creditor will have
priority over the mortgagee.

But the rules change for purchase-money mortgages. A purchase-money mortgage ("PMM") is a
mortgage to secure part or all of the purchase price of the property mortgaged. The mortgage can
be given by the buyer to the seller of the property or to a third-party, such as a bank. It is a
general rule that a PMM is entitled to preference (or priority) over all liens, even those liens
arising earlier than the PMM. The rationale behind the rule is that even if the judgment lien was
one that contained an after-acquired clause, the property that was the subject of the PMM cannot
be deemed to have been acquired by the buyer, because by taking out the mortgage to purchase
the property, the buyer merely obtained equitable title, subject to his obligation to re-pay the loan
to the legal title holder (the mortgagee). Because the property was not later acquired by the
buyer, it is not subject to the judgment lien, and the purchase-money mortgagee retains priority.

Privity of Estate/Privity of Contract


Analyzing assignment questions in the landlord/tenant context requires an understanding of
privity of estate and privity of contract. In fact, these concepts are essential to answering such
questions correctly.

Let's assume that landlord (x) leases property to tenant (y). The lease is a contract, so x and y are
in privity of contract. In addition, a lease of property gives rise to a privity of estate, so x and y
are in privity of estate. These privities cause legal obligations from x->y, and from y->x. As an
example, y owes rent to x, and x can collect this rent because of the privity of contract and
privity of estate that he has with y.

Now let's assume that y assigns his lease with x to an assignee (z). Z never directly contracted
with x, so z and x are not in privity of contract. But because z is now the assignee of the lease, z
is in privity of estate with x. This allows x to collect rent from z. And although y is no longer in
privity of estate with x (because y assigned his lease to z) y is still in privity of contract with x.
This allows x to collect rent from y, if x first tries unsuccessfully to collect rent from z.

And now let's assume that z further assigns the lease to a second assignee (t). Z was never in
privity of contract with x, and z lost his privity of estate with x when he assigned his lease to t.
Therefore because z is in neither privity of estate nor privity of contract with x, z no longer has
any obligations towards x. T, however, now has an obligation towards x, based upon his privity
of estate with x. And if t fails to pay, x can still go after y because they are still in privity of
contract.

Know the privities well, as they will lead you to the correct answer, if analyzed properly.

Posted by Sean Silverman at 11:29 AM 0 comments


Email ThisBlogThis!Share to TwitterShare to FacebookShare to Pinterest
Labels: Real Property
Monday, July 1, 2013
MBE Fast Fact: Landlord/Tenant Law
Three tenancies (landlord/tenant law) you'll be required to distinguish between when taking the
MBE are the tenancy for years, the periodic tenancy, and the tenancy at will. The relevant
characteristics of each are below:

Tenancy for years:

It’s important to note that a tenancy for years continues for a fixed period of time. That time
could differ, but it will always be fixed. If the amount of time is greater than a year, then the
statute of frauds requires a writing to create a this type of tenancy. It ends automatically upon the
expiration of the stated time, but the tenant can surrender the tenancy prior to the expiration of
the stated time. The landlord must accept such surrender, and the surrender must be in writing if
more than one year remains on the lease.
Periodic Tenancy:

A periodic tenancy continues for successive periods (for example month to month). This type of
tenancy can only be terminated upon notice by either party. In addition, the tenancy will be
renewed automatically if notice is not provided. There are no bright-line rules here, but usually
the notice must be one full term so that if it’s a month-to-month tenancy, one month of notice is
required. A year-to-year tenancy, however, usually required only 6-months notice under the
common law.

Tenancy at Will:

The most important characteristic of this type of tenancy is that it is terminable at the will of
either the landlord or tenant. No notice on either end is required. An express agreement that the
tenancy is terminable at will is needed, otherwise the courts will treat the tenancy as a periodic
tenancy requiring notice to terminate.
Posted by Sean Silverman at 1:54 PM 0 comments
Email ThisBlogThis!Share to TwitterShare to FacebookShare to Pinterest
Labels: Real Property
Friday, June 28, 2013
Tenancy at Sufferance
The following question was asked by a reader of the blog:

The questions sometimes make distinctions between residential and nonresidential tenants with
the hold over doctrine. What should we know about this?

Response:

There are a bunch of tenancies to know in regards to landlord-tenant law, and one of them is a
tenancy at sufferance. A tenancy at sufferance arises when a tenant wrongfully remains in
possession after the expiration of the tenancy. In this context, the tenant is deemed to be holding
over, so that the hold-over doctrine applies.

The hold-over doctrine provides that a tenancy at sufferance will be terminated when the
landlord takes steps to evict the tenant. The landlord has a few options here, however. The
landlord can evict the tenant, or bind the tenant to a new periodic tenancy. If the
original nonresidential tenancy was for a shorter duration than year-to-year, then, if the
landlord chooses to bind the nonresidential tenant to a new periodic tenancy, the tenant will be
bound by the frequency of rent payments in the previous tenancy. For example, if the previous
tenancy was month-to-month, the tenant will be bound by a new month-to-month tenancy. Note
that if the question presents a nonresidential tenant, and the tenancy was for year-to-year or
longer, that tenant can be bound by a new year-to-year tenancy. In contrast, a residential
tenant who remains in possession after the expiration of the least will be bound only by a
month-to-month tenancy at most (if the landlord chooses not to evict,) regardless of the terms of
the previous tenancy.

The tenant (both residential and nonresidential) also assumes the risk that the rent will increase
for the new tenancy.
Posted by Sean Silverman at 10:46 AM 0 comments
Email ThisBlogThis!Share to TwitterShare to FacebookShare to Pinterest
Sunday, June 23, 2013
MBE Fast Fact: Life Tenants
The rights and obligations of a life tenant (in regards to that owed to a future interest holder such
as a remainderman), is a commonly tested area with Real Property questions.

In regards to rights, it's very straight-forward. A life tenant is entitled to ordinary uses and profits
of the land. The obligations that a life tenant has to a remainderman is a bit more complicated,
but a general rule to keep in mind is that a life tenant cannot do anything that might injure
someone holding a remainder. More specifically, a life tenant may not exploit the land (for
example, by removing natural resources) unless doing so is necessary for repair or maintenance
of the land, the land is suitable only for such use, or doing so has been expressly or impliedly
allowed by the grantor. The "open mines doctrine" states that if mining was done on the property
prior to the life tenant taking possession, then that life tenant can continue to mine but is limited
to the mines already open.

In addition, a life tenant must keep the land in a reasonable state of repair, pay interest on
mortgages (but need not pay the principal on those mortgages,) and pay ordinary taxes on the
land. Note that a life tenant is not obligated to insure the property for the benefit of the
remaindermen, and if a third party tortfeasor were to damage the property (for example a
trespasser), the life tenant would not be responsible for that damage.

Even if the life tenant were to improve the property (known as ameliorative waste), the change
was actionable at common law. Today, a life tenant may commit ameliorative waste if the
market value of the future interest is not diminished, or if either the remainderman does not
object to the change, or a substantial and permanent change in the neighborhood conditions has
deprived the property in its current form of reasonable productivity or usefulness, necessitating
the change.

Posted by Sean Silverman at 11:46 AM 0 comments


Email ThisBlogThis!Share to TwitterShare to FacebookShare to Pinterest
Labels: Real Property
Thursday, June 20, 2013
Fee Simple
The following question was e-mailed to me by a reader of the blog:

Sean,

I am having trouble identifying whether something is a fee simple subject to a condition


subsequent, fee simple determinable or fee simple subject to an executory interest. I know the
definitions but they all seem so similar. Can you explain this better?

Thanks

Response:

Future interests are probably the most cited area of difficulty in regards to Real Property. I think
the best way to address this question is to first deal with the fee simple determinable and fee
simple subject to condition subsequent, as there are important similarities and differences among
them that must be known.
A fee simple determinable is a present possessory estate that terminates upon the happening of a
stated event. Importantly, once that event occurs, the estate automatically reverts back to the
grantor. For example, x conveys to y "Blackacre for so long as y uses the property for farming
purposes." In that conveyance, the stated event is the failure to use the property for farming, and
upon that stated event, Blackacre will revert right back to x without x having to take any action
to retrieve it. Note that here x has an interest in the property because the property might revert
back to him upon the happening of the stated event. It's a future interest, and it's called a
possibility of reverter.

A fee simple subject to condition subsequent, however, does not automatically revert back to the
grantor upon the happening of a stated event. It is, like the above, a present possessory estate that
terminates upon the happening of a stated event. For example, x conveys to y "Blackacre but if y
fails to use the property for farming purposes then x reserves the right to re-enter the property to
retrieve it." The differences are significant here; though both grants speak of the same event
(failure to use the property for farming), the difference lies in what will result should that event
come to pass. If a fee simple determinable was granted to y, then, as stated above, the property
would automatically revert back to x, but if a fee simple subject to condition subsequent was
granted, then x is under an obligation to re-enter the property to retrieve it. If he does not, then
the property will remain with y. The interest that x has here is a future interest known as a right
of re-entry (sometimes called a right of termination).

A fee simple subject to an executory interest is similar to the two fee simples already mentioned
above, in that the present estate will terminate upon the happening of a stated event. For
example, x conveys to y "Blackacre for so long as y uses the property for farming purposes, and
then to z." Notice the addition of z here. That additional party is what makes this estate different
than those previously mentioned. With the previous two, the future interest was held by the
grantor, whereas here the future interest is held by a third party (z). Similar to the fee simple
determinable, upon the happening of that event, the interest will automatically pass to z without z
being obligated to affirmatively retrieve it. The future interest held by z here is called an
executory interest.

As an aside, as it's also a topic that causes confusion, remainders differ from the future interests
mentioned above. Specifically, remainders do not become possessory until the natural
termination of the preceding estate, whereas in the three examples above, the future interest
would not become possessory upon the natural termination of the preceding estate, but rather
would become possessory upon the happening of a stated event that would cut off the preceding
estate.

MBE Fast Fact: Right to Counsel/Right to a Jury Trial


The rules regarding right to counsel during a criminal trial and right to a jury trial in a criminal
case are straight-forward, but it's also easy to mix up the different standards. Below are the rules
that should be known regarding these two rights on the MBE:

Right to Counsel: Defendant has a right to counsel during felony trials. In regards to
misdemeanors, defendant will only have a right to counsel if incarceration is actually imposed or
if a suspended sentence is imposed. What this means is that in a misdemeanor trial, if defendant
is convicted and sentenced to incarceration, defendant can attack that conviction if counsel was
not provided. But if incarceration is not imposed, then defendant cannot attack the judgement
based upon lack of counsel.

Note also that defendant is entitled to counsel when pleading guilty.

Right to Jury Trial: The right to a jury trial is only afforded for serious offenses. Serious
offenses are defined as offenses in which imprisonment of more than 6 months is authorized
(even if not imposed). A petty offense is one in which imprisonment of more than 6 months is
not authorized, and there is no right to a jury trial for such offenses. Even in those cases in which
a jury trial is required, there is no right to a jury of 12 but there must be at least 6 jurors to satisfy
the right to jury trial. Though not all verdicts must be unanimous, all verdicts of a six-person jury
must be unanimous.

There is also no right to a jury trial in juvenile delinquency proceedings.

Posted by Sean Silverman at 8:32 PM 0 comments


Email ThisBlogThis!Share to TwitterShare to FacebookShare to Pinterest
Labels: Criminal Procedure
Saturday, June 1, 2013
MBE Fast Fact: Rights and Obligations of a Life Tenant
Let's assume that x grants a life-estate to y with a remainder to z. Questions appear on the MBE
requiring knowledge as to which duties, if any, y (as the holder of a life-estate) owes to z (as the
holder of a remainder.)

Before addressing that, it's helpful to understand the rights of y. Y, as a holder of a life-estate, is
entitled to any ordinary use and profits of the land. Put another way, as long as y's use of the land
is reasonable, there will be no liability from y towards z. And that said, it's important to know
what might constitute an unreasonable use of the land.

Y cannot do anything that might injure the interests of the person holding the remainder (here, z),
or anything that might injure a person holding a reversion (here the grantor, x). The most
common types of unreasonable use that show up on the MBE are the three types of waste:
affirmative waste, permissive waste, and ameliorative waste.

If, in the example above, y were to create any of the three types of waste (for example,
permissive waste, if y fails to repair the land in situations in which he is obligated to do so) then
z would have the option of either suing y for damages (a remedy at law), or enjoining the act
constituting waste (a remedy at equity).

MBE Fast Fact: Informants


The MBE writers can go a few ways when testing how informants influence the validity of a
search warrant. In other words, warrants are issued if there is probable cause to believe that
evidence will be found on the person or premises at the time the warrant is executed. An affidavit
is submitted by the police to the magistrate setting forth circumstances allowing the magistrate to
determine if this standard has been satisfied. But what if the information on the affidavit is
entirely the result of information learned from an informant, and one whose credibility is
questioned?

In fact, an affidavit can be sufficient even though the informer’s credibility is questioned,
provided that the totality of the circumstances indicate that the affidavit presents reliable and
credible information.

Note also that there is no requirement that the informer’s identity be revealed.
Posted by Sean Silverman at 6:47 PM 0 comments
Email ThisBlogThis!Share to TwitterShare to FacebookShare to Pinterest
Labels: Criminal Procedure
Saturday, May 18, 2013
MBE Fast Fact: Fruit of the Poisonous Tree
The "Fruit of the Poisonous Tree" doctrine makes it possible for the exclusionary rule to not only
exclude evidence obtained in violation of the Fourth, Fifth, and Sixth Amendments, but also to
exclude evidence obtained from exploitation of the unconstitutionally obtained evidence. To
extend the symbolism, not only is the tree excluded (the evidence obtained in violation of the
Amendments), but the fruit of that tree (evidence that would not have been obtained had the
illegally obtained evidence not been obtained) is also excluded.

For MBE purposes, knowing when this doctrine does not apply is just as important as
understanding the doctrine itself. The doctrine does not apply to evidence derived in violation of
Miranda, evidence obtained from a source independent of the illegally obtained evidence,
evidence obtained as a result of an intervening act of free will by the defendant, evidence that the
police would have discovered regardless of the illegally obtained evidence (often deemed the
"inevitable discovery doctrine,) and evidence obtained in violation of the “knock and announce
rule.”
Posted by Sean Silverman at 2:28 PM 0 comments
Email ThisBlogThis!Share to TwitterShare to FacebookShare to Pinterest
Thursday, May 16, 2013
A Reasonable Expectation of Privacy (4th Amendment)
An initial consideration in any 4th Amendment issue is to determine if a person has a reasonable
expectation of privacy. Without first analyzing that consideration, it's easy to fall into the trap of
analyzing other considerations that require no analysis. If a person does not have a reasonable
expectation of privacy, then that person is not protected by the 4th Amendment, so a full 4th
Amendment analysis would be unnecessary.

This is not an uncommon tactic by the writers of the MBE. You'll read a fact pattern and
overlook one line which will then take you down the wrong road, wasting time, and eventually
leading to the wrong answer. That said, you should know well when the 4th Amendment does
not apply.

A person does not have a reasonable expectation of privacy in the following:

The sound of one's voice

One's handwriting
Paint on the outside of one's vehicle

Garbage left for collection

Areas outside the home (often referred to as curtilage)

Account records held by a bank

The smell of one's car or luggage

Land visible from a public place.

What the above means is that the government can "search" these areas without having to concern
itself with the warrant requirement that is implicated when the 4th Amendment does apply.

Posted by Sean Silverman at 9:09 AM 0 comments


Email ThisBlogThis!Share to TwitterShare to FacebookShare to Pinterest
Labels: Criminal Procedure
Monday, May 13, 2013
MBE Fast Fact: Present Bodily Condition vs. Past Bodily Condition
Statements about one's present bodily condition and statements about one's past bodily condition
have the potential to be admitted into evidence even if those statements were made out of court,
and are offered for the truth of the matter asserted. In other words, they are both potential
exceptions to the rule against hearsay.

But there is an important distinction to keep in mind here. A declaration of a present bodily
condition is admissible even though that declaration is not made to a medical professional, while
a declaration of a past physical condition is admissible only if made to a medical professional to
assist in diagnosing or treating the condition. Be sure to note this distinction, and also watch out
for those questions in which someone makes a statement to a physician about a past bodily
condition, but the statement is not made for the purpose of diagnosis or treatment. Those
statements are excluded, as they are not covered by this exception.

**Need additional help learning Evidence? My resource MBE Essentials: Evidence contains the
essential information tested in Evidence on the MBE.

Posted by Sean Silverman at 12:38 PM 0 comments


Email ThisBlogThis!Share to TwitterShare to FacebookShare to Pinterest
Labels: Evidence
Wednesday, May 8, 2013
Cross Examination vs. Extrinsic Evidence
I've written a bit on the blog about impeaching witnesses. In addition to knowing the rules
regarding when to impeach a witness (for example, if the witness has committed a felony, made
a prior inconsistent statement, etc.), it's also important to know how the witness may be
impeached. The two possibilities are by cross examination and extrinsic evidence. (Extrinsic
evidence is evidence other than that elicited by cross-examination, such as calling a witness to
testify about another witness, or offering a copy of a judgement to prove that a witness has been
convicted of a felony.)

Below is a list to help keep the distinctions in mind:

Prior Inconsistent Statements: Cross Examination and extrinsic evidence are allowed (but no
extrinsic evidence to prove a prior inconsistent statement if the inconsistent statement is a
collateral matter). Note also, that if extrinsic evidence is offered here, the witness (unless it's a
hearsay declarant) must at some point be provided the opportunity to explain or deny the
inconsistent statement.

Bias: Cross examination and extrinsic evidence are allowed. Before extrinsic evidence is offered,
the witness must first be asked on cross examination about the facts showing bias.

Conviction of a Crime: Cross examination and extrinsic evidence (a copy of the judgement) are
allowed.

Specific Instances of Misconduct: Cross examination only is allowed.

Opinion or Reputation for Truthfulness: Extrinsic evidence is allowed (calling other


witnesses to testify to the reputation or give an opinion about another witness).

Sensory Deficiencies: Cross examination and extrinsic evidence are allowed.

**Need additional help learning Evidence? My resource MBE Essentials: Evidence contains the
essential information tested in Evidence on the MBE.

Posted by Sean Silverman at 10:05 PM 0 comments


Email ThisBlogThis!Share to TwitterShare to FacebookShare to Pinterest
Labels: Evidence
Monday, May 6, 2013
Expert Witnesses
A few points to know about experts, as you study Evidence in preparation for the MBE:

Questions arise as to whether a witness is allowed to provide an opinion regarding a subject-


matter that requires specialized knowledge. The Federal Rules of Evidence does not allow for
such testimony by a lay witness, but may allow such testimony by an expert. So, then, who is an
expert, and when is such testimony allowable?

An expert is one who possesses specialized knowledge, skill, experience, training, and education.
If the subject-matter is one where scientific, technical or other specialized knowledge would
assist the trier of fact, then an expert may be allowed to provide opinion testimony, provided that
the witness has been qualified as an expert, the expert possesses reasonable probability regarding
his opinion, and the opinion is supported by a proper factual basis. The factual basis will be
proper if it is based on the expert’s personal observation, facts made known to the expert at trial,
or facts not known personally but supplied to the expert outside the courtroom and of a type
reasonably relied upon by experts in the particular field.

Also note that other commonly tested questions within this area deal with the cross-examination
of experts. If an expert has, on direct examination, established as reliable authority a certain
publication, the expert may be cross-examined as to any statement contained in that publication.
Even if the expert has not personally established such facts as reliable, the statements can still be
used to cross-examine the expert if another expert established the reliability, or if the court
chooses to take judicial notice of the reliability. These statements may be used not only to
impeach the expert, but also as evidence of the truth of the matter asserted, as these statements
fall within an exception to the rule against hearsay.

MBE Fast Fact: Impossibility vs. Impracticability


When there is a fine distinction, it's likely it'll be tested on the MBE. One such distinction is that
between impossibility and impracticability, both of which can be used to discharge a contractual
obligation.

An important distinction here is between the standards of subjectivity and objectivity. When
assessing impossibility, a contractual obligation can be discharged if the obligation is deemed
impossible to perform, using an objective standard. In other words, it must be true
that nobody could perform according to the terms of the contract(for example, if the subject
matter of the contract has been destroyed). The impossibility must arise after the contract has
been entered into.

In contrast, impracticability is measured by a subjective standard. Rather than asking whether


anyone could perform according to the terms of the contract, the proper analysis is to determine
whether an extreme and unreasonable difficulty has arisen that was not anticipated by the party
claiming impracticability, so that the party is subjectively unable to perform according to the
terms of the contract. It's important to note that although a subjective test is proper here, it's not
enough that performing under the contract has been rendered more difficult, if those difficulties
were anticipated prior to entering into the contract.
MBE Fast Fact: The Rule of Completeness
The "Rule of Completeness" shows up on occasion on the MBE. The rule should be considered
whenever a party introduces a writing (or recording) into evidence. Once that party has
introduced any part of the writing, the adverse party may require the party to introduce any other
part of that writing which in fairness ought to be considered along with the part that was
introduced.

Another way to think about this rule is that the party that introduced a part of a document, merely
by introducing it, has waived the right to object to introduction of other parts of that same
document by the opposing party.
MBE Fast Fact: The Spending Power
There are only a few things to know for the MBE about Congress's spending power. First, the
power of Congress to spend (and tax) can be exercised only for the general welfare of the United
States. A very common question appearing on the MBE is one in which Congress requires states
to comply with specified conditions in order to qualify for federal funding. The fact that
Congress has offered the funding is allowable, but the issue is whether it can condition receipt of
the funding on compliance. The rule to note is that Congress can require states to comply in
order to receive the funding. For example, Congress can withhold federal highway funds from
any state that refuses to enact a law permitting the purchasing of alcoholic beverages by only
those over the age of 21.

An important distinction to note is where Congress attempts to compel the states to enforce
federal regulations. Though, as stated above, Congress can encourage the states to adopt certain
federal regulations through the spending power (by attaching conditions to the receipt of federal
funds), or through the commerce power (by directly pre-empting state law), Congress cannot
directly compel states to enforce federal regulations, as this would violate the 10th Amendment.

Miranda Exceptions
A lot has been said on the news this week regarding the public safety exception to the Miranda
rule. And though this exception is important to know for the MBE, there are other exceptions
you should be aware of as well.

First the rule: Miranda warnings (if no exceptions apply) must be provided prior to any statement
made during "custodial interrogation." A suspect is in custody when, by an objective standard,
the suspect is not free to leave. If a suspect while in custody is interrogated without having been
read Miranda warnings, those statements may be excluded from evidence during trial as per the
exclusionary rule.

The exceptions that must be kept in mind as to when Miranda warnings are not required are as
follows:

(1) Questioning by private security guards (as opposed to the government).

(2) Statements made before a grand jury.

(3) On the scene questioning if it's unreasonable for a suspect to believe that he/she is not free to
leave the scene.

(4) Spontaneous, unsolicited statements, as these statements would not be deemed a consequence
of interrogation. These are voluntary statements, and these statements will not be excluded for
lack of Miranda warnings.

(5) Interrogation of a taxpayer during a criminal investigation by the I.R.S.

(6) When the right to receive the warnings is outweighed by the immediate threat posed to the
public safety.

(7) Routine booking questions by the police such as age, date of birth, height, and weight.
Burglary Elements
Burglary under the common law had some interesting elements. The breaking and entering of the
dwelling (and it had to be a dwelling) of another had to take place in the nighttime (strange) with
an intent to commit a felony therein. A very commonly tested element is the requirement that
there be an intent to commit a felony prior to, or at the time of, the breaking and entering. Watch
carefully for those questions where "x" breaks into another's home (or often the garage) in an
attempt to take back something that x believes to be his own. If you are attempting to retrieve
your own property, you do not intend to commit a felony, and there can be no burglary. Of
course, once x is in the dwelling, he'll see something that he likes and decide to take it along with
whatever it is he originally entered to retrieve. That is still not burglary, because the intent to
commit the felony cannot be formed after the breaking and entering has already taken place.

But be careful; don't forget about larceny.

The Insanity Defense


It's important to know the various tests used to determine a defendant's sanity, because the MBE
can ask you to apply any of them to a given fact pattern. Though similar, each has its own
distinctions.

A majority of jurisdictions follow the M'Naghten test. Under this test, a defendant will be
relieved from criminal liability upon proof that at the time of the commission of the criminal act,
a disease of the mind prevented him from knowing the nature and quality of the act, or if he did
know the nature and quality of the act, the disease of the mind prevented him from knowing that
the act was wrong.

As with the M'Naghten test, the Irresistible Impulse test also requires a disease of the mind.
The distinction to note is that under this test, a defendant will be relieved of liability if the
disease prevented him from controlling his conduct. Note that under this test, it's irrelevant if
defendant understood the nature of his act, or if he understood that his act was wrong. Even if he
understood such things, liability will be relieved if defendant, knowing that what he was doing
was wrong, could not prevent himself from doing it.

The Durham test is quite broad and merely states that defendant is not criminally liable if his
unlawful act was the product of a mental disease or defect. The proper test to apply here is a "but
for" test; namely, a crime is a product of a mental disease or defect if the crime would not have
been committed but for the disease or the defect.

Finally, there is a test from the Model Penal Code. This test shows up more often than the
others, though for this test to apply, the fact pattern will have to advise you to apply the Model
Penal Code. Under this test, a person is not responsible for criminal conduct, if, at the time of the
act, as a result of a mental disease or defect, the person lacked substantial capacity to appreciate
the criminality of the act, or lacked the ability to conform his conduct to the requirements of the
law.

Essentially, the test required under the Model Penal Code allows for a defense of insanity if
either the M'Naghten test, or the Irresistible Impulse test would have been satisfied.
False Imprisonment
General Rule:

The general rule to note about false imprisonment on the MBE is that a person is subject to
liability to another for false imprisonment if (1): he acts intending to confine a person within
fixed boundaries, and his act directly results in such confinement, and (2): the person who is
confined either is conscious of the confinement or is harmed by the confinement.

Exception:

The requirement that a person is conscious of the confinement or harmed by the confinement
does not apply if the confinement is of a child or mentally incompetent person. In such instances,
one may be liable for false imprisonment even if the confined person is completely unaware of
the confinement.

Limitations on Firm Offers


The following question was asked by a reader in the comments of the blog:

If a merchant expressly states in signed writing that the offer is open for 6-months, is the offer
(1) irrevocable for the first 3 months but subsequent revocable between 3-6 month period OR (2)
irrevocable for the full 6-month period?

Response:

An important distinction between the common law and the UCC is how each treats a promise not
to revoke an offer. Under the common law, offers are revocable unless consideration is given to
keep the offer open (though you should always keep in mind substitutes for consideration, such
as promissory estoppel).

Under UCC article 2, however, offers made by merchants that are written and signed by the
merchant giving assurances that the offer will be held open, will be irrevocable without
consideration for the time stated, or for a reasonable time, but not to exceed 3 months. The 3
month rule applies whether or not a particular time period is stated in the offer. Thus, if an offer
stated that it would be irrevocable for six months, the offer would only be irrevocable for 3
months.

Note another angle tested on the MBE: If a written and signed offer by a merchant states that it
will be irrevocable for only 1 month, the period of irrevocability will last only one month, rather
than the 3 months that would apply if no time period were stated.

You might also like